Return to flip book view

Property

Page 1

AMERIBAR BAR REVIEW REAL PROPERTY MBE OUTLINES REAL PROPERTY 1 REAL PROPERTY INTRODUCTION Unless otherwise indicated, this outline covers majority legal principles. The MBE generally tests majority rules unless a question expressly provides that the minority rule applies or the question otherwise would not make sense. I. OWNERSHIP OF REAL PROPERTY A primary purpose of property law is defining and establishing the nature and extent of property ownership. A useful means of understanding the concepts of ownership is looking at a parcel of real property as a bundle of sticks that represent interests, rights, or estates in the property. An individual or entity that has complete ownership, possession, and use of the parcel holds the entire bundle. If that ownership is incomplete in some respect, then the individual or owner will not hold all of the sticks (interests, rights, or estates in the property). The right of ownership is the greatest interest of all. Other lesser interests include the right to possession and use, the right of transfer, and the right to exclude others from a parcel of real property. A. Present Estates and Future Interests 1) PRESENT ESTATES a) General Considerations (1) Rights and Interests in Property, whether Possessory or Not Present estates concern the types of rights and interests in real property that are held by individuals or entities. Present estates may either be possessory or non-possessory because an ownership interest does not necessarily require or involve possession and use of the property. (2) Transfer of Interest in Property, whether Possessory or Ownership A first party may hold title to a parcel of real property, which constitutes an ownership interest. A second party may simultaneously have a right to occupy that parcel, which is a possessory interest. If the second party seeks to transfer that interest to a third party, the second party is limited to transferring a possessory interest and cannot transfer an ownership interest. A party may only transfer the rights it possesses in property. (3) Determining Interest Type One should consider the following factors when determining the nature and extent of an interest in property: • the language used to establish an estate; • the nature and type of rights the grantor conveys; • the duration of the estate; and • the involvement of any future interest. (4) Types of Present Estates

Page 2

AMERIBAR BAR REVIEW REAL PROPERTY MBE OUTLINES REAL PROPERTY 2 Present estates include freehold estates and qualified estates. A fee simple estate is a freehold estate. Qualified estates, on the other hand, include defeasible fees simple, life estates, estates for a period of years, and fees tail. Many exam questions provide the language of a conveyance and require the examinee to identify what type of interest was created. Therefore, it is important for you to understand the different types of interests and how they are created. b) Fees Simple (1) Fee Simple Absolute (a) Nature and Characteristics Most real property owners hold their property in fee simple absolute (also referred to as fee simple). A fee simple absolute is the most substantial type of property interest for three reasons. First, it neither involves, nor is subject to, a future interest. Second, its duration is for a potentially infinite period. Third, its holder may exclusively occupy and enjoy the parcel of real property. (b) Creation Grantors typically transfer property by a conveyance in a written instrument. The usual language in a conveyance that creates a fee simple absolute is either “from a grantor to a grantee,” or “from a grantor to a grantee and his heirs.” It may also state “from Grantor to Grantee in fee simple absolute” or other similar language. A conveyance of a fee simple interest is neither qualified in any way nor limited by any other interest. (c) Rights of Possession and Use The grantee of a fee simple absolute has the exclusive rights of possession and use. The grantee may share these rights with others, including invitees and licensees, or may transfer the rights to third parties. c) Defeasible Fees (1) Nature and Characteristics A defeasible fee simple is an estate in land in which the holder possesses fee simple title, but the title is subject to being divested upon the occurrence of a certain condition. This type of interest is also called a defeasible fee or a qualified fee. Thus, a conveyance of a defeasible fee simple includes language that may limit its duration. For example, a grantor may convey this type of estate for charitable purposes or to ensure that a parcel of real property will be used pursuant to the grantor’s wishes for some duration. A defeasible fee simple creates an accompanying future interest. (2) Creation There are three main types of defeasible fee simple estates. The three types of defeasible fees include: 1) a fee simple determinable, 2) a fee simple subject to a condition subsequent, and 3) a fee simple subject to an executory interest. On the MBE, you must be able to identify each of them. (a) Fee Simple Determinable A fee simple determinable is an estate that ends automatically if the condition specified in the conveyance occurs. Upon the happening of the event, the interest will automatically revert back to the grantor or the heirs of the grantor. Question Key 27,30,57,114 Question Key 7,57,169

Page 3

AMERIBAR BAR REVIEW REAL PROPERTY MBE OUTLINES REAL PROPERTY 3 A possibility of reverter is the accompanying future interest created with a fee simple determinable and held by the grantor. The grantor must use durational language to create this estate. The usual language that creates a fee simple determinable may include: “to Grantee and her heirs for as long as [or, alternatively, so long as, until, or while] a conveyed parcel of real property is used for a public park [or alternatively some other purpose].” Unlike in a fee simple subject to a condition subsequent, set forth below, in a fee simple determinable the possibility of reverter automatically vests legal possession of the parcel in the grantor when the condition that limits the conveyance is no longer satisfied. In order to create a fee simple determinable, the durational language in the granting instrument must be clear and unequivocal. Consider the following example: QUESTION Anders conveyed her only parcel of land to Burton by a duly executed and delivered warranty deed, which provided: “To have and to hold the described tract of land in fee simple, subject to the understanding that within one year from the date of the instrument said grantee shall construct and thereafter maintain and operate on said premises a public health center.” The grantee, Burton, constructed a public health center on the tract within the time specified and operated it for five years. At the end of this period, Burton converted the structure into a senior citizens’ recreational facility. It is conceded by all parties in interest that a senior citizens’ recreational facility is not a public health center. In an appropriate action, Anders seeks a declaration that the change in the use of the facility has caused the land and structure to revert to her. In this action, Anders should: (A) win, because the language of the deed created a determinable fee, which leaves a possibility of reverter in the grantor. (B) win, because the language of the deed created a fee subject to condition subsequent, which leaves a right of entry or power of termination in the grantor. (C) lose, because the language of the deed created only a contractual obligation and did not provide for retention of a property interest by the grantor. (D) lose, because an equitable charge is enforceable only in equity. The correct answer is choice (c). This question tests the requirements for a defeasible fee simple. A defeasible fee simple is an estate in land in which the holder possesses a fee simple title, but the title is subject to being divested upon the occurrence of a certain condition. Thus, a conveyance of a defeasible fee simple includes language that may limit its duration. This durational language must be clear and unequivocal. When conveyances include language explaining the purpose of the transfer, the vast majority of courts will hold the language as precatory – not intended to have any legal significance – and will interpret the conveyance to have transferred all the interests the grantor owned. Thus, if the grantors owned a fee simple, the courts presume that is what they intended to convey. This result follows from the presumption against forfeitures and protects the interests of the grantee, placing the burden on grantors to be clear if they intend to retain a future interest in the property. Here, the conveyance lacks any durational language. Although it may, at first glance, appear to be a defeasible estate, the language in the deed created only a contractual obligation and did not provide for retention of a property interest by Anders.

Page 4

AMERIBAR BAR REVIEW REAL PROPERTY MBE OUTLINES REAL PROPERTY 4 (b) Fee Simple Subject to Condition Subsequent A fee simple subject to a condition subsequent is a defeasible fee estate that does not end automatically upon the occurrence of the condition specified in the conveyance. The holder of the interest must claim the future interest in the estate. The grantor holds the accompanying future interest, which is called a “right of reentry” or “right of entry.” The language used to create this interest would include: “to Grantee, but if Grantee sells alcohol on the property, then Grantor has the right of reentry.” Remember, in order to create a fee simple subject to a condition subsequent, the grantor must provide language containing a right of reentry upon the happening of a specified event. The language must expressly state that if the grantee does not use the parcel as designated, then the grantor may reenter and occupy the parcel. The occurrence of the condition authorizes the grantor to reenter the parcel. The grantor may or may not ultimately decide to do so. To exercise the right, the grantor must take affirmative steps in order to regain possession and use of the parcel. (c) Fee Simple Subject to an Executory Interest A fee simple subject to an executory interest is a fee simple estate that is subject to divestment in favor of someone other than the grantor if a specified event happens. For example, Grantor may deed his property to Grantee providing, “To Grantee and his heirs, but if the property is ever used as a liquor store, then to Edward.” If Grantee violates the terms of the deed, the property would then go to a third party (in this case, Edward). An executory interest, which is further explained below, is an interest that automatically transfers to another grantee upon the occurrence of a certain enumerated change in the use of the specified property. The first grantee’s interest in the estate automatically terminates if the grantee does not fulfill the condition of the grant. The second grantee then obtains use and possession of the parcel. (3) Rights of Possession and Use Under all three types of defeasible fees, the original grantee retains the right of possession and use, if it is not legally transferred, for as long as the terms of the language of conveyance are satisfied. If the grantee does not fulfill the terms of the conveyance, then the grantor may regain possession and use of the parcel under either a fee simple determinable with a possibility of reverter or a fee simple subject to a condition subsequent. If the grantee does not satisfy the terms of a grant with respect to a fee simple subject to an executory interest or limitation, the law automatically terminates the grantor’s and the first grantee’s interests in the parcel and transfers the interests to a second grantee who obtains the rights of possession and use. d) Life Estates (1) Nature and Characteristics A life estate is a possessory interest in a parcel of real property that lasts for the life of some person. A conveyance of a life estate provides the grantee a right to occupy the parcel for a lifetime. The measuring lifetime may either be that of the grantee or that of a third person. Such a grantee is considered a life tenant. (2) Creation The usual language that creates a life estate may include: “to a grantee for the grantee’s life,” or “to a grantee for the life of [another person],” or “to a first grantee for life, but [if something specific occurs], then to a second grantee.” Question Key 7,22,45,52,58,59,92,140,168

Page 5

AMERIBAR BAR REVIEW REAL PROPERTY MBE OUTLINES REAL PROPERTY 5 (3) Classification of Interests A life estate creates a corresponding future interest that may become possessory upon the end of the measuring lifetime. That future interest may either be a reversion or a remainder (reversions and remainders will be covered in another section of the outline). (a) Pur Autre Vie A life estate pur autre vie is a life estate where the measuring lifetime is that of someone other than the life tenant. For example, suppose that a grantor conveys a life estate to a first grantee for that grantee’s life. Ten years later, the first grantee conveys her interest to a second grantee. The second grantee possesses a life estate pur autre vie for the life of the first grantee. (b) Term of Years An estate for a term of years is another similar type of possessory estate that a grantee holds for the duration set forth in the conveyance. A term of years lease specifies both a beginning and an ending date. For example, suppose that a grantor conveys a possessory interest to a grantee for five years. The grantee possesses an estate for that term of years. (c) Determinable Life Estate An instrument that also contains a limitation creates a determinable life estate. The limitation might, for example, automatically terminate the life estate owner's interest in the property prior to the death of the measuring life. For example, suppose Harold conveys Blackacre “to Wanda for life or until she remarries.” The limitation is valid, and Wanda possesses a determinable life estate. (4) Rights of Possession and Use A grantee of a life estate, a life tenant, retains the rights of possession, use, and profits from the parcel for the duration of the measuring life. A grantee of an estate for a term of years retains the rights of possession, use, and profits from the parcel for that number of years. (a) Encumbrances A life tenant may only encumber a parcel of real property by a mortgage, lease, or other creditor interest to the degree of the tenant’s interest in that parcel. A life tenant, who takes possession of real property that is encumbered by a mortgage, unless otherwise provided in the conveyance, will be responsible for paying the interest on that mortgage. The holder of the reversionary interest has the duty to pay the principal. (b) Waste A reversionary interest holder, or remainderman, has the right to receive a parcel in essentially the same condition that a life tenant found it upon taking possession of it. Accordingly, a life tenant possesses a duty not to commit waste that would unreasonably impair the value of the parcel when a reversionary interest holder takes possession of it. An example of waste would be destruction of the parcel or any structures upon it. The law prohibits the life tenant from committing several types of waste. (i) Ameliorative Waste

Page 6

AMERIBAR BAR REVIEW REAL PROPERTY MBE OUTLINES REAL PROPERTY 6 Ameliorative waste occurs when a life tenant improves the parcel’s value in a manner that violates the intent of its grantor. In order to take any lawful action to improve a parcel subject to a life estate, the life tenant must obtain the consent of all reversionary interest holders. (ii) Permissive Waste Permissive waste results from a life tenant’s failure to fulfill legal duties. Those duties include maintaining the parcel with its structures and making property tax and mortgage payments. Those payments are limited to the degree of profits or income from the parcel. If there are no profits, the payments are limited to the amount of the parcel’s reasonable rental value. The life tenant may insure the parcel, but the common law does not impose a duty to insure. (iii) Affirmative Waste Voluntary or affirmative waste consists of a life tenant’s activity that diminishes the parcel’s value. Demolishing an existing structure on the parcel that is not in a condemned condition would be an example of affirmative waste. Another type of such activity would be an unauthorized taking of natural resources from the parcel. (iv) Exceptions (A) Open Mine Doctrine The open mine doctrine allows a life tenant to continue operating a previously functioning mine on the parcel of real property to extract natural resources. (B) Owners in Possession Owners of a fee simple interest who are in possession of it are not subject to the law of waste. (5) Legal and Equitable Remedies If a life estate tenant, a tenant for a term of years, or someone in legal privity with such a tenant commits waste, then the grantor or the holder of the reversionary interest may, according to the common law or state statutes, seek the following remedies: • forfeiture of possession; • legal damages based on the parcel’s diminished value or the cost of restoring the parcel’s original condition; or • equitable remedies of injunctions or accountings. e) Fee Tail (1) Nature and Characteristics A fee tail is an estate in land that is limited in inheritance to a particular class of heirs of the grantee. Generally this type of estate no longer exists because most jurisdictions have abolished it by law. A fee tail is similar to, or in the nature of, a fee simple absolute. The goal in creating a fee tail is to ensure that an estate remains within a family’s ownership. (2) Creation The following types of language of conveyance may effectively establish a fee tail: “to a grantee and the heirs of her body.” Thus, a fee tail will endure as long as the grantee continues to have a bloodline. Question Key 45

Page 7

AMERIBAR BAR REVIEW REAL PROPERTY MBE OUTLINES REAL PROPERTY 7 (3) Classification of Interests A fee tail establishes a future interest of a reversion in a grantor that becomes effective when and if all of the grantee’s heirs are deceased. (4) Rights of Possession and Use A grantee of a fee tail and the grantee’s heirs obtain the rights of possession, use, and profits from a parcel during the successive lifetimes of the grantee and each of the heirs. 2) FUTURE INTERESTS a) Definition A future interest is a present non-possessory estate or interest in a parcel of real property that may become a possessory interest in the parcel at some time in the future. A future interest is a right to receive possession of the parcel at a future time. A future interest follows a qualified estate and may be created in a grantee or in a grantor. There are several different types of future interests. An exam question may require an examinee to determine what type of interest has been created by a specific conveyance. b) Reversions (1) General A grantor’s conveyance of something less than a fee simple interest to a grantee creates a reversion interest. For example, if a grantor conveys a life estate or an estate for a term for years, the grantor reserves a reversion interest in the estate. (2) Nature and Characteristics A reversion is a grantor’s future interest that is derived from a conveyance of an interest in real property that is of a lesser quantity than the interest that the grantor owns. (3) Creation A grantor may create a reversion by conveying to a grantee an interest in real property that is of a lesser quantity than the one that the grantor owns. For example, a grantor that owns a parcel in fee simple absolute might convey a life estate or a fee tail to a grantee and retain a reversionary interest in fee simple absolute. The grantor need not explicitly reserve a future interest in a conveyance for it to exist. Operation of law, rather than the express terms of the deed, creates a reversion. (4) Rights of Possession and Use By retaining a future interest, a grantor relinquishes a present right to possess and use a parcel of real estate but reserves that right for the future. The grantee exercises a present right of possession and use of the parcel until it reverts to the grantor pursuant to the conveyance’s terms. c) Remainders, Vested and Contingent (1) General Question Key 22,59,96,103,123,140,168 Question Key See questions keyed for present estates.

Page 8

AMERIBAR BAR REVIEW REAL PROPERTY MBE OUTLINES REAL PROPERTY 8 A remainder interest generally follows a life estate or an estate for a term of years. A remainder will never follow a fee interest. It only comes into existence at the natural expiration of the prior estate. For example, a second grantee possesses a remainder interest in the property when a grantor makes a conveyance to a first grantee for life, then to the second grantee. (2) Nature and Characteristics A remainder is a future interest that may be established in a grantee as a result of a prior estate’s termination. (3) Creation A single instrument, such as a deed, must establish both a future interest and a present interest in order to establish a remainder. A remainder may not prematurely end a proceeding estate. In other words, a remainder becomes a present possessory interest at the natural expiration of the prior possessory interest. Thus, the estates that a remainder may follow are life estates, estates for a term of years, or fee tails (estates that end naturally or at a pre-determined definite time). (4) Classification of Interests If a remainder is not vested, then it is contingent. The distinction between vested and contingent remainders is important for several purposes, including the application of the Rule Against Perpetuities. (a) Vested Remainders (i) General A remainder is a vested remainder if: 1) it exists in an ascertainable person or persons (i.e., specific and identifiable), and 2) it is not subject to an express condition precedent. For example, a grantor conveys an interest “to a first grantee for life, then to a second grantee.” The second grantee possesses a vested remainder because she is ascertainable, and no specific contingency qualifies the second grantee’s interest. The first grantee’s death is not a contingency but rather the natural expiration of the first grantee’s estate. (ii) Condition Subsequent A vested remainder cannot be created as a result of the occurrence of a condition precedent. The occurrence of a condition subsequent to the creation of a vested remainder, however, will cause the vested remainder to take effect and operate to become a present possessory interest (as opposed to a future interest). For example, a grantor conveys an interest to a first grantee for ten years, then to a second grantee. The second grantee’s interest is vested when the original conveyance creates the remainder, and the remainder takes effect as a present possessory interest when the ten year period ends. (iii) Defeasance or Divestment Vested remainders may be subject to defeasance or divestment upon the happening of a particular condition. For example, a grantor conveys an interest “to a first grantee for life, then to the second grantee and his heirs; if the second grantee dies before the first grantee, then to a third grantee and her heirs.” Through this written conveyance, the first grantee obtains a life estate. Under that conveyance, the second grantee receives a vested remainder subject to divestment because the second grantee’s death before the end of the first grantee’s life estate will eliminate the second grantee’s remainder. A vested remainder subject to defeasance or divestment is subject to the Rule Against Perpetuities. (A) Indefeasibly Vested A vested remainder may be indefeasibly vested. A vested remainder that is indefeasibly vested is a remainder that is not dependent upon any condition precedent or any condition subsequent that may completely or partially defeat the

Page 9

AMERIBAR BAR REVIEW REAL PROPERTY MBE OUTLINES REAL PROPERTY 9 remainder. If it cannot be defeated, then it is said to be indefeasibly vested. (B) Vested Subject to Complete Defeasance A vested remainder subject to complete defeasance is a remainder that can be completely defeated by the occurrence of a condition subsequent or other limitation. (iv) Subject to Open Vested remainders may be subject to open when a specified class of persons, such as children or grandchildren, holds a remainder and at least one member of the class is ascertainable. In that event, the number of members of the class may increase or decrease until all of its members have been ascertained. That may happen when the parents of the class members are unable to have children or have died. An open vested remainder is subject to the limitations of the Rule Against Perpetuities. A vested remainder subject to open is also called a vested remainder subject to partial defeasance. (A) Ascertainable Persons Generally, if a person is named and alive, the person is ascertainable (i.e., identifiable). A person’s heirs are not identifiable as such until the person’s death. Thus, if a person is not identifiable or not yet born, the person is not ascertainable as heir. For example, assume that a grantor conveys an interest to a first grantee for that grantee’s life, then to a second grantee’s children. If the second grantee has one child who is living at the time of conveyance, then that child is ascertainable and possesses a vested remainder subject to open. If the second grantee is alive, then the second grantee’s unborn children possess remainders that are contingent upon them being born. (b) Contingent Remainders (i) General A written conveyance establishes a contingent remainder when it contains either or both of the following conditions: 1) no grantee is specified or identifiable; or 2) the grantee’s receipt of the remainder interest is conditional. For the interest to be a contingent remainder, it must potentially become possessory at the natural expiration of the prior estate. For example, suppose Otis conveys Blackacre to “Bob for his life, then to Carl’s heirs.” If Carl is alive, his heirs cannot be specified or identified. Therefore, Carl’s heirs possess a contingent remainder. (ii) Conditional Element If a condition in a written conveyance must be fulfilled to establish a remainder interest, then the remainder may be contingent. In other words, if a remainder interest is subject to an express condition that must be met before the interest becomes possessory in a grantee, then it is contingent. For example, “to a first grantee for the first grantee’s life, then if a second grantee is alive and has two children at the time of her death, to the second grantee and her heirs.” The second grantee possesses a contingent remainder because she must fulfill a condition before she may take possession. (A) Contingent versus Vested If one clause in a written conveyance creates a remainder interest, and a separate and subsequent clause provides a condition that may eliminate the interest, the remainder is a vested remainder subject to divestment by a condition subsequent. For example, a grantor conveys an interest “to a first grantee for the first grantee’s life, then to a second grantee and her heirs, but if the second grantee is not employed as an attorney when the first grantee dies, then to a third grantee and her heirs.” The first grantee possesses a life estate. The second grantee possesses a vested remainder subject to divestment by a condition subsequent because the provision creating the property interest comes before a separate and subsequent clause that provides for the potential elimination of the interest. The third grantee possesses a remainder that is contingent upon the second grantee’s failure to satisfy the condition.

Page 10

AMERIBAR BAR REVIEW REAL PROPERTY MBE OUTLINES REAL PROPERTY 10 The way that a conveyance is written dictates the type of remainder created. For example, suppose the above conveyance was written as follows: “To a first grantee for the first grantee’s life, then if the second grantee is employed as an attorney when the first grantee dies, to the second grantee and her heirs.” In this example, the second grantee possesses a contingent remainder because the clause creating the condition appears before the clause granting the interest to the second grantee. The placement of commas or clauses can make all the difference. (iii) Destructibility of Contingent Remainders (A) Common Law (Minority) At common law, destruction of a contingent remainder occurs unless it vests at or before the expiration of a prior life estate. Assume that a grantor conveys an interest “to a grantee for life, then to the grantee’s children who reach the age of 30.” If at the grantee’s death, the grantee’s only child is 29, then that child’s remainder is destroyed, and the property reverts to the grantor. (B) Modern Law (Majority Rule) Statute or case law has abolished the harsh common law rule in a majority of jurisdictions. Today, a remainderman who fulfills a condition of a contingent remainder will generally be entitled to receive a remainder interest. For example, a grantor conveys an interest “to a grantee for life, then to the grantee’s children who reach the age of 30.” If the grantee’s only child is 29 at the grantee’s death, the child will be entitled to the property when and if she turns 30. In the interim, the grantor would be entitled to possession of the property. (5) Rights of Possession and Use A holder of a remainder interest lacks any present rights of possession and use of the parcel. The holder’s right of use only exists when the interest becomes possessory. (6) Legal and Equitable Remedies (a) Vested Remainders (Damages Allowed) The holder of a vested remainder may file a lawsuit to enforce its property interests and seek a recovery of damages or injunctive relief. (b) Contingent Remainders (Damages Prohibited) The holder of a contingent remainder may file a lawsuit to enforce its property interest and seek a recovery of injunctive relief. That holder may not recover damages. The holder may also file a lawsuit seeking an injunction to stop waste but not a lawsuit to recover damages for the waste. d) Executory Interests (1) Nature and Characteristics An executory interest cuts short the natural termination of a preceding estate. Executory interests may be either shifting or springing. (a) Shifting Executory Interest A second grantee’s shifting executory interest may cut off a first grantee’s interest. The interest shifts or transfers from the first grantee to the second grantee pursuant to some condition. For example, suppose Opel conveys Blackacre “to Arnold for so long as the premises are not used as a brothel, then to Charles.” Charles possesses a shifting executory interest. Question Key 76,91

Page 11

AMERIBAR BAR REVIEW REAL PROPERTY MBE OUTLINES REAL PROPERTY 11 (b) Springing Executory Interest A grantee’s springing executory interest may cut off a grantor’s interest. A springing executory interest is an interest that “springs” from the grantor to the grantee pursuant to a condition. For example, suppose Opel, owner of Blackacre in fee simple, conveys Blackacre “to Gertrude in five years,” or “to Gary two years after my death.” In these examples, the grantees’ executory interests cut short Opel’s fee simple interest. When the interests become possessory, they “spring” directly from the grantor to the grantee. (2) Creation (a) General A conveyance may create a springing or a shifting executory interest, which follows a fee interest, in a grantee. (b) Shifting Executory Interest A shifting executory interest is an interest that shifts by becoming possessory directly from one grantee to another. For example, a grantor conveys an interest to a first grantee for so long as the premises are not used as a brothel, then to a second grantee. The second grantee possesses a shifting executory interest because title passes from the first grantee to the second grantee upon fulfillment of the requisite condition. A shifting executory interest includes language similar to the following example: “To a first grantee and her heirs, but if a second grantee gets married, to the second grantee and her heirs.” (c) Springing Executory Interest A springing executory interest is an interest that becomes possessory directly from the grantor to an interest holder grantee. For example, a grantor conveys an interest “to a grantee when and if the grantee reaches the age of 30.” The grantee possesses a springing executory interest because title passes from the grantor to the grantee. In another example, a grantor conveys an interest “to a first grantee for that grantee’s life, then 10 years after the first grantee’s death, to a second grantee.” Title will pass from the grantor to the first grantee as a life estate holder, then back to the grantor for 10 years, and then finally to the second grantee. The second grantee possesses a springing executory interest because she obtains title from the grantor. (3) Classification of Interests An executory interest is an interest in a third party that always follows a fee interest. For example, a grantor conveys an interest to a first grantee for so long as the premises are not used as a brothel, then to a second grantee. The first grantee possesses a fee simple determinable because the happening of the event eliminates the grantee’s estate. The second grantee possesses an executory interest because, if it comes into existence, it cuts off a fee simple determinable interest. (4) Rule Against Perpetuities Executory interests are subject to the limitation of the Rule Against Perpetuities. e) Possibilities of Reverter, Powers of Termination (1) Nature and Characteristics

Page 12

AMERIBAR BAR REVIEW REAL PROPERTY MBE OUTLINES REAL PROPERTY 12 A fee simple determinable is an interest that is subject to potential reversion to the grantor if a certain event occurs. The two types of future interests that accompany a fee simple determinable are the possibility of reverter and the power of termination. (a) Possibilities of Reverter A possibility of reverter is a future interest that a grantor retains when the grantor conveys a fee simple determinable. For example, a grantor conveys an interest “to a grantee for so long as the premises are not used as a brothel.” The grantee’s interest is conditioned upon the premises not being used in a certain way. No additional event needs to occur before the grantor automatically regains possession of the premises other than a failure of the stated condition. Thus, the grantor retains a possibility of reverter. (i) The Rule Against Perpetuities Does Not Apply The Rule Against Perpetuities is not applicable to a possibility of reverter. (b) Powers of Termination A power of termination (i.e., a right of re-entry) is similar to a possibility of reverter. For example, the grantor conveys an interest “to a grantee, but if the premises are used as a brothel, the grantor reserves the right to re-enter and terminate the grantee’s interest.” The additional event of the grantor’s re-entry must occur for the grantor to retake the estate. In other words, with a power of termination, a grantor must take affirmative action in order for the estate to revert back to the grantor. Contrast this situation to an automatic transfer that would occur upon the happening of a condition with a possibility of reverter. (i) The Rule Against Perpetuities Does Not Apply The Rule Against Perpetuities is not applicable to a right of re-entry. (2) Creation (a) Possibilities of Reverter A conveyance may create a possibility of reverter in a grantor. It may only follow a fee simple determinable. For example, a grantor conveys an interest “to a grantee subject to a provision that if the parcel ceases to be used as a park, it will revert to the grantor.” If the grantee stops using the land as a park, then the interest automatically goes to the grantor, and the grantee’s interest ends. Sometimes, the operation of law, instead of the express terms of a deed, may create a possibility of reverter. (b) Powers of Termination A conveyance may create a power of termination or right of entry in a grantor. It may only follow a fee simple subject to a condition precedent. For example, a grantor conveys an interest “to a grantee subject to a provision that if the parcel ceases to be used as a park it will revert to the grantor.” If the grantee begins to build a house on the parcel, then the grantor may re-enter the premises and terminate the grantee’s estate. (3) Rights of Possession and Use (a) Possibilities of Reverter A grantor does not need to expressly communicate to a grantee that the grantor possesses a possibility of reverter in order to exercise it because of the possibility’s automatic nature.

Page 13

AMERIBAR BAR REVIEW REAL PROPERTY MBE OUTLINES REAL PROPERTY 13 (b) Powers of Termination A grantor needs to expressly communicate to a grantee that the grantor possesses a power of termination or right of entry in order to exercise it. f) Powers of Appointment (1) Nature and Characteristics The Restatement (Third) of Property states that a power of appointment is “a power created or reserved by a person (the donor) having property subject to his disposition, enabling the donee of the power to designate, within such limits as the donor may prescribe, the transferees of the property or the shares in which it shall be received.” (2) Creation Using the appropriate language in an instrument of conveyance including, but not limited to, a will may create a power of appointment. (3) Rights of Possession and Use The holder of a power of appointment may or may not have a right of possession and use in the property that is subject to transfer pursuant to the power. That right of possession and use will depend upon the extent of the holder’s interest in the parcel that is subject to the power of appointment. The holder possesses the right to appoint the recipients of property or to select who will receive property. The donor usually conveys the power by will or trust to a donee as the holder. A trustee of a trust may possess a power of appointment for the purpose of disposing of the property that is held in a trust. (a) General Power of Appointment The donee of a general power of appointment receives all rights to appoint the property to herself, her creditors, or any others. Therefore, if a power is exercisable in favor of the holder, it is a general power of appointment. A power of appointment is general unless the terms of its creation limit it. (b) Special Power of Appointment A special power of appointment is a power that is not exercisable in favor of a donee of this power, her estate, her creditors, or the creditors of her estate. With a special power of appointment, a holder of that power cannot appropriate the property into the holder’s own hands. The terms of creation of the holder’s exercise of that power must limit it in order for it to be a special power of appointment. (c) Exercise of Power of Appointment A power of appointment will be properly exercised if the holder intends to exercise the power. A common question is whether a power of appointment can be exercised in a general residuary clause of a will. In that event, a testatrix’s will bequeaths “the rest of the property in my estate or over which I possess a power of appointment to a holder of the power of appointment and his heirs.” In the absence of a requirement in that grant of the power that specific reference to the power is necessary to exercise the power of appointment, a general residuary clause may provide a basis for an effective exercise of the power only if 1) the power is a general power or 2) the testatrix's will manifests an intention to include the property subject to the power. (d) Subject to the Rule Against Perpetuities Question Key 149

Page 14

AMERIBAR BAR REVIEW REAL PROPERTY MBE OUTLINES REAL PROPERTY 14 A power of appointment is subject to the Rule Against Perpetuities. It must become exercisable within the perpetuities period. Any interest that is created by the exercise of a special power of appointment is valid only if the interest will in all events vest or fail to vest no later than 21 years after the death of a life in being at the time the power was executed. g) Acceleration of Future Interests If a party disclaims a life estate or other future interest, the issue may arise of whether succeeding interests or estates accelerate in possession or enjoyment or whether the disclaimed interest must be marshaled to await the actual happening of a contingency. For example, assume that a grantor conveys an interest “to a first grantee for that grantee’s life, then to a second grantee.” If the first grantee disclaims the interest, must the second grantee await the first grantee’s death to take possession? Generally, under those circumstances, remainder interests accelerate, and the “future interest” takes effect as if the disclaiming grantee had predeceased the grantor, and the contingent event was satisfied. A grantor may draft a written conveyance to avoid such an acceleration of future interests. B. Co-tenancy The term co-tenancy refers to concurrent ownership of an estate. There are three main concurrent estates in property: 1) tenancy in common; 2) joint tenancy; and 3) tenancy by the entirety. 1) TYPES: TENANCY IN COMMON, JOINT TENANCY, AND TENANCY BY THE ENTIRETY a) Tenancy in Common (1) Nature and Characteristics A tenancy in common is a form of concurrent estate in which each owner, referred to as a tenant in common, owns separate and distinct shares that may differ in proportion. The tenancy in common is the default form of concurrent estate. A tenancy in common exists when at least two co-tenants possess an ownership interest in the same parcel(s) of real property. The following factors exemplify this estate: • each co-tenant’s interest may, but need not, be equal in extent; • each co-tenant’s right to use and enjoy the property is equal; and • none of the co-tenants possess a right of survivorship. For example, suppose Bob and Carl own Blackacre as tenants in common. Bob may own a 60% interest, and Carl may own a 40% interest. Bob and Carl, however, possess the equal right to use and enjoy Blackacre. In a tenancy in common, none of the co-tenants possess a right of survivorship. Therefore, a co-tenant’s ownership interest may pass by intestacy or bequest. (2) Creation (a) Unity of Possession In order to create a tenancy in common, there must be unity of possession. Unity of possession means that both co-tenants must have an equal right to enjoy and possess the entire parcel when the tenancy in common is created. (b) Other Means of Creation (i) Presumption of Creation Question Key 6,11,15,31,39,48,53,62,74,77,88,104,113,120,121,133,135,145,153,160

Page 15

AMERIBAR BAR REVIEW REAL PROPERTY MBE OUTLINES REAL PROPERTY 15 Generally, a presumption exists that a conveyance to at least two unmarried individuals establishes a tenancy in common. For example, suppose O conveys “Blackacre to Edward and Josephine.” Pursuant to the presumption, the conveyance establishes a tenancy in common. (A) Overcoming Presumption The typical manner of overcoming this presumption is by including in the deed’s words of conveyance the phrase “joint tenants” or “joint tenancy” and the word “survivor” or “survivorship.” Sheldon F. Kurtz, Moynihan’s Introduction to the Law of Real Property 282 (4th ed. 2005). Some case law finds that the word “jointly” in a deed is sufficient to overcome the presumption of a tenancy in common. William B, Stoebuck & Dale A. Whitman, The Law of Property 185–86 (3d ed. 2000). Other case law provides that the use of the word “jointly” in the deed merely shows the grantor’s intent that the grantees “own together” and does not make the grantees joint tenants with the right of survivorship. Id. Use of the phrase “equally, to share and share alike” in conjunction with the word “jointly” may indicate a joint tenancy if it evidences the grantor's intent to grant each grantee an equal interest in the real property. This constitutes another key indicator of a joint tenancy that is not essential for a tenancy in common. Suppose, for example, that a grantor duly executes a deed “jointly in fee to my sons, Bob and Tom, equally, to share and share alike.” In this example, the grant language should be analyzed in light of other relevant facts and the foregoing rules in order to determine whether the grantor intended to create a tenancy in common or a joint tenancy. (ii) Creation by Succession A transfer resulting from intestate succession may create a tenancy in common. (iii) Automatic Creation A tenancy in common may arise by default when a divorce of married individuals terminates a tenancy by the entirety. That subtopic is discussed below. (3) Classification of Interests (a) Distinguishing “Tenants” from “Co-tenants” The term “tenant” as used in the concurrent estate context does not have the same meaning as that of a tenant who rents or leases premises from a landlord. The term “co-tenant” means an owner of premises in common with other co-tenants. (4) Interest Is Transferable or Descendible If a tenancy in common only consists of two co-tenants, the death of one of them does not terminate the deceased person’s tenancy. Rather, a deceased co-tenant’s ownership interest is descendible to his heirs. The heir becomes a co-tenant in common. b) Joint Tenancy (1) Nature and Characteristics A joint tenancy exists when at least two co-tenants own a parcel subject to a right of survivorship. The right of survivorship associated with a joint tenancy is the critical distinction between the joint tenancy and other concurrent estates. As with tenants in common, joint tenants possess the right to concurrently enjoy and possess the parcel. (2) Creation

Page 16

AMERIBAR BAR REVIEW REAL PROPERTY MBE OUTLINES REAL PROPERTY 16 (a) Four Unities The common law requires the existence of four unities to create a joint tenancy. These unities are: (1) unity of interest, (2) unity of title, (3) unity of time, and (4) unity of possession. All joint tenants must acquire an interest or estate by the same conveyance, commencing at the same time, and hold it for the same term of undivided possession. (i) Unity of Interest The term “unity of interest” refers to the necessity that all joint tenants have interests of the same duration, and accordingly, one cannot be a joint tenant for life and another joint tenant for years. 2 Tiffany Real Prop. § 418 (3d ed. 2001). There is no requirement of equality of interest (i.e., the same proportion). (ii) Unity of Title The joint tenants must receive their interest in land under the same document or conveyance. (iii) Unity of Time Each of the joint tenants’ titles must vest at the same time. Suppose, for example, that Blackacre was left to three brothers “upon attaining the age of 30.” If they are of different ages, and therefore will attain the age of 30 at different times, then no unity of time would exist. (iv) Unity of Possession As with tenants in common, the joint tenants have a right to possess the entire parcel and may not exclude each other. (3) Rights of Possession and Use (a) Inter Vivos Transfer A joint tenant’s attempt to make an inter vivos transfer of an interest in a joint tenancy cannot be valid because it would violate the four unities. (4) Right of Survivorship When one of the joint tenants dies, that tenant’s ownership interest automatically passes to a single surviving joint tenant or pro rata among all surviving joint tenants. This right of survivorship is the primary distinguishing feature of a joint tenancy that is lacking in a tenancy in common. If a single joint tenant survives, then the “joint” tenancy is ended. c) Tenancy by the Entirety (1) Nature and Characteristics A tenancy by the entirety is a concurrent estate held by spouses. In a tenancy by the entirety, each spouse owns an undivided whole of the property, along with the right of survivorship. Upon the death of one of the spouses, the survivor is entitled to the entire estate. A minority of the states recognize a tenancy by the entirety. (2) Creation

Page 17

AMERIBAR BAR REVIEW REAL PROPERTY MBE OUTLINES REAL PROPERTY 17 In a jurisdiction recognizing the interest, a tenancy by the entirety may be created by a conveyance of property to spouses clearly expressing an intention to create a tenancy by the entirety. In some states, a conveyance of property to a married couple automatically creates a tenancy by the entirety. (3) Classification of Interests A tenancy by the entirety is a type of co-tenancy because more than one person has an ownership interest in a concurrent estate. A tenancy by the entirety is only available for, and applicable to, a married couple. (4) Rights of Possession and Use (a) Right to Transfer Either spouse may transfer rights of possession and survivorship of the property. For example, a spouse may convey a fee simple interest in property held as a tenancy by the entirety. (i) Conveyance Cannot Defeat Right of Survivorship Neither spouse may defeat the other’s right of survivorship by making a conveyance of the spouse's interest in the tenancy by the entirety to a grantee. For example, although a spouse may convey a fee simple interest in the property, a spouse cannot convey the spouse's interest in the tenancy by the entirety to a third person (ostensibly to defeat the other spouse's right of survivorship). An attempted conveyance of a spouse's interest in the tenancy by the entirety would fail and the property would remain as a tenancy by the entirety. (b) Right of Survivorship A tenancy by the entirety provides a right of survivorship so that the entire estate transfers to the surviving spouse upon one spouse’s death. (c) Termination Several events may terminate a tenancy by the entirety including the death of either spouse, mutual agreement, or the termination of the marriage. (5) Legal and Equitable Remedies (a) Majority Rule In a majority of states that recognize tenancies by the entirety, one spouse’s creditor may not acquire an interest in the estate of a tenancy by the entirety. (b) Minority Rule In a minority of states that recognize tenancies by the entirety, a creditor may acquire an interest in one spouse’s ownership interest but may not encumber the other spouse’s ownership interest. In those states the creditor may not interfere with the right of survivorship. 2) RIGHTS AND OBLIGATIONS OF CO-TENANTS Regardless of the type of estate, co-tenants possess certain rights regarding the property, which they are free to modify through an agreement among themselves.

Page 18

AMERIBAR BAR REVIEW REAL PROPERTY MBE OUTLINES REAL PROPERTY 18 a) Partition A co-tenant (tenant in common or joint tenant) may bring an action for partition to divide property and obtain a final accounting of all co-tenants’ respective interests in the property. A co-tenant may seek and obtain a partition in two ways: 1) by voluntary written agreement; or 2) through a partition action by a tenant in common seeking judicial proceedings to obtain an involuntary partition. In such an action, the tenants may also seek the equitable remedy of an accounting to determine their respective financial interests relative to the estate. (1) Types of Partition There are two types of partition: 1) partition in kind by actually dividing the parcel; and 2) partition by sale by selling the parcel and dividing the profits. Partition in kind is preferred over partition by sale, but a partition by sale may occur when it is not possible to divide the parcel fairly. (a) Partition Sale In a partition sale, a co-tenant whose efforts and funds resulted in improvements to a parcel possesses the right to recover the parcel’s increase in value that resulted from the improvements. That tenant has no right to recover those improvements’ actual costs out of the sale proceeds. If other co-tenants contributed efforts and funds toward making the improvements, then they also possess the right to a pro rata recovery of the parcel’s consequent increased value. b) Severance A severance of joint tenancy is the process of changing a joint tenancy into a tenancy in common. (1) Severance by Mortgage A severance may occur by the taking of a mortgage on a parcel of real property that is subject to a joint tenancy. Whether the mortgage results in a severance depends upon whether the state is a title theory or lien theory state. In a lien theory state, the taking of a mortgage merely places a lien on the property, leaving the joint tenancy undisturbed. Conversely, in a title theory state, the mortgage actually conveys title to the mortgagee until the mortgagor pays the mortgage. In such states, the taking of a mortgage by one owner breaks the joint tenancy. (2) Severance by Conveyance A severance may occur as the result of a joint tenant making a conveyance. For example, suppose one of the joint tenants conveys his interest in the parcel to a third party. In that event, the joint tenancy transforms into a tenancy in common. A lease was treated as a conveyance under the common law. Therefore, if a joint tenant conveyed a lease interest, the joint tenancy severed. Most states follow the common law approach (but a minority hold that a lease does not sever a joint tenancy). (a) Severance by Contract A severance may occur as the result of a joint tenant signing a contract to sell an interest in a parcel to a buyer. Under those circumstances, the equitable conversion doctrine applies. This outline addresses the equitable conversion doctrine later. c) Relations among Co-tenants (1) Rights of Possession and Use

Page 19

AMERIBAR BAR REVIEW REAL PROPERTY MBE OUTLINES REAL PROPERTY 19 A tenant possesses an unrestricted right of access to the real property. If a co-tenant wrongfully excludes another from using the property, the excluded co-tenant may bring a cause of action for ouster. The ousted tenant may receive fair rental value of the property for the time of dispossession. Co-tenants may exercise these rights either concurrently or separately. Their separate and disproportionate exercise of those rights usually does not affect the validity or availability of their respective rights. For example, although Co-tenant A owns 30% of a parcel with Co-tenant B who owns 70% of the parcel, both co-tenants equally share the right of possession and enjoyment of the estate. (a) Boundary Disputes A co-tenant in possession may, without any input from an out-of-possession co-tenant, handle situations such as boundary disputes. (2) Costs Each of the co-tenants may be separately assessed his ownership interest in the property maintenance expenses, insurance, and taxes. A co-tenant possesses a right of contribution from the other co-tenants for the costs of owning the property. Co-tenants may be required to contribute to the payment of expenses, such as property taxes and mortgages, on the entire property. (a) Right of Reimbursement for Expenses A co-tenant in possession of a parcel possesses the right to reimbursement from a co-tenant who was out of possession of the parcel for a pro rata amount of the payments that were made for property maintenance, insurance, and taxes. In a partition sale, that amount may be credited towards and added to the paying co-tenant’s share of the proceeds. That amount is subject to an offset based on the value of the paying co-tenant’s use and enjoyment of the property. (b) No Reimbursement for Improvements Co-tenants are not responsible for reimbursing a co-tenant for the costs of improvements to the parcel. A first co-tenant lacks a right to reimbursement from any other co-tenants who decline to help pay for the cost of improvements to the parcel of real property. (i) Enhancements and Devaluations If a co-tenant adds something that enhances the value of the property, the co-tenant generally possesses no right to demand that the other co-tenants share in the cost of this enhancement. This is the rule even if the enhancement results in greater profits for the property. Nonetheless, if the property is partitioned, then the co-tenant possesses the right to recover the value added by the improvement to the property. If, however, the co-tenant's enhancements decrease the value of the property, then the co-tenant will be responsible for the decrease in value. (3) Income A co-tenant possesses the right to an accounting of all profits derived from the property. If the property generates rent, for example, each co-tenant has the right to a proportion of the income. (a) Profits When profits are derived from a parcel, all of the co-tenants possess a right to receive a percentage of the profits according to their ownership interest in the common estate.

Page 20

AMERIBAR BAR REVIEW REAL PROPERTY MBE OUTLINES REAL PROPERTY 20 (b) Rent If one of the co-tenants is in exclusive possession of the parcel, then the other co-tenants possess no right to receive rent from the one co-tenant. Also, a co-tenant in possession has no obligation to share any profits that result from that co-tenant’s efforts with another co-tenant who did not exercise possession of the estate. (i) Exception for Ouster A co-tenant possesses a right to recover rent, however, if the co-tenant is subject to an ouster from the parcel due to a wrongful exclusion. Ouster results when a first co-tenant makes a claim of ownership against a second co-tenant. Making a claim of adverse possession during an existing partition action between them will not constitute an ouster. (4) Outstanding Title Title to a parcel of real property that is subject to a co-tenancy may become outstanding as a result of either a foreclosure proceeding or tax sale. One co-tenant may acquire title to the parcel, and other co-tenants may assert and regain their interest in the parcel. (a) Mortgage Foreclosure A co-tenant who acquires title to the parcel in a mortgage foreclosure sale may recover from each co-tenant who is obligated on the mortgage their ownership interest in its debt. Likewise, the co-tenants may recover their ownership interest by reimbursing the purchasing co-tenant. (b) Tax Sale A co-tenant who acquires title to the parcel in a tax sale may recover from each co-tenant his ownership interest in the tax debt. Likewise, the co-tenants may recover their ownership interests by reimbursing the purchasing co-tenant. C. Landlord-Tenant Law Landlord and tenant law concerns leasehold estates when a landlord owns real property and a structure on that property while a tenant makes a payment in return for use and possession of the property and the structure that is upon it (i.e., a premises). 1) TYPES OF TENANCIES a) Periods of Tenant's Possession of Leasehold Estate, per Lease Agreement A tenant is entitled to possession of a landlord’s rented premises for a period that is agreed upon under a lease agreement (“lease”). Such a rental contract provides a tenant with a leasehold estate or interest in the premises. There are four primary types of leasehold estates. The most common types tend to be either periodic tenancies or terms of years. (1) Terms of Years A term of years may be referred to as an estate for years or a tenancy for years because its duration is fixed and established. Notice of termination is not required by either party for this type of lease because each party knows the termination date. For example, the lease might provide for a term of two years, at which time the lease would expire. The parties may add a notice requirement to the lease or provide advance notice as a matter of courtesy. A term of years lease normally cannot be terminated by the tenant prior to the end of the term. However, a tenant may terminate a term of years lease if the tenant is constructively evicted. Question Key 32,78,82,87,115,164,171

Page 21

AMERIBAR BAR REVIEW REAL PROPERTY MBE OUTLINES REAL PROPERTY 21 (2) Tenancies at Will A tenancy at will does not provide either party with any certainty of a fixed duration of a leasehold estate. Each of the parties may elect to terminate the estate at any time. A court will conclude that a tenancy at will exists if the parties neither have a lease nor a series of rental payments from which to imply that the leasehold existed for any term. (3) Holdovers and Tenancies at Sufferance A tenant who continues to occupy a landlord’s premises beyond the expiration of a leasehold interest has holdover possession as a tenant at sufferance. A tenancy at sufferance only continues until either the landlord or tenant agrees to a new term, or the landlord evicts the tenant. The landlord is entitled to recover rent from a tenant at sufferance for the period between the end of the existing tenancy and the eviction date. (4) Periodic Tenancies A periodic tenancy runs for a specified duration and may continue after that period for successive intervals until either party provides a notice of termination to the other party. (a) Notice of Termination Requirement For example, the lease might run from month to month and only require a notice of termination one month in advance of that period’s end date. If the lease runs from year to year, the common law requires that a party provide a notice of termination at least six months before the current year’s end. Although the common law allowed oral notice, some statutes require written notice. (b) Creation An agreement’s express terms may create a periodic tenancy, or a court may imply and enforce a periodic tenancy from the parties’ conduct with respect to the assessment and payment of rent. b) Effect of Incapacity or Death of Party to Lease In the majority of states, the incapacity or death of either party to a lease does not result in an automatic termination of the lease. Instead, the party’s rights, interests, and obligations in the lease pass to his estate. (1) Enforceability of Lease If a lease was valid, it is enforceable by or against the administrator of a deceased party’s estate, such as a personal representative. For example, an estate may enforce a valid purchase option in a lease. The majority rule provides that the option is not subject to the Rule Against Perpetuities. 2) POSSESSION AND RENT a) General Considerations A lease includes a tenant’s obligation to pay rent to a landlord in order to possess an exclusive right to occupy the premises. The tenant neither obtains nor acquires an ownership interest in the premises as a result of paying rent. b) Landlord’s Remedies (1) Tenant Out of Possession When a tenant fails to pay rent during the term of a lease and while not in possession of the premises, then a landlord

Page 22

AMERIBAR BAR REVIEW REAL PROPERTY MBE OUTLINES REAL PROPERTY 22 may respond in one of three ways: • rent the premises as soon as possible while holding the tenant liable for any deficiency during the lease term; • disregard the abandonment and hold the tenant liable for unpaid rent; or • acknowledge the abandonment that the tenant had communicated. (2) Tenant in Possession When a tenant fails to pay rent while in possession of the premises, then a landlord may either sue to recover the unpaid rent or seek to evict the tenant. (a) Self-Help Prohibited When a tenant fails to pay rent while in possession of the premises, a landlord may not exercise self-help remedies against the tenant, such as changing the locks or forcibly removing the tenant’s clothing or possessions from the premises. c) Destruction of Premises (1) Partial Occupancy If a tenant only occupied part of a premises that was entirely destroyed, then the tenant’s lease duty to pay rent ended when the destruction occurred. (2) Total Occupancy If the tenant occupied the entire premises that was completely destroyed, then the tenant’s lease duty to pay rent will continue if the leased premises was comprised of both land and a building. d) Duties of Landlord and Tenant (1) Landlord’s Duties (a) Covenant of Quiet Enjoyment An implied covenant obligates a landlord to provide a tenant with quiet enjoyment of the premises that the tenant uses, whether the premises are residential or commercial. That duty of the landlord includes: • exercising control of any common areas; • not evicting the tenant either actually or constructively; and • protecting against any superior claim of possession. (i) Actual Eviction A landlord is not permitted to wrongfully evict a tenant. If a landlord wrongfully evicts a tenant, the tenant may sue for trespass or ejectment, or may treat the lease as terminated. The tenant’s obligation to pay further rent terminates. Additionally, even if the landlord’s eviction of the tenant is only partial, the tenant’s obligation to pay rent ends. (b) Constructive Eviction (i) Untenantable Condition

Page 23

AMERIBAR BAR REVIEW REAL PROPERTY MBE OUTLINES REAL PROPERTY 23 “[A] constructive eviction arises where a landlord, while not actually depriving the tenant of possession of any part of the premises leased, has done or suffered some act that renders the premises untenantable and has thereby caused a failure of consideration for the tenant's promise to pay rent. In addition to proving that the premises are untenantable, a party pleading constructive eviction must prove that (1) the landlord caused the problem, (2) the tenant vacated the premises because of the problem, and (3) the tenant did not vacate until after giving the landlord reasonable time to correct the problem." Welsch v. Groat, 897 A.2d 710 (Conn. Ct. App 2006). For example, a landlord who refuses to provide heat in the winter or water to an apartment may be responsible for a constructive eviction of the tenant. The landlord's action, or failure to take action, must render the property untenantable (violating the covenant of quiet enjoyment). Some authority suggests that the tenant must prove that the landlord’s breach of duty caused a loss of the substantial use and enjoyment of the premises and that he gave the landlord adequate notice to permit the landlord to meet his duty to tenant. (ii) Traceability to Landlord In order to maintain an action for damages, the tenant must demonstrate that the uninhabitability resulted from the landlord's actions, not the actions of a third party, and that the tenant vacated the premises in a reasonable time. (c) Covenant of Possession A landlord has a duty to provide a tenant with physical possession of a leased premises pursuant to a lease. If the landlord fails to timely deliver possession of the premises due to a holdover tenant or fails to adequately prepare the premises for occupancy, then the tenant is entitled to recover damages for that breach of the lease. (d) Duty to Repair (i) No Duty Under Common Law Under the common law, there was no implied duty on the part of a landlord to repair leased premises. This duty arose only if it was expressly set forth in the lease. (ii) Duty Under Modern Approach (A) Residential Leases More recently, court have generally implied a duty to repair in residential leases either as part of a revised constructive eviction doctrine or based on an implied warranty of habitability. The justification for this shift from the common law is based on the economic disparity between landlord and tenant, as well as the fact that tenant lacks authority to make repairs to common areas. (B) Commercial Leases Courts are less inclined to imply a duty to repair in commercial leases where the tenant is often a valuable business and in a better position than the landlord to assess and make repairs. When courts have implied a duty to repair on the part of a landlord in a commercial lease, it is typically when the repair has been mandated by public authorities and involves work so substantial that it would not ordinarily fall within the tenant’s common law repair duty and/or the value of the repair would primarily inure to the landlord’s reversionary interest. Some courts have allowed constructive eviction claims by commercial tenants of office buildings based on repairs required in common areas. (2) Tenant’s Duties

Page 24

AMERIBAR BAR REVIEW REAL PROPERTY MBE OUTLINES REAL PROPERTY 24 (a) General Obligations A tenant possesses the primary duty to pay rent in order to be entitled to occupy a premises. A constructive eviction caused by the landlord excuses that duty. A tenant has a duty not to use the premises for illegal purposes. (b) Maintenance Obligations If a lease includes an explicit covenant that a tenant must maintain the premises in good condition, then the tenant is liable for any destruction or harm to the premises that occur while the tenant occupies the premises. If a lease lacks any terms or conditions with respect to the responsibility to repair the premises, then the tenant possesses an obligation not to commit waste and to keep the premises in reasonably good condition. (c) Waste May Impose Repair Obligations If a tenant’s negligent harm to a premises leads to extensively damaging the premises, then the courts will consider it waste. In that event, the tenant would possess the duty to repair and not be excused from the duty to pay rent. (i) Premises Liability Issue In certain states, a tenant may be liable for an invitee’s injuries on the leased premises despite a landlord’s promise to repair the premises. Other states permit an invitee to bring a lawsuit against a landlord if the landlord received written notice of the problem. 3) TRANSFERS BY LANDLORD OR TENANT Transfers by landlord or tenant can occur by means of assignment or subletting the premises. a) Nature and Characteristics (1) Assignment An assignment arises when a tenant transfers all or some of the leased premises to another for the remainder of the lease term, retaining no interest in the assigned premises. Thus, when a third party receives a tenant’s complete interest in a lease, including its remaining duration, the third party becomes an assignee. Absent an explicit provision that prohibits assignment, the parties to a lease may assign it by means of a writing that satisfies the Statute of Frauds. Without a written prohibition otherwise, a tenant may assign his interest without the consent of his landlord. (a) Enforceability of Prohibition The courts will not enforce a lease’s covenant against assignment when: • an initial landlord provides consent to an assignment, which serves as consent to subsequent assignments; or • an initial landlord waives the covenant by allowing an assignee to pay rent. (b) Purchase Option A split of authority exists on the issue of whether a party may assign a purchase option in a lease independently from the lease. (2) Sublease

Page 25

AMERIBAR BAR REVIEW REAL PROPERTY MBE OUTLINES REAL PROPERTY 25 A sublease occurs when a third party and a tenant enter into their own agreement that does not include all of the interests reflected in the lease. The third party becomes a sub-lessee for a duration that generally is less than the full term of the tenancy. Absent an explicit provision in a lease that prohibits subleasing, the parties to the lease may sublet a premises by means of a writing that satisfies the Statute of Frauds. A sublease is less than a complete assignment of rights. For example, a sub-lessor may retain possession for the last day prior to the expiration of the main lease. b) Privity of Contract Parties are each bound by a contract’s terms when they are in privity. Thus, a party may sue another party for breach of contract if the parties possess privity of contract. (1) Assignment Privity of a landlord and a tenant continues if either the landlord or the tenant assigns the lease to an assignee. As such, the tenant remains contractually obligated to pay the rent to the landlord. (a) Assignment by Landlord A landlord may assign his interest as lessor to an assignee. Although an assignee and a landlord will be in privity of contract, a tenant will not be in privity with the assignee. (b) Assignment by Tenant A tenant may assign his interest as lessee to an assignee. The assignee and the tenant will be in privity of contract. Generally, the landlord will not be in privity with the assignee. (2) Sublease With respect to a sublease, a tenant and a landlord stay in privity of contract, and a tenant and a sub-lessee continue in privity of contract. c) Privity of Estate A lease contains covenants that run with the land and to which a landlord and a tenant are bound. A tenant’s covenant is to pay rent. A landlord’s covenant is to maintain habitability. Thus, a party may sue another party for rent due or to maintain the premises as habitable if the parties possess privity of estate. (1) Assignment An assignee and a tenant are in privity of estate if a landlord assigns the lease to the assignee. An assignee and a landlord are in privity of estate if a tenant assigns the lease to the assignee. In that event, the privity of estate between the original tenant and the landlord ends. An assignee and a landlord are in privity of estate as long as the assignee stays in possession of the property. Thus, an assignee is liable for any covenants that run with the lease such as the duty to pay rent, taxes, or insurance, etc. (2) Sublease Privity of estate exists between a landlord and a tenant if the tenant subleases the premises to a sub-lessee. d) Consent

Page 26

AMERIBAR BAR REVIEW REAL PROPERTY MBE OUTLINES REAL PROPERTY 26 (1) Assignment By agreement, a tenant may need a landlord’s consent to assign a lease to an assignee. A landlord does not generally need a tenant’s consent to assign a lease to an assignee. (2) Sublease A lease may provide that a landlord’s consent is required for a tenant to sublease the premises. (3) Restraints on Alienation Disfavored The law disfavors restraints on alienation. Thus, if a lease expressly required consent for assignment of the tenant’s interest, without expressly addressing subletting, a court may conclude that subletting is permissible and vice-versa. e) Liability A party who possesses privity of contract with another may seek to enforce the contractual provisions. If no privity of contract exists, a party may seek to enforce covenants that run with the land. (1) Assignment by Tenant When a tenant assigns a lease, an assignee becomes liable to a landlord on all covenants that run with the land. The tenant continues to be liable on the lease’s other covenants. (2) Assignment by Landlord When a landlord assigns a lease, an assignee becomes liable to a tenant on all covenants that run with the land. The landlord continues to be liable on all covenants in the lease. (3) Sublease A tenant is liable for all covenants in a lease and may enforce them. A sub-lessee is not liable for the covenants in a lease and cannot enforce them. 4) TERMINATION a) Surrender Surrender constitutes a premature ending of a leasehold interest in either of two ways: • the parties mutually agree to terminate the leasehold; or • the leasehold terminates due to the operation of law. (1) Agreement to Terminate Under the Statute of Frauds, unilateral notice of surrender and a mutual surrender agreement must be in writing. Some courts interpret the Statute of Frauds as allowing for a verbal surrender if the remaining duration of a lease agreement is short enough to allow for the creation of a parol agreement. Certain statutes generally require a written surrender for most leaseholds except those of a short duration. (2) Operation of Law

Page 27

AMERIBAR BAR REVIEW REAL PROPERTY MBE OUTLINES REAL PROPERTY 27 A termination of a lease by operation of law results when two conditions are fulfilled: • a tenant engages in a non-verbal act of abandoning a premises; and • a landlord accepts the premises back for his own account. Abandonment of a premises does not, of itself, result in a surrender by operation of law. The landlord must also act. A landlord has several options in responding to a tenant’s abandonment. The landlord may enter the premises and rent them for the tenant’s account. By taking this action, the landlord may charge the tenant with the deficit between the lease’s rent and the new rent received from another party. Alternatively, the landlord may accept the abandonment as a surrender of the leased property and enter the premises for his own account. Finally, the landlord may do nothing. As a result of doing nothing, the leasehold will remain, as will the duty of the tenant to pay rent. (a) Retention of Keys A majority of courts addressing the issue have held that retention of the keys by a landlord alone does not constitute acceptance of tenant’s surrender by the landlord. b) Mitigation of Damages A landlord may incur a financial loss when a tenant abandons a leasehold without having fully paid for the remainder of its term. (1) No Duty to Mitigate under Common Law Under the common law, a lease was viewed as a conveyance instead of a contract, so the landlord had no duty to mitigate damages resulting from a tenant’s wrongful termination of a lease. Therefore, a landlord could recover the full value of rents that were due and unpaid at the time of the suit. (a) No Mitigation Duty-Still Majority The majority of jurisdictions continue to follow the common law, and do not apply the contract law doctrine of mitigation of damages when a landlord takes no action to find a new tenant after a former tenant abandons a leasehold. The Restatement (Second) of Property supports this majority rule, under which a landlord will not incur any legal detriment from doing nothing. Thus, a landlord lacks an affirmative legal duty to mitigate damages by means of seeking new tenants for abandoned premises. (i) Future Rents Under the majority rule, a landlord cannot sue a tenant for rents due in the future (the landlord is only entitled to those rents as they come due). However, responding to the fact that a tenant may disappear or be judgment-proof by the time a lease term is expired, some courts are abandoning the no-recovery-for-future-rent rule and allowing a landlord to collect damages equal to the value of rent over the entire lease term minus the property’s fair rental value when a tenant has wrongfully terminated a lease and unequivocally shown an intention not to return to the premises or pay future rent. Under this approach, a landlord receives approximately the same amount he would have received were there a duty to mitigate damages. (2) Modern Trend Imposes Duty to Mitigate The minority rule, and modern trend, applies contract law doctrine to require a landlord to mitigate damages by seeking to find a new tenant to lease a premises for the abandoning tenant’s account. The Model Residential Landlord-Tenant Code and the Uniform Residential Landlord and Tenant Act (URLTA) also require some mitigation of damages by a landlord. The rejection of the no-mitigation-of-damages rule has been based on efficiency concerns and society’s interest in assuring that resources remain in the stream of commerce rather than lying vacant. This also allows

Page 28

AMERIBAR BAR REVIEW REAL PROPERTY MBE OUTLINES REAL PROPERTY 28 landlords to sue tenants who have wrongfully terminated a lease for damages equal to the difference between the unpaid rent due under the lease and the property’s fair market rental value. (3) Impact of Attempt to Mitigate Damage (a) For Landlord’s Account If a landlord attempts to mitigate damages by re-entering the premises for his own account (i.e., so that the landlord himself can use the premises), then a tenant’s legal liability for unpaid rent ceases because operation of law ends the leasehold. (b) For Tenant’s Account If a landlord attempts to mitigate damages by re-entering and re-letting the premises “for the tenant’s account,” then a tenant’s legal liability for unpaid rent is reduced without ending the leasehold by operation of law. c) Anticipatory Breach (1) Definition Anticipatory breach occurs when either party to a lease repudiates the lease or any of its promises, terms, or covenants before their scheduled performance. Examples of covenants that are subject to repudiation include those of quiet enjoyment and habitability. (2) Remedies to Breach A promisee may consider a promisor’s repudiation as a breach and may exercise any available remedies such as an action for damages or specific performance, as well as termination of the leasehold. The promisee’s only requirement is to respond to the repudiation before the statute of limitations expires. The promisee may, however, ignore the repudiation unless doing so would increase the extent of the repudiating promisor’s damages. (3) Non-Breaching Party’s Obligation A promisee must prove that he was ready, willing, and able to perform in the absence of a promisor’s repudiation. A repudiation relieves the promisee of any obligations under the lease. To be effective, words or conduct of repudiation must be definite and unequivocal. (4) Retraction of Repudiation The promisor may retract a repudiation before the promisee: • acts in reliance on the repudiation; • communicates an affirmative acceptance of the repudiation; or • files a civil lawsuit. d) Security Deposits (1) Introduction (a) Definition

Page 29

AMERIBAR BAR REVIEW REAL PROPERTY MBE OUTLINES REAL PROPERTY 29 A security deposit is a bond or cash that is placed in another's custody to secure occupancy or possession or as a guarantee of performance. Black's Law Dictionary, 2d ed. (b) Creates Creditor-Debtor Relationship A security deposit creates a creditor-debtor relationship between the party placing it (e.g., tenant) and the party holding it (e.g., landlord). Although a tenant has no priority over the landlord's other creditors, as a general creditor, a statute may provide the tenant with priority over any creditor regarding the security deposit amount. The tenant's security deposit is subject to the landlord's potential claim, as described later, pursuant to their agreement and the law. (c) Purpose and Use Typically, a landlord requires that a tenant make a security deposit to ensure that that the tenant will keep the rental premises in good condition. Commonly, a landlord may apply the security deposit to cover the cost of the tenant's harm to the rental premises exceeding usual "wear and tear" (e.g., everyday usage). Gerald and Kathleen Hill, The People's Law Dictionary, Fine Communications. An issue may arise as to whether harm to the rental premises exceeds usual "wear and tear." (i) Distinguished from Deposit of "Last Month's Rent" A security deposit differs from a deposit of the "last month's rent" that may be used as a rent credit. Id. (2) Types of Statutory Provisions Most states' statutes limit the amount of a deposit, specify its use, and provide rules for how and when landlords must return the deposit. Some provisions of the URLTA govern security deposits. (a) Return of Security Deposit Common statutes require that a landlord return a security deposit within a specified time period after the tenant vacates, less the cost of repairing any harm or repairs to the rental premises. Hill, Law Dictionary. (b) Issue about Landlord's Use of Security Deposit An issue can arise when the landlord uses the security deposit to improve the rental premises instead of to cover harm to the premises resulting from the tenant's occupancy. Id. In that event, the tenant may seek to recover the security deposit from the landlord. (c) Separate Bank Account; Interest Payments A few states require that the security deposit be maintained in a separate bank account (e.g., in escrow). Id. Some states require that the security deposit generate interest payments. Id. (d) Amount of Security Deposit (i) May Not Exceed One Month's Rent A landlord of residential property may not receive or demand a security deposit having value greater than one month's rent. Unif. Residential Landlord and Tenant Act § 2.101(a). A tenant can demand return of this deposit. (e) Disposition of Security Deposit and Rent (i) Applying Security Deposit to Rent and Damages Upon termination of a tenancy, a landlord may apply a security deposit to payment of rent owed and damages sustained as a result of a tenant's failure to maintain a dwelling unit. A landlord must itemize the rent and the damages in a

Page 30

AMERIBAR BAR REVIEW REAL PROPERTY MBE OUTLINES REAL PROPERTY 30 written notice delivered to the tenant along with the amount owed 14 days after termination and the tenant's delivery of demand and possession. Unif. Residential Landlord and Tenant Act § 2.101(b). (ii) When Tenant May Recover Property, Money, and Damages from Landlord If a landlord keeps a security deposit and does not comply with the itemization and notice requirements, then the tenant can recover the property and money owed along with damages of at least twice the amount wrongfully withheld and reasonable attorney's fees. Unif. Residential Landlord and Tenant Act § 2.101(c). 5) HABITABILITY AND SUITABILITY a) Implied Warranties (1) Common Law The common law provides that residential leases include an implied warranty of habitability requiring that the premises be fit and suitable for basic human habitation. This warranty would be violated by, for example, a lack of working plumbing, running water, or heat during the winter. (2) Codified Provisions State statutes or regulations may provide express warranties regarding habitability, additional rights to report a violation of proper housing standards, and procedures to address such violations. These types of codified provisions may also protect a tenant who reports a landlord’s failure to comply with the legal standards for housing. (a) Retaliation Prohibited The codified provisions may preclude a landlord from responding to the reporting in any of the following ways: harassing a tenant, raising the rental cost, terminating the tenant’s lease, or engaging in a retaliatory eviction. D. Special Problems 1) RULE AGAINST PERPETUITIES a) General Considerations The Rule Against Perpetuities (the "Rule") is partially based on the concept that living people, and not deceased people, should control the disposition of property interests. The Rule is a clear expression of the common law's abhorrence of things uncertain. The Rule derives its name from the fact that it limits the duration of legal interests that suspend property rights in perpetuity. The common law prefers certainty in property rights and that property be fully owned by just one person at a time. (1) 21-Year Standard The common law will tolerate temporary contingent interests for a certain time only. Under the common law rule against perpetuities, that limitation on the duration of temporary contingent interests is no longer than 21 years after the death of a life in being at the time of the initial conveyance. If an interest in property continues to be contingent after that period, it is considered void under the Rule. (a) Applicable to Executory Interests If the Rule renders an executory interest such as a fee simple determinable with a contingent remainder void, then Question Key 4,72,76,91,96,97,103,118,123,126,127

Page 31

AMERIBAR BAR REVIEW REAL PROPERTY MBE OUTLINES REAL PROPERTY 31 courts will consider that estate a fee simple determinable with an implied possibility of reverter back to a grantor or the grantor’s successors. In other words, the prohibited executory interest will fail. b) Common Law Rule The Rule states that an interest must vest, if at all, within 21 years of the death of a life in being at the time of the interest’s creation. If there is any possibility, no matter how remote, that the interest will not vest, or will fail to vest, within the time period, then the conveyance will be void as violating the Rule Against Perpetuities. An interesting and educational way of looking at the Rule is to put it in modern terms. If you are going “to delay some gift, either by will or while you're still alive, full title has to land fair and square within 21 years after the death of a person alive at the time of the gift.’” Duhaime Law, at http://www.duhaime.org. (1) Scope of Application The Rule is applicable to contingent remainders (not vested remainders as they are already vested), executory interests, powers of appointment, and options to purchase land. The Rule does not apply to interests that the grantor retains, such as a reversion or the possibility of reverter. (a) Charitable Conveyances Exception The charity-to-charity exception to the Rule applies if a second charitable organization will receive a contingent future interest from a first charitable organization. Such a conveyance will not be subject to the Rule. c) Rule Analysis Example Let us examine how the Rule may arise in practice. Suppose a grantor conveys an estate “to a grantee for life, then to the grantee’s first child who marries and the heirs of that child.” The grantee is alive when the devise is made, but the grantee does not have any children. One should consider the following factors when analyzing this example under the Rule: (1) Is the property interest at issue subject to the Rule? In the above situation, the grantee received a life estate, any unborn children may receive a contingent remainder because no children are ascertainable, and a condition precedent must be satisfied. (2) On what date does the perpetuities period commence? The effective date of an instrument that creates an interest in property is the starting date of the perpetuities period. In the above situation, a “devise” by the grantor indicates that the instrument that created the property interest is a will. The will’s effective date is the day of the grantor’s death. (3) Must a condition precedent occur before a future interest vests? The important inquiry is if any event must occur before an interest will vest in a grantee. It does not matter if any event must occur before the grantee obtains possession. With respect to a class gift, all of the class members’ interests must be valid before that gift will take effect. Because the condition precedent requires that a child of the grantee get married, the interest may fail if the grantee does not have children or such children do not marry. (4) Determine whose lives were in being on the instrument’s effective date. In other words, which of the potential devisees were alive on that date? Such devisees, if any, were either born before

Page 32

AMERIBAR BAR REVIEW REAL PROPERTY MBE OUTLINES REAL PROPERTY 32 that date or in gestation on that date and will affect the vesting of the interest. In the above situation, only the grantor and the grantee were alive when the will became effective. Thus, either may be a life in being when applying the test. (5) Will validation of the interest timely occur? The critical question is if the interest would vest or fail with respect to the lives in being during their lifetimes plus 21 years. If the interest could, under any possible scenario, vest or fail to vest outside of this period (21 years after the death of a life in being), the interest will be invalid under the Rule. Therefore, for the interest to be valid under the Rule, there must be no possible scenario under which the contingent remainder vests, or fails to vest, more than 21 years after the death of a life in being (in this case either the grantor or grantee). In order to reach the appropriate conclusion, one must experiment with the lives in being and determine whether there is a chance that the interest could vest, or fail to, more than 21 years after the death of a life in being. Let us return to our example. Remember, the grantor conveys the estate “to a grantee for life, then to the grantee’s first child who marries and the heirs of that child.” In the example, the grantee is alive with no children at the time of the conveyance. Of course, the grantee can have a child (or children) after the conveyance. Is it possible for the grantee to have a child marry more than 21 years after the grantee’s death? Of course it is. For example, the grantee can have a child two years after the conveyance. The grantee can then die. Thirty years later (more than 21 years after his death), that child may marry. Thus, the interest may vest more than 21 years after the death of a life in being (the grantee). Therefore, the interest violates the Rule Against Perpetuities. Remember, the rule does not provide that an interest is valid if it can somehow vest within 21 years of the death of a life in being. On the contrary, the interest will fail if the interest can somehow vest (or fail to) more than 21 years from the death of a life in being. d) Wait and See Rule Many states have reformed the harsh common law Rule and have taken a “wait and see” approach. Under those state statutes, a court will wait out the perpetuities period to determine if an interest at issue actually vests or fails within the period. In contrast, the harsh common law Rule would invalidate an interest if it is merely possible that the interest may vest or fail after the period. e) Cy Pres Doctrine Some jurisdictions have statutes adopting the Cy Pres doctrine, under which a court may reform a non-vested interest to assure that it will vest within the permissible period. In exercising this reformation power, the statutes direct the courts to reform the non-vested interest to most closely approximate the intention of the grantor of an interest in order that the non-vested interest will vest during the perpetuities period. f) Other Reforms A modern trend exists to reform the rigid common law Rule Against Perpetuities. Some states have implemented a statute that repeals the Rule. These statutes may be based on the Uniform Statutory Rule Against Perpetuities (USRAP). The USRAP will validate interests that: 1) would satisfy the common law rule, or 2) would vest or fail within 90 years after the creation of an interest. g) Shelley’s Case Most jurisdictions have abolished the Rule in Shelley’s Case. It concerned conveyances which provided: “to Gertrude for life, then to Gertrude’s heirs.” The rule operated to merge the life estate in a person and a remainder interest in

Page 33

AMERIBAR BAR REVIEW REAL PROPERTY MBE OUTLINES REAL PROPERTY 33 the person’s heirs. As a result of the merger, for example, Gertrude would obtain both the life estate and the remainder. By operation of law, the Rule in Shelley’s Case resulted in Gertrude possessing a fee simple absolute. 2) ALIENABILITY, DESCENDABILITY, AND DEVISABILITY Both absolute estates, such as fee simple estates, and qualified estates, such as future interests, are generally transferable (i.e., alienable, descendible, and devisable). a) Restraints on Alienation The law disfavors restraints on alienation. A restraint on alienation is a restriction on a grantee's right to sell or transfer real property. Courts will only permit a restraint on alienation if it is reasonable and for a legitimate purpose and will strike down any unreasonable or undue restraint. Courts have considered keeping land ownership within a family to be a reasonable purpose. (1) Invalid Restraints on Alienation (a) Indefinite Restraints The common law generally prohibits restraints on alienation of fee simple estates unless reasonable and for a legitimate purpose. For example, a restraint that violates public policy will not be enforceable. Additionally, the common law usually does not countenance a forfeiture restraint that lacks any limitations upon its scope and duration. For example, suppose O conveys Blackacre “to Gary, provided that Blackacre is never sold to a person who owns a red car.” The restriction, which lacks any limitation upon its scope and duration, is invalid. As a consequence, Gary possesses a fee simple absolute. (b) Unconstitutional Restraints on Alienability Even if a restraint on alienability is only applicable for a reasonable duration, courts will generally not enforce a restraint that discriminates against an individual or entity based on a constitutionally protected classification, such as race or religion. 3) FAIR HOUSING/DISCRIMINATION a) Fair Housing The Fair Housing Act of 1968 (FHA) was passed as part of the Civil Rights Act of 1968. The FHA protects people from discrimination when renting, buying, or securing financing for housing. (1) Unlawful Conduct (a) Protected Characteristics The FHA prohibits discrimination in housing based on: • race; • color; • national origin; • religion; • sex; • disability; and • the presence of children. Question Key 72

Page 34

AMERIBAR BAR REVIEW REAL PROPERTY MBE OUTLINES REAL PROPERTY 34 (b) Prohibited Actions Specifically, it is unlawful for a landlord or owner of a dwelling to engage in the following discriminatory actions against people based on the foregoing characteristics (the "Protected Characteristics"): • refuse to rent or sell to a person; • discriminate in provided terms; • publish anything indicating discrimination or preference in the rental or sale; • represent to any person that an available dwelling is unavailable; • for profit, to attempt to induce a person to rent or sell a dwelling by representations about the potential entry or entry into the neighborhood of anyone having the Protected Characteristics 42 U.S.C. § 3604(a)-(e). For example, the FHA would prohibit a residential landlord from charging male tenants more rent than female tenants. (c) Related Transactions It is also unlawful to discriminate against another person based on the Protected Characteristics in a residential real estate-related transaction. 42 U.S.C. § 3605(a). Related transactions include brokering, selling, appraising, and financing residential real property (purchase, renovation, and construction). 42 U.S.C. § 3605(b). (d) Membership in Listing Service or Brokers Organization It is unlawful to deny any person access to or participation or membership in any multiple-listing service, real estate brokers’ organization, or other organization, service, or facility relating to the business of renting or selling dwellings, or to discriminate against the person in the conditions or terms of such access, participation, or membership, based on the Protected Characteristics. 42 U.S.C. § 3606. (2) Exemptions The FHA exempts certain landlords of smaller properties to protect their close personal relationships from invasion of privacy. Landlords meeting the criteria for exemption would not be subject to the FHA. (a) Single-Family Dwelling An owner leasing or selling his own single-family dwelling is exempt if: • the owner does not simultaneously own more than three dwellings; and • the owner does not reside in a dwelling when it is sold or was not its most recent resident before such sale, for 1 such sale within 24 months; and • the owner has no interest in, title to, or right to any of the proceeds from the simultaneous rental or sale of more than one dwelling; and • the dwelling is sold or rented without: 1) the use of any sales or rental facilities or services of any broker, salesman, or agent, or such person's employee; and 2) use of any advertisement or written notice indicating any limitation, preference, or discrimination or any intention to make any such limitation, preference, or discrimination on account of the Protected Characteristics. 42 U.S.C. § 3603(b)(1). (i) Civil Rights Act of 1866 Lacks Single-Family Dwelling Exemption

Page 35

AMERIBAR BAR REVIEW REAL PROPERTY MBE OUTLINES REAL PROPERTY 35 A related law, the Civil Rights Act of 1866 (the "Act"), has no exemption for a single-family dwelling. Therefore, a plaintiff could bring an action for conduct that violates that Act but is exempt under the FHA. Bush v. Kaim, 297 F. Supp. 151 (N.D. Ohio 1969). The Act will be addressed in another portion of the outline. (b) Multiple-Family Dwelling, If Owner Occupies One The FHA does not apply to an owner leasing units (e.g., apartments) or rooms in dwellings (e.g., buildings) containing living quarters for occupancy by up to four families living independently of each other, if the owner actually occupies one of the living quarters. 42 U.S.C. § 3603(b)(2). (i) Act Lacks Multiple-Family Dwelling Exemption The Civil Rights Act of 1866 has no exemption for a multiple-family dwelling where its owner actually occupies one of the living quarters. Therefore, a plaintiff could bring an action for conduct that violates that Act but is exempt under the FHA. Bush v. Kaim. (c) Religious Entities Preference to Members Religious entities may limit the rental, sale, or occupancy of dwellings that it operates or owns for noncommercial purposes to persons of the same religion, or give preference to such persons, unless membership in that religion is limited by color, race, or national origin. 42 U.S.C. § 3607(a). Religious entity includes a religious association, organization, or society, or any nonprofit organization or institution operated, controlled, supervised by or in conjunction with such religious entities. Id. (d) Private Clubs Preference to Members If a private club is not open to the public, and as an incident to its primary purpose(s) provides lodgings that it operates or owns for noncommercial purpose(s), then the private club may give preference to its members or limit the occupancy or rental of such lodgings to its members. Id. (3) Enforcement (a) Enforcement by the United States The Secretary of the United States Department of Housing and Urban Development, on behalf of an aggrieved person, may enforce the FHA against a lessor or seller of property engaging in unlawful conduct. (i) Example of Aggrieved Person An aggrieved person could include, for example, nonwhite and white tenants who allege that a landlord racially discriminates against nonwhites in the rental of apartments. Trafficante v. Metro. Life Ins., 409 U.S. 205 (1972), citing 42 U.S.C. § 3610(a). Another example of an aggrieved person could include a homeowner in a village who claims to be harmed by a real estate brokers' discriminatory conduct of racial "steering" practices. Gladstone Realtors v. Village of Bellwood, 441 U.S. 91 (1979), citing 42 U.S.C. § 3612. "Steering" means "directing prospective home buyers interested in equivalent properties to different areas according to their race." Id. (A) One Year Limitation for Complaint An aggrieved person may file a written complaint with the Secretary within one year after an alleged discriminatory housing practice. 42 U.S.C. § 3610(a)(1)(A)(i). (ii) Secretary May Investigate and File Complaint The Secretary may investigate the alleged housing practice to determine whether a complaint should be brought and file a complaint. 42 U.S.C. § 3610(a)(1)(A)(i)-(ii).

Page 36

AMERIBAR BAR REVIEW REAL PROPERTY MBE OUTLINES REAL PROPERTY 36 (A) Administrative Hearing Absent an election for a civil action, the Secretary will provide an administrative hearing before an administrative law judge who may order damages, equitable relief, and/or assess a civil penalty against a party to prevent or remedy a discriminatory housing practice. 42 U.S.C. § 3612(b), (g). (B) Election to Pursue Civil Action Once a complaint is filed, any party may elect, with 20 days written notice to all parties, to have the claims asserted in that charge decided in a civil action in lieu of an administrative hearing. 42 U.S.C. § 3612(e)(1). Within 30 days of an election for a civil action, the Secretary will pursue a civil action in a federal district court. 42 U.S.C. § 3612(o)(1). If the court finds a discriminatory housing practice, then the court may grant any relief available in a civil action under the following provisions. 42 U.S.C. § 3612(o)(3). (b) Enforcement by Private Person As an aggrieved person, a private person may enforce the FHA against a lessor or seller of property engaging in unlawful conduct. (i) Two Year Limitation for Civil Action Within two years after such alleged discriminatory housing practice, an aggrieved person may bring a civil action in a state or federal court seeking relief from such practice. 42 U.S.C. § 3613(a)(1)(A). The person may do so regardless of any complaint filed by the Secretary, unless: 1) the Secretary or a local or state agency has obtained a conciliation agreement with the aggrieved person's consent; or 2) an administrative hearing is occurring on the same charge. 42 U.S.C. § 3613(a)(2)-(3). (ii) Relief that May Be Granted If the court finds a discriminatory housing practice, then the court may • award actual and punitive damages, • grant any injunctive or other appropriate order, • allow the prevailing party, other than the United States, a reasonable attorney’s fee and costs 42 U.S.C. § 3613(c)(1)-(2). (iii) Relief May Not Affect Rentals, Contracts, Sales, and Encumbrances Consummated Prior to Relief No relief granted may affect any lease, contract, sale, or encumbrance consummated prior to granting such relief and involving a good faith tenant, purchaser, or encumbrancer who lacks actual notice of the filing of a complaint with the Secretary or civil action under the FHA. 42 U.S.C. § 3613(d). b) Discrimination Several federal statutes prohibit discrimination in the rental and sale of real property on specified grounds such as color, race, national origin, or religion. (1) Federal Legislation per Congressional Authority over Private Action

Page 37

AMERIBAR BAR REVIEW REAL PROPERTY MBE OUTLINES REAL PROPERTY 37 (a) Civil Rights Act of 1866 Congress enacted the Civil Rights Act of 1866 to enforce the Thirteenth Amendment. U.S. Const. Amend. XIII, § 2. (i) Statement of Equal Rights Everyone within the United States has the same right therein "to make and enforce contracts, to sue, be parties, give evidence, and to the full and equal benefit of all laws and proceedings for the security of persons and property as is enjoyed by white citizens." 42 U.S.C. § 1981(a). (ii) Protection against Private and Public Impairment These rights are protected against impairment by private discrimination and under color of State law. 42 U.S.C. § 1981(c). (iii) Discrimination Must Be Purposeful to Be Actionable Discrimination in violation of this federal statute must be purposeful (i.e., intentional) to be actionable under this statute. Gen. Bldg. Contractors Ass'n, Inc. v. Pennsylvania, 458 U.S. 375 (1982), interpreting 42 U.S.C. § 1981. (iv) Citizens' Property Rights per Act's Section 1982 "All citizens of the United States shall have the same right, in every State and Territory, as is enjoyed by white citizens thereof to inherit, purchase, lease, sell, hold, and convey real and personal property." 42 U.S.C. § 1982. This Section 1982 of the Act bars all racial discrimination, public and private, in the rental and sale of real property. Jones v. Mayer, 392 U.S. 409 (1968) applying 42 U.S.C. § 1982 (Thirteenth Amendment authorizes Congress to define "badges and incidents of slavery" as including private discrimination in real estate transactions). (A) Act Only Bans Racial Discrimination Only racial discrimination is banned by the Act, which does not ban discrimination based on national origin or religion. The FHA applies to discrimination based on national origin or religion. (B) The Act Applies to Property Transfers The Act applies to rental or sales of real property. Therefore, where a commercial building owner decides to not sell his building to a potential buyer because the buyer is an African-American, the potential buyer can sue under the Act. (C) Remedies Available per the Act A person subject to discrimination in violation of federal law may recover damages from, or obtain an injunction against, a property seller or landlord. Id.; Sullivan v. Little Hunting Park, 396 U.S. 229 (1969). (b) Conspiracy to Interfere with Civil Rights (i) Depriving Persons of Equal Protection A plaintiff may bring a private cause of action against a private conspiracy seeking to deprive him of equal protection of the laws' privileges or rights. The cause of action is constitutional under the Thirteenth Amendment where the plaintiff proves intentional racial discrimination. Griffin v. Breckenridge, 403 U.S. 88 (1977), interpreting 42 U.S.C. § 1985(3). The plaintiff may seek damages for harm resulting from such conspiracy. c) Proving Violation of Federal Statutes (1) Proof of Discriminatory Effect, or Impact, Makes Prima Facie Case

Page 38

AMERIBAR BAR REVIEW REAL PROPERTY MBE OUTLINES REAL PROPERTY 38 Suppose, for example, that a lessor of real estate has a method of screening tenants that filters out more green people than blue people. In order to prove a prima facie case of discrimination under either Section 1982 or the FHA, a green person must show that the lessor's method has a discriminatory impact. The green person need not show that the lessor acted with discriminatory intent in order to prove a prima facie case of discrimination. (a) Elements of Prima Facie Case A plaintiff alleging a lessor's or seller's discriminatory denial of housing would prove a prima facie case by showing: • the plaintiff is within a class protected against discrimination (e.g., color, race, national origin); • the plaintiff is qualified to, and applied to, rent or purchase such housing; • the lessor or seller rejected the plaintiff's application for housing; and • the housing option stayed available to people not within the protected class. Robinson v. 12 Lofts Realty, Inc., 610 F.2d 1032 (2d Cir. 1979). (2) Burden of Proof Shifts After the plaintiff makes a prima facie case of discrimination, the defendant will have the burden to present evidence to establish that the defendant's conduct was not motivated by improper considerations such as the plaintiff being in a protected class such as race. (3) Rebutting Plaintiff's Case To rebut the plaintiff's prima facie case, the defendant must persuade the trier of fact that he has sincere reasons for his conduct and negate any discrimination that could be inferred from the events. If the plaintiff's race is any of the factors motivating the defendant's conduct, then the defendant violates the applicable statute. For example, if the defendant establishes that he rejected the plaintiff because the plaintiff was an agitator and advocate, then the rejection would be lawful. However, if the plaintiff establishes that the defendant also rejected him based on his race, then the rejection would be unlawful. d) Liability for Exploitation in Housing Prices Section 1982 applies to exploitation of blacks in housing prices and is intended "to assure that a dollar in the hands" of a black person can buy the "same thing as a dollar in the hands of a white" person. Jones v. Mayer. Thus, a violation of Section 1982 occurs when, on account of residential racial segregation, one housing market exists for black purchasers and another housing market exists for white purchasers, and sellers take advantage of this situation by exacting prices from blacks that exceed those paid by whites for comparable housing. Clark v. Univ. Builders, Inc., 501 F.2d 324 (7th Cir. 1974), cert. denied, 419 U.S. 1070 (1974). "[T]he benchmark for guiding" the sellers' conduct is reasonableness with respect to housing transaction prices and terms. Id. 4) CONFLICT OF LAWS RELATED TO DISPUTES INVOLVING REAL PROPERTY a) Traditional Approach – Law of Situs of Land The traditional choice of law rule is that all issues affecting title to land are determined by the law of the situs of the land. Occasionally, a court will characterize a case involving property as a contract case and apply the law of the place where the contract was made. Generally, situs law governs the validity of contracts to convey land. Restatement (Second) Conflict of Laws § 189. b) Modern Approach – More Significant Relationship

Page 39

AMERIBAR BAR REVIEW REAL PROPERTY MBE OUTLINES REAL PROPERTY 39 Modern approaches to choice of law issues would permit departure from the traditional rule if a state other than the situs state had a "more significant relationship" to the facts of the case or a greater interest in the outcome. Courts may examine factors such as: • where the contract at issue was negotiated; • where the parties entered the contract; and • where the parties to the contract are domiciled. Courts will also examine any state policy of protecting the expectations of land purchasers or a general interest in securing the integrity of transactions that occur within a state’s borders. However, the validity of contracts to convey land would most likely be determined by the situs state’s law. After all, the situs state has a strong interest in any action that will affect title to its land. In the end, it is only the situs state that can award title to anyone. II. RIGHTS IN REAL PROPERTY A. Restrictive Covenants 1) NATURE AND TYPE A restrictive covenant is a legal obligation, usually imposed in a deed, to do or not to do something with regard to real property. Examples of restrictive covenants include a duty to maintain a property in a reasonable state of repair, to preserve a sight-line for a neighboring property, not to run a business from a residence, or not to build on certain parts of the property. a) Covenants at Law (1) Affirmative Covenant Distinguished from Negative An affirmative covenant involves the promise to do some act. A negative covenant is the promise to refrain from doing some act. For example, a covenant may prohibit the use of a parcel for a specific purpose, such as for a brothel. (2) Compared to Easements Covenants are sometimes referred to as “negative easements” because they are comparable to easements. For example, a beneficiary of a covenant may not enter upon the burdened property but may require the servient property’s owner to do or not do something on that property. (3) Applicability of Zoning Ordinances Zoning ordinances do not necessarily override a private restrictive covenant. The stricter of either the zoning ordinance or the covenant will prevail. (4) Covenant Provides Benefit and Imposes Burden A covenant often both provides a benefit to a promisee and imposes a burden on a promisor. The benefit is the promisee’s receipt of a promised performance. The burden is the promisor’s obligation to fulfill the promise. b) Covenant in Equity (Equitable Servitude) Question Key 13,21,25,47,65,66,81,89,90, 100,101,109,110,128,150,166

Page 40

AMERIBAR BAR REVIEW REAL PROPERTY MBE OUTLINES REAL PROPERTY 40 Covenants that run in equity are also known as equitable servitudes. They are promises regarding the use of real property that bind an initial promisor and promisee and their successors. Whether these successors may enforce the servitude depends on whether the covenant(s) run with the land that the initial promisee and promisor conveyed to the successors. If the covenant runs with the land, courts may consider it a cloud or “burden” on the title to the land. In that event, the covenant is not personally limited in its application to the initial promisor and promisee. When an equitable restriction runs with the burdened property, one can describe it as being rooted in that land. (1) Types of Equitable Servitudes (a) Affirmative A covenant to perform an affirmative act is a type of equitable servitude that may not necessarily touch and concern the land. Whether such a covenant touches and concerns the land may depend on if, or to what extent, the act affects the land. (b) Negative Negative equitable servitudes impose prohibitions that will touch and concern the land. (c) In Gross If an equitable servitude involves a benefit in gross to an estate, the benefit cannot run with the land. The term “in gross” means that a party that does not own adjacent land and may not even own any land receives the benefit of the servitude. (d) Implied Reciprocal Servitudes An implied reciprocal servitude exists under the general or common scheme doctrine. It exists when a builder of property or a real estate developer subdivides a large parcel of land into equally-sized lots. Grantors initially sell and convey those lots using a deed containing a restrictive covenant limiting use of the lots, generally for single-family dwellings. If a subsequent buyer receives such a deed for such a lot that lacks the restrictive covenant language, then the other owners may seek and obtain injunctive relief against the building of anything other than a single-family dwelling on the lot. The parties for whom the benefit of the restrictive covenant are intended (typically other lot owners) may enforce the restrictive covenants on others similarly bound. Enforcement of such covenants, however, rests upon the existence of a common plan. (i) Analysis of Enforceability The test to determine if an implied reciprocal servitude is generally enforceable is if: • A lot was originally part of a parcel held in common ownership; • The lot was subject to a common building plan or scheme involving a use restriction; and • A buyer of a deed to the lot possessed inquiry notice of the use restriction. (A) Bases of Inquiry Notice The following factors may provide inquiry notice even if a developer failed to record a map, plan, or plat of the subdivision: • physical evidence that a subdivision only includes single-family homes, and

Page 41

AMERIBAR BAR REVIEW REAL PROPERTY MBE OUTLINES REAL PROPERTY 41 • the restriction’s existence in recorded deeds. 2) CREATION To be binding, the grantor must place a restrictive covenant on property at the time it is conveyed. Someone who has no interest in the property cannot attach the burden to the property after the conveyance. a) Covenant at Law (1) Burdens Running with Land The following elements are necessary for a burden to run with the land: • A covenant is in writing; • The intent of the initial promisee and promisor is for the covenant to run with the estates in land; • The covenant relates to, or touches and concerns, the estates in land; • The initial promisee and promisor are in horizontal privity; and • The successors of the promisee and the promisor are in vertical privity. The first two elements are straightforward and thus require no elaboration. (a) Touch and Concern The “touch and concern” element is satisfied if: • the burden diminishes the use and enjoyment of the servient estate; or • the burden increases the use and enjoyment of the benefited estate. C. Clark, Real Covenants and Other Interests Which “Run With Land” 97 (2d ed. 1947), states that: “If the promisor’s legal relations in respect to the land in question are lessened—his legal interest as owner rendered less valuable by the promise—the burden of the covenant touches or concerns that land; if the promisee’s legal relations in respect to that land are increased—his legal interest as owner rendered more valuable by the promise—the benefit of the covenant touches or concerns that land.” (b) Horizontal Privity The element of horizontal privity means that the initial promisor and promisee were connected in terms of their relationship, such as mortgagor and mortgagee, grantor and grantee, or landlord and tenant. (c) Vertical Privity The element of vertical privity means that a connection exists between the initial promisor and promisee and their successors. For vertical privity to exist, an original promisor and each subsequent grantor must have conveyed an entire estate to each successor. (2) Benefits Running with Land The following elements are necessary for a benefit to run with an estate in land (“estate”) and benefit the successors: • A covenant is in writing; • The intent of the initial promisee and promisor is for the covenant to run with the estate;

Page 42

AMERIBAR BAR REVIEW REAL PROPERTY MBE OUTLINES REAL PROPERTY 42 • The covenant relates to, or touches and concerns, the estate; and • The successors of the promisee and the promisor are in vertical privity. The required elements for a benefit to run with the land in a covenant at law do not include the element of horizontal privity, which is required for a burden to run with the land in a covenant at law. b) Covenant in Equity The Statute of Frauds applies to equitable servitudes because they affect interests in land and consequently must be in writing to be enforceable. (1) Burden Running with Land The following elements are necessary for a burden to run with the land: • A promise that is in writing; • The initial promisee and promisor intended for the promise to be enforceable; • The promise relates to, or touches and concerns, the land; and • The successors of the promisee and the promisor have notice of the promise. Unlike restrictive covenants, equitable servitudes that burden the land do not require the element of horizontal privity. (2) Benefit Running with Land The following elements are necessary for a benefit to run with the land and bind the successors: • A covenant that is in writing; • The initial promisee and promisor intended that the promise be enforceable; and • The promise relates to, or touches and concerns, the land. 3) SCOPE Covenants at law and covenants in equity differ in their scope of remedies available. a) Covenant at Law (1) Remedies at Law A covenant “at law” is a covenant that can only be enforced for damages, not in equity. (a) Covenant Enforcement The only means of enforcing rights under a covenant at law is to file a lawsuit seeking damages. No equitable remedies are available. (b) Deed as Basis of Enforcement A deed may provide an owner of a parcel in a subdivision an independent right to enforce the restrictive covenants with respect to all other deeds in the subdivision. (c) Third Party Enforcement

Page 43

AMERIBAR BAR REVIEW REAL PROPERTY MBE OUTLINES REAL PROPERTY 43 An unintended and incidental third-party beneficiary lacks any basis to enforce a covenant at law. Similarly, a covenant at law is not enforceable against remote grantees. The parties must be in privity to enforce a covenant at law. b) Covenant in Equity (1) Equitable Remedies Most modern litigation involving covenants running with the land is based upon equitable servitudes and seeks an equitable remedy, rather than seeking to obtain enforcement of restrictive covenants to recover a legal (monetary) remedy. (a) Covenant in Equity Is Only Enforceable at Equity A party may only enforce the provisions of a covenant in equity (i.e., equitable servitude), “at equity.” The remedy for a breach that a dominant estate owner possesses distinguishes an equitable servitude from a covenant at law. If the owner seeks damages, the owner must demonstrate the requirements for a covenant at law. If the owner requests equitable relief such as an injunction, the owner must establish the elements of an equitable servitude. (b) Equitable Servitude Enforcement The means of enforcing an equitable servitude is to file an action in equity seeking injunctive relief for the following reasons: • legal remedies are inadequate; • to halt a current breach of an equitable servitude; and • to prevent any future breach of the equitable servitude. (c) Implied Reciprocal Servitudes Exception The courts may grant injunctive relief on the basis that an implied reciprocal servitude exists. This exception is to the general rule that equitable servitudes, like restrictive covenants, must satisfy the Statute of Frauds to be enforceable. Otherwise, usually if no restriction on land use is written in a recorded deed, the courts cannot enforce that restriction at law or in equity. (d) Third Party Enforcement An unintended and incidental third-party beneficiary lacks any basis to enforce an equitable servitude. The parties need not have privity to enforce an equitable servitude. 4) TRANSFER Transfer of real property subject to a restrictive covenant could affect enforcement of the restrictive covenant. a) Enforcement of Restrictive Covenants An initial promisee and promisor may enforce a restrictive covenant. The issue of whether the successors in interest (“successors”) may enforce the covenant depends on whether the covenant runs with any estates in land that the initial promisee and promisor convey to their successors. A conveyance from an initial promisee and promisor to their successors will be effective and provide for an enforceable covenant when the covenant runs with an estate. The lands at issue are either burdened or benefited, and they may either exist within the same or different estates.

Page 44

AMERIBAR BAR REVIEW REAL PROPERTY MBE OUTLINES REAL PROPERTY 44 (1) Duration A covenant may specify when it will end or provide a method of determining when it will end. (2) Rights of Successor in Interest A successor in interest to a parcel of land may invoke rights under a covenant if: • the successor of the burdened land received constructive, inquiry, or actual notice of the covenant; • the covenant touches and concerns the land at issue; and • sufficient evidence indicates that the necessary intent existed to bind successors and assigns. (a) Sufficient Evidence Requirement If a covenant expressly states that the parties to it intended the covenant to run to successors and assigns, it satisfies the element of sufficient evidence. For example, a covenant might provide that every buyer of a lot in a subdivision must pay an annual fee to maintain its recreation area. Such a provision is a way to minimize potential litigation. 5) TERMINATION a) Termination by Merger If one party obtains title to both the land burdened, and the land benefitted, by a restrictive covenant, then the covenants merge and no longer exist. b) Unconstitutional Restrictions A court may terminate a restrictive covenant if it is unconstitutional. For example, a court may refuse to enforce a restrictive covenant that prohibits the sale of property by race. c) Equitable Defenses When a party claims an equitable servitude, a party-opponent can assert the following defenses to enforcement of the equitable servitude imposing restrictions. (1) Acquiescence If a plaintiff benefitted by restrictions has not brought an action against anyone else who has violated them, then a court may find that the plaintiff acquiesced in the defendant's violation of them. (2) Estoppel A plaintiff benefitted by restrictions may be estopped to enforce them because the plaintiff's conduct would lead a reasonable person to believe that the restrictions relied upon by the plaintiff would not be enforced. (3) Abandonment As with an easement, an equitable servitude may be extinguished by abandonment in circumstances such as: 1) by common consent the landowners in a subdivision have violated restrictions such that the restrictions' purpose can no longer be fulfilled; or 2) when a subdivision developer sells some parcels having restrictions before selling the other parcels without restrictions, a court may find that the developer abandoned the restrictions such that they may not be enforced against the parcels sold earlier.

Page 45

AMERIBAR BAR REVIEW REAL PROPERTY MBE OUTLINES REAL PROPERTY 45 (4) Relative Hardship A court may decline to grant an injunction having little benefit and imposing great hardship, and vice versa. (5) Change of Neighborhood Doctrine The change of neighborhood doctrine provides for judicial relief from a covenant if the neighborhood has sufficiently changed so that a court should either not enforce the covenant or consider it terminated. A test that courts use to apply the doctrine provides that: 1. a change in a neighborhood has rendered: ● a covenant outmoded, ● its usefulness lost, and ● its benefits substantially lost; and 2. judicial enforcement of the covenant would be inequitable. Some courts hold that changes in zoning and decreased value in benefited land are only evidence that indicates a change of neighborhood but do not serve to conclusively establish that the change occurred. 6) COMMON INTEREST OWNERSHIP COMMUNITIES AND ASSOCIATIONS Private property developments can be referred to commonly by various terms including, for example: proprietary communities, common interest developments, common interest communities, residential community associations, homeowners associations, or property owners associations ("Association(s)"). Lee Anne Fennell, Contracting Communities, Univ. of Illinois L. Rev., Vol. 2004, Issue 4, p. 830, n. 4. a) Property Owners Associations (1) Association May Add Restrictive Covenant and What that Controls A real property developer, property owner, or Association may put a restrictive covenant in place: 1) that controls land use -- the character, type, and location of structures to be built on the land; and 2) that may involve payment of dues or fees to the Association. Judon Fambrough and Cindy Dickson, Living with Deed Restrictions, Tierra Grande, 1983 (rev. September 2013), p. 16. The Association could use dues or fees paid by property owners to pay for its administrative and/or property maintenance costs, for example. (a) Examples of Restrictive Covenant Provisions Restrictive covenants can provide, for example, that: • no fence may be constructed on the property other than of stone and not over 5 feet high; • no structure may be built within 15 feet of the property line; or • no tennis court may be built within 20 feet of the frontage street. Restrictive covenants could even place limits on exterior paint colors, lawn ornaments, pets, and motorcycles, in relation to the subject property. Fennell, Contracting Communities, p. 830. (2) Association May Enforce Restrictive Covenant Commonly, an Association bands property owners (e.g., homeowners) together in a planned community (e.g., residential housing subdivision) and may enforce restrictive covenants controlling the affected properties (i.e., subdivision restrictions).

Page 46

AMERIBAR BAR REVIEW REAL PROPERTY MBE OUTLINES REAL PROPERTY 46 (3) Membership Is Mandatory when Restrictive Covenant Is Recorded A property owner's membership in an Association may be "voluntary" or "mandatory." Usually, membership is mandatory if the restrictive covenant is recorded in a property's chain of title. (4) Common Ownership that Includes Mandatory Association Traditionally, these most common types of common ownership include creation of a mandatory Association that maintains and manages common amenities (e.g., community center): • “planned unit developments” (e.g., residential housing subdivisions), • condominiums, and • real estate cooperatives, which exceed this outline's scope. Unif. Common Interest Ownership Act, Prefatory Note, (1982, 1994). (a) Association May Manage Common Interest Community An Association may manage a common interest community (the "CIC"). Unif. Common Interest Ownership Act §§ 1-103(3), 3-101 to 3-319 (1982, 1994). b) Common Interest Ownership Communities (1) CIC Includes Individually Owned Units and Condominiums Essentially, CICs are housing developments comprised of individually owned units (e.g., single-family homes, townhouses) or condominiums that share ownership of common areas (e.g., parking, landscaping, swimming pools). Unif. Common Interest Ownership Act § 1-103(7)-(8) (1982, 1994). (2) Uniform Common Interest Ownership Act (UCIOA) (a) UCIOA Governs Various Aspects of CIC The UCIOA governs creation, management, and termination of a CIC. Unif. Common Interest Ownership Act §§ 2-101, 3-101 to 3-319, 2-118 (1982, 1994). The CIC may be a residential planned community, a condominium, or a real estate cooperative. Unif. Common Interest Ownership Act §§ 1-103(7)-(8), (10), 1-207 (1982, 1994). (i) Creation by Recording Declaration as Deed A CIC may be created by recording a declaration executed in the same way as a deed. Unif. Common Interest Ownership Act § 2-101(a) (1982, 1994). (ii) Declaration of Covenants, Conditions, Restrictions Commonly, a subdivision planned unit development has a "declaration of covenants, conditions, and restrictions" (the "CCR"). Unif. Common Interest Ownership Act § 1-103(13), cmts. (1982, 1994). Reciprocal, community-wide, CCR can control each unit owner’s use of his own property, the maintenance and use of common property and services, and other aspects of community governance and life. Fennell, Contracting Communities, p. 830. (b) UCIOA Requires Disclosures to CIC Property Purchaser The UCIOA requires disclosure of key facts regarding CIC property at its sale to a purchaser, such as: • warranties of sale; Low Priority

Page 47

AMERIBAR BAR REVIEW REAL PROPERTY MBE OUTLINES REAL PROPERTY 47 • a purchaser's rescission rights in a buy and sell contract, • resale disclosure for any sale after the developer sells the property initially, including a copy of: the declaration, the Association's rules, regulations, or bylaws; and a certificate containing statements of all required disclosures; and • deposits made in escrow to secure a buy and sell contract. Unif. Common Interest Ownership Act §§ 2-105, 4-109 to 4-110, 4-113 to 4-116 (1982, 1994). (i) Disclosure Can Provide Notice of CCR As a practical matter, one effect of such disclosures should be to provide notice to the purchaser of any CCR (e.g., restrictive covenant) to which such CIC property would be subjected upon its sale. The CIC property purchaser would assume the place of its seller with respect to the CCR. (c) Transfer of Interest in Unit of CIC (i) Unit Is Part of CIC for Occupancy or Ownership Under the UCIOA, a "unit" is a physical part of the CIC designated for separate occupancy or ownership. Unif. Common Interest Ownership Act § 1-103(31) (1982, 1994). For example, a unit could be a condominium in a CIC or a house in a subdivision CIC. (ii) Owner May Dispose of Interest by Transfer A unit owner may "dispose" of an interest in a unit by a voluntary transfer to a purchaser, other than a security interest. Unif. Common Interest Ownership Act § 1-103(15) (1982, 1994). Such a transfer may include more than conveyances and would include contracts of sale. Id. (1994). (iii) Purchaser May Acquire Interest via Transfer A "purchaser" may acquire certain types of interest in a unit by means of a voluntary transfer. Unif. Common Interest Ownership Act § 1-103(25) (1982, 1994). (iv) Conveyance Transfers All Warranties of Quality Conveyance of a unit transfers to the purchaser all (i.e., express and implied) warranties of quality made by prior declarants in the declaration. Unif. Common Interest Ownership Act §§ 4-113(c), 4-114(f) (1982, 1994). (A) Definition of Declarant A "declarant" is any person(s) who: • applies for registration of a CIC; or • within a common promotional plan, offers to dispose of their interest in a unit; or • has any special declarant right(s), which exceed this outline's scope. Unif. Common Interest Ownership Act § 3-103(12) (1982, 1994) (B) Means of Transfer of Declarant's Rights Transfer of an ordinary declarant's rights may occur:

Page 48

AMERIBAR BAR REVIEW REAL PROPERTY MBE OUTLINES REAL PROPERTY 48 • as a transfer to another person; or • as an arm's-length transaction, or • by foreclosure or a deed in lieu of foreclosure. Unif. Common Interest Ownership Act §§ 4-113(c), 4-114(f) (1982, 1994), cmt. 7. B. Easements, Profits, and Licenses 1) GENERAL The law originally referred to easements and profits as “incorporeal hereditaments,” which meant a “non-possessory” and “inheritable” estate. Easements, profits, and licenses are considered non-physical interests in land. a) Reduced Right of Possession Easements, profits, and licenses are considered servitudes on the possessory interests in land. Thus, an owner of land that is subject to a servitude relinquishes the right of exclusive possession in favor of the holder of a servitude. Easements, profits, and licenses exist separately from, and are created in addition to, the “natural rights” that arise from land ownership. 2) EASEMENTS a) Nature and Characteristics An easement is a limited right to access the real property of another. (1) Limited Right An easement ordinarily exists when an owner of property grants to a non-owner of property (an easement holder) a restricted right to use or access that property. Examples of easements include driveways, walkways, and utility routes. Certain criteria, such as duration or particular purpose, may restrict the scope of use or access of an easement. (a) Duration An easement may exist perpetually, for a specified duration, or for as long as it is required to continue based on the nature of the easement. (2) Types of Estates The property that the easement benefits is the dominant estate. The property that is burdened by an easement is the servient estate. (3) Statute of Frauds A written easement’s creation generally is subject to the Statute of Frauds. The Statute, however, does not prevent the creation of an oral easement by means of a verbal conveyance or oral agreement to convey property interests. b) Methods of Creation Question Key 3,5,9,33,38,60,61,68,69,79,80, 83,111,136,137,142,164

Page 49

AMERIBAR BAR REVIEW REAL PROPERTY MBE OUTLINES REAL PROPERTY 49 Grantors may use several different methods to create easements. For example, a reservation in a deed that conveys an interest of property from a grantor to a grantee may create an easement. The method of creation depends upon the type of easement. (1) Express Easement by Grant A conveyance of a deed from a grantor to a grantee that allows a grantee to use the property may establish an easement by grant. To be valid, an express easement by grant must: • be written; • be executed by a grantor; • express the grantor’s intent to provide an easement to a grantee; • adequately describe the land that is subject to the easement; and • identify the grantor and the grantee. (2) Easement by Necessity An easement by necessity arises only when there has been a conveyance of a portion of the grantor’s land, the grantor retains the remaining portion, and, after severance of the grantor’s land, it is necessary for the grantee to pass over the grantor’s retained portion to reach a public street or highway. A party must make a strong showing of strict necessity for such an easement. There must be no other means of ingress or egress to the dominant estate’s land. Landlocked land would clearly satisfy the necessity test. In that event, a party need not show proof of pre-existing use. When such an easement by “way of necessity” is warranted, a court will require a servient estate’s owner to make a conveyance of an easement to the party. A strict necessity easement does not need to be in writing to be enforceable. The grant or conveyance implies this type of easement. In other words, it is implied that the grantor would have provided the grantee of a land-locked parcel with access to a public road. (3) Implied Easement / Easement by Implication An implied easement (also known as an easement implied from prior use) results from a non-owner’s ongoing, permitted, and lawful use of an owner’s estate. Although no express agreement creates such easements, they may be reduced to writing and/or judicially inferred from prior use if all essential facts exist. “The principle underlying the creation of an easement by implication is that it is so evidently necessary to the reasonable enjoyment of the granted premises, so continuous in its nature, so plain, visible and open, so manifest from the situation and relation of the two tracts that the law will give effect to the grant according to the presumed intent of the parties.” Rischall v. Bauchmann, 46 A.2d 898, 902 (Conn. 1946). An implied easement arises in favor of a grantee when (1) two parcels of land are in common ownership; (2) one of the parcels is conveyed to a grantee; (3) the parcel conveyed had been receiving a benefit from the parcel retained prior to the conveyance to the grantee; (4) the usage is reasonably necessary or convenient; and (5) the usage is apparent. The “reasonably necessary” requirement is usually interpreted to mean that the easement must be important to the enjoyment of the conveyed land or highly convenient. The “apparent” requirement is met when the usage is visible. Most courts will find an implied easement from prior use where access to the transferred land is extremely difficult by other routes. However, courts generally will not find an implied easement when the transferred land has easy access to public roads in another direction. (a) Subdivision Example When lots are sold in a subdivision containing streets that access the lots, those lots include implied, appurtenant easements to use the streets for ingress and egress to their lots. This rule is an exception to the general rule that an existing use must result in an implied easement. The law usually does not permit the holder of an implied easement

Page 50

AMERIBAR BAR REVIEW REAL PROPERTY MBE OUTLINES REAL PROPERTY 50 to improve or upgrade it further than the parties who established the easement reasonably contemplated at the time of its creation. (4) Easement by Prescription Prescriptive easements allow a non-owner of an estate who is using an owner’s estate to obtain a legal right to use the estate if the non-owner fulfills these conditions: • The non-owner engages in open and notorious use of the estate; • The non-owner’s use of the estate is hostile (no permission); • The non-owner’s use of the estate is continuous; and • The non-owner’s use is uninterrupted for a legally-required period (depends upon the jurisdiction). (a) Requirement of Use Some states may require that the non-owner engage in actual use of the property to obtain a prescriptive easement upon it. The doctrine of prescriptive easement focuses on the element of use, as compared with the adverse possession doctrine’s focus upon the element of possession. Both doctrines may, for example, apply to an access driveway. In that case, the factual situation will determine if an adverse claimant is entitled to just use of, or full possession of, the driveway. c) Scope and Apportionment (1) Appurtenant Compared to In Gross (a) Appurtenant Easement An appurtenant easement is an easement that is attached to a dominant estate. An owner of dominant property and an owner of servient property may create an appurtenant easement by an express written or verbal agreement. The owner of the dominant property may reasonably exercise rights that are created by and arise from the easement after it is created. This owner, as the easement holder, receives the benefit of physical use of a servient estate. The fact that those estates are adjacent does not, of itself, establish that the dominant estate owner is the holder of an appurtenant easement. (b) Easements in Gross Easements in gross belong to and are attached to an easement holder. An easement in gross is not attached to its holder’s real property; it only burdens a servient estate. An easement in gross benefits no dominant estate. Easements in gross, for example, authorize utility companies to place their conduits on or in an owner’s property. (i) Creation Operation of law or an express written or verbal agreement by an owner of servient property and either a non-owner of property or an owner of non-adjacent property may create an easement in gross. The non-owner of property or owner of non-adjacent property may reasonably exercise rights that are created by and arise from the easement after it is created. (2) Affirmative and Negative Easements (a) Affirmative Easements

Page 51

AMERIBAR BAR REVIEW REAL PROPERTY MBE OUTLINES REAL PROPERTY 51 Affirmative easements empower a dominant estate owner to engage in permissible conduct on the servient estate. For example, affirmative easements may enable a dominant estate owner to cross the servient estate for the purposes of ingress and egress. Generally, a written easement needs specific language to that effect to establish the appurtenant nature of those crossings. An example of an appurtenant easement is an access road that allows ingress and egress to its holder’s estate by the holder, invitees, and licensees. (b) Negative Easements Negative easements authorize a dominant estate’s owner to prevent a servient estate’s owner from engaging in some otherwise permissible conduct on the servient estate. Negative easements, for example, preclude a servient estate owner from erecting a barrier that prevents a dominant estate owner from crossing the servient estate for ingress and egress. (3) Rights of Possession and Use (a) General Considerations An easement is a right of use of land without a right of possession. For example, an affirmative easement would be a property owner’s grant of permission to a holder to freely roam the property for recreational purposes. An example of a permissible negative easement is one that provides a dominant estate’s owner with a right against interference with “light, air, and view.” (b) Duration of Easements The language that is used to establish an easement may determine the easement’s duration. If the easement is created in a manner other than in writing, then its duration will depend upon the circumstances that surround its creation. For example, if a structure on a servient estate is essential to an easement, then the structure’s destruction will end the easement. An easement is extinguished when the same party owns both of its estates. (c) Maintaining Easements The holder of an express easement possesses a duty to maintain the easement. That holder also possesses a right to enter upon the servient estate’s land to make reasonable repairs of an easement. The servient estate owner may not prohibit such repairs. (d) Modification of Easements In order to accommodate change, parties may make reasonable modifications to express rights of easement that are generally described. An easement may also, however, prohibit modifications. d) Transfer The transferability (i.e., alienability) of easements depends on the type of easement involved. (1) Easements in Gross Easements in gross are alienable and may only be transferred if they are for a commercial purpose. (2) Appurtenant Easements Appurtenant easements are subject to, and included with, a conveyance of a dominant estate. They also are usually conveyed with a servient estate unless the buyer lacks notice of the easement and buys the premises in good faith.

Page 52

AMERIBAR BAR REVIEW REAL PROPERTY MBE OUTLINES REAL PROPERTY 52 (a) Dominant Estate An appurtenant easement may only benefit the dominant estate to which it is appurtenant. If the easement holder uses the easement to benefit any other property, directly or indirectly, the courts may extinguish the easement. (b) Servient Estate A property owner of a servient estate is entitled to use the land in a manner that does not unreasonably or unduly interfere with an easement. The holder of an easement is not necessarily entitled to an exclusive right to use the area that is subject to the easement. That area may be subject to concurrent use. e) Termination of Easements (1) Abandonment Abandonment of an easement may result from non-use by its holder and the holder’s demonstrated intention not to use the servient estate again. Note that non-use is not enough. The party must also demonstrate an intention not to use the servient estate again. For example, a railroad company might not have used its easement for many years and could remove the train tracks that are upon it. The removal of the tracks would demonstrate the intent not to use the servient estate again. (2) Release A written release of an easement by its holder to the servient estate’s owner will end the easement. (3) Adverse Possession Adverse possession of an easement by the servient estate owner will terminate the easement if he fulfills the following elements: adverse interests, actual use, open use, notorious use, hostile use, non-permissive use, and continuous use, for the statutory period. (4) Destruction Destruction of a servient estate will end an easement. (5) Condemnation Condemnation of a servient estate by a government will end an easement. (6) Necessity If the need for an easement by necessity ends, the easement will expire. (7) Estoppel Estoppel will end an easement when an easement holder communicates to the owner of the servient estate that the easement will not be enforced, and the servient estate owner acts in reasonable reliance upon that assurance. (8) Merger

Page 53

AMERIBAR BAR REVIEW REAL PROPERTY MBE OUTLINES REAL PROPERTY 53 When the owner of the dominant estate acquires the servient estate, a merger results, and the easement is extinguished. The easement is extinguished because after the post-sale unity of ownership of the two estates, the dominant estate owner could enforce the easement only against himself, and lawsuits against one’s self are impossible. (9) Misuse Misuse of an easement does not usually terminate the easement, but may give rise to claims for legal or equitable remedies. f) Legal and Equitable Remedies (1) Legal Remedies (a) Actions to Construe Easements Legal proceedings may be necessary to interpret and determine the scope of easements. (b) Actions for Damages (i) Injury to Land (A) Unavoidable Harm If a dominant estate owner reasonably exercises a right to repair an easement, which results in unavoidable or necessary harm to a servient estate, the servient estate’s owner cannot complain about the harm in order to recover damages. (B) Unreasonable Harm The servient estate owner may, however, recover damages from a dominant estate owner for injury to its land due to unreasonable harm that resulted from repairs of an easement that the dominant estate holder made. The damages are usually limited to the amount that is required to restore the land to its pre-existing condition. (ii) Tort Liability If a holder of an express easement negligently breaches the holder’s duty to maintain the easement, the holder may face tort liability for any consequent injury to property, persons, or deaths. (2) Equitable Remedies (a) Actions for Injunctive Relief Proceedings in equity may be necessary to obtain injunctive relief regarding an easement. With respect to prescriptive and implied easements, the majority rule is that, where an easement was intended for accessing a first parcel of land, but the grantee instead used it to access a second parcel of land, such use automatically constitutes an "overburdening" of the easement, and may be enjoined by the owner of the servient estate. (b) Actions to Enforce Oral Easements Courts may enforce an easement arising from a verbal conveyance or oral agreement to convey property interests on two equitable grounds. The equitable doctrines of part performance and estoppel allow for such enforcement of this exception to the Statute of Frauds. Those doctrines apply under these circumstances:

Page 54

AMERIBAR BAR REVIEW REAL PROPERTY MBE OUTLINES REAL PROPERTY 54 • an oral grantor informs an oral grantee that; • the grantee possessed an easement or profit on certain land; • the grantee relied on the grantor’s representation; and • the grantee acted in detrimental reliance upon the representation by; • spending funds, engaging in labor, or improving the easement or profit. 3) PROFITS a) Nature and Characteristics (1) Nature of a Profit A profit (i.e., a profit à prendre) is a legal right to enter onto certain real property (“property”), that is a servient estate, and to remove natural resources from this property such as minerals, timber, or wildlife. (2) Characteristics of a Profit A valid profit usually includes these features: • a written instrument; • a certain duration; • a particular area; • a payment of significant consideration; • the ability to exert control over the interest by improvements; and • not being terminable at will by an owner. (a) Duration A profit exists either perpetually or for a duration that the language used to establish it specifies. (b) Statute of Frauds The Statute of Frauds requires a legally sufficient written document to establish and enforce a profit. b) Methods of Creation A reservation in a deed that conveys some interest in property from a grantor to a grantee may create a profit. c) Scope and Apportionment There are two main types of profits--profit appurtenant and profit in gross. (1) Profit Appurtenant A profit appurtenant provides benefits to an owner of a dominant estate that arise from, and are inseparable from, a parcel of land that belongs to an owner of a servient estate. The fact that those estates are adjacent does not, of itself, establish that the dominant estate owner is the holder of a profit appurtenant. An instrument of conveyance must contain specific language to that effect to establish the appurtenant nature of the profit. (2) Profit in Gross Question Key 124

Page 55

AMERIBAR BAR REVIEW REAL PROPERTY MBE OUTLINES REAL PROPERTY 55 A profit in gross provides benefits that only personally serve its holder and are separate from the profit holder’s use or ownership of any property. A profit in gross usually involves the removal of a natural resource from property. It does not include a dominant estate in addition to a servient estate. Sometimes the holder of a profit in gross may not own property that the servient estate benefits. (3) Rights of Possession and Use (a) Rights of a Profit Holder (i) Possession and Use A profit is a right of use of land without a right of possession. Thus, a profit holder lacks a right to possess the property that is subject to the profit. Instead, the profit holder has only the right to take certain resources from the property. (4) Rights of a Property Owner (a) Use The owner of a servient estate has the right to use the land in a manner that does not unreasonably or unduly interfere with a profit. (b) Possession Like easements, a profit imposes a burden on a servient estate owner’s right of possession. In that respect, profits are generally subject to the rules governing easements. d) Transfer A profit in gross is transferable (i.e., alienable). The intent of a grantor of a profit in gross is determinative of whether a non-exclusive profit is divisible. A transfer of a profit to two or more persons who will use a profit “as one stock” for business purposes is not a prohibited division of the profit. e) Termination A written release of a profit by its holder to the servient estate owner will end the profit. f) Legal and Equitable Remedies The courts generally seek to allow a proper use of a profit by its holder while disallowing or sanctioning its improper use. (1) Legal Remedies (a) Property Owner’s Remedies The legal remedies that are available to a property owner for an improper use of a profit by a profit holder may include a recovery of damages or a termination of the profit. (b) Profit Holder’s Remedies A profit holder may bring an action to receive compensation from a government that takes that property interest by condemnation using the power of eminent domain.

Page 56

AMERIBAR BAR REVIEW REAL PROPERTY MBE OUTLINES REAL PROPERTY 56 (2) Equitable Remedies The equitable remedies that a property owner may seek for an improper use of a profit may include injunctive relief from a profit holder’s use of the profit. 4) LICENSES a) Nature and Characteristics (1) Nature of a License A property owner or possessor may grant a license that enables a non-owner of a property, as a license holder, to access the property for a particular reason and duration. A license is "[a]n authority to do a particular act or series of acts upon the land of another without possessing any estate or interest therein." Black's Law Dictionary (3rd Ed.). A license protects a non-owner from being considered a trespasser while on the owner’s property. Some examples of a license would be a temporary parking permit or a ticket for admission to an entertainment event. (2) Characteristics (a) General Elements The following features identify a license: • a specified or uncertain duration; • a particular or general area; • a payment of minimal consideration to obtain access; • the inability to control a property interest by improvements or repairs; and • terminable at will by an owner unless it is irrevocable. (b) Additional Terms A license may include disclaimers of warranty regarding premises liability or responsibility for personal injury or death on the premises. b) Methods of Creation (1) Verbal or Written Licenses The Statute of Frauds does not apply to a license because a license is not considered an interest in land. A license may be verbally granted or may be reflected in a formal writing or by a ticket of admission. c) Scope and Apportionment (1) Revocable License A license is revocable earlier than anticipated at the will of a property owner. (2) Irrevocable License A license coupled with an interest is irrevocable for a reasonable duration. Question Key 64,146

Page 57

AMERIBAR BAR REVIEW REAL PROPERTY MBE OUTLINES REAL PROPERTY 57 (a) License Coupled with an Interest A license coupled with an interest is a privilege incidental to the ownership of an interest in a personal property located on the land with respect to which the license exists. For example, under a loan agreement, a car lender may possess a license to enter the borrower's land to repossesses the car upon default of the loan. As a result, the lender possesses a license coupled with an interest. (b) Executed License An executed license, which is a license pursuant to which some action has been taken, such as the expending of money or labor, is irrevocable. (3) Rights of Possession and Use (a) Non-Owner’s Rights A license is a non-owner’s limited privilege of access upon and right to use and occupy a property owner’s land or premises. A license does not convey any right to possess the land. (b) Owner’s Rights A property owner possesses the right not to issue a license. A property owner may only revoke a license under certain circumstances. The doctrine of estoppel prevents a property owner from revoking a license if a non-owner makes an expenditure in reliance on the license. Similarly, a license that a non-owner executes provides an easement by estoppel that the property owner cannot revoke if the non-owner reasonably relies on the license to its detriment. d) Transfer Because a property owner decides whether to issue a license and its scope, the property owner controls whether the license holder may transfer the license. For a transfer to be valid, the property owner must have allowed it. e) Termination A license may provide that it exists for, and ends at, the expiration of a limited duration. A property owner may terminate a revocable license by revocation at will. f) Legal and Equitable Remedies (1) Legal Remedy A recovery of damages is a property owner’s legal remedy for a non-owner’s improper use of a license. (2) Equitable Remedy Injunctive relief is an equitable remedy for an improper use of a license. C. Fixtures It is important to be able to determine if a piece of personal property becomes a fixture of the land. If it becomes a fixture, then generally, it passes with the land and cannot be removed (absent agreement). 1) COMMON LAW Question Key 41,131,143

Page 58

AMERIBAR BAR REVIEW REAL PROPERTY MBE OUTLINES REAL PROPERTY 58 a) Elements of a Fixture An item is a fixture if it satisfies the following three elements: • Chattel It is a chattel, an item originally of tangible personal property; • Attached It is attached to real property or a structure that is located on real property; • Injury It cannot be removed without injury to the freehold, such as real property or a structure to which it is attached. Severance of a fixture from the real property does not alter its classification as a fixture. b) Fixtures in Leasehold Tenancies When a leasehold estate holder attaches a chattel to the estate property for any purpose, the courts usually do not consider this attachment permanent. The courts often allow the holder to remove such a “fixture” during the tenancy. Alternatively, the holder’s representative may remove it within a reasonable period from the holder’s death. Generally, the removal may not occur if it would result in: • substantial injury to property to which it is attached; or • substantial destruction of the attached chattel. c) Trade Fixtures Doctrine (1) Definitions Trade fixtures are personal property attached to the property as an essential element of conducting a tenant’s business. Usually such fixtures are not especially adapted or fitted to the leased premises and may be used in other premises for business purposes. (2) General Rules The trade fixtures doctrine generally allows a tenant to remove fixtures from leased premises unless that removal would cause serious harm to the premises. The tenant must remove the fixtures during the term of a lease of a specified duration or within a reasonable period after a lease ends. The tenant is liable to the landlord for the cost of repairing any damage that results from the removal of the fixtures. d) Accessions Doctrine The accessions doctrine generally provides that if a tenant expends labor and cost to attach a chattel to a landlord’s property in a situation that does not allow the chattel’s removal, then the landlord is entitled to ownership of the chattel. e) Fee or Freehold Estates

Page 59

AMERIBAR BAR REVIEW REAL PROPERTY MBE OUTLINES REAL PROPERTY 59 A question may arise as to whether certain originally personal property will be transferred with an estate as a result of a sale or conveyance of the estate. (1) General Rule Usually, if personal property is attached to real property, it will be considered an integral part of the real property under two conditions: • removal of the personal property would substantially damage the real property; and • the attaching party intended that the personal property would become a part of the real property. (2) Transferability When the personal property becomes an integral part of the real property, it is subsequently owned with, and will be transferred with, the real property. (3) Priority of Claims When an owner of personal property perfects a lien in that property before affixing it to real property, the owner may obtain priority over other creditors’ claims against the real property, such as those of a subsequent mortgagee of the real property. D. Zoning 1) ZONING LAWS a) Nature and Characteristics “The essence of zoning is territorial division in keeping with the character of the lands and structures and their peculiar suitability for particular uses, and uniformity of use within the division.” Katobimar Realty Co. v. Webster, 20 N.J. 114, 118 A. 2d 824 (1955). b) Bases for Creation (1) Enabling Acts The legal basis for local zoning provisions in the form of regulations, ordinances, or codes is usually a state’s enabling act or statute. The zoning power is not generally premised upon a government’s police power obligation to provide for the public’s health, safety, morals, and welfare under the Tenth Amendment to the U.S. Constitution. (2) Home Rule Sometimes, local governments, such as counties or municipalities, impose zoning provisions pursuant to home-rule powers that a state provides. Those provisions may pre-exist or follow the creation of a master plan that establishes the vicinities of, and defines the various types of, zones of use. c) Rights of Possession and Use The zoning category that applies to the owner’s property will determine a property owner’s rights of possession and use. (1) Nonconforming Uses

Page 60

AMERIBAR BAR REVIEW REAL PROPERTY MBE OUTLINES REAL PROPERTY 60 Under some circumstances, a grandfather provision will allow the pre-existing and nonconforming use of the property that occurred before the government imposed zoning restrictions to continue. (a) Just Compensation Other enabling statutes provide that zoning provisions cannot mandate the elimination of nonconforming uses without providing just compensation. Zoning statutes that eliminate nonconforming uses without providing just compensation are subject to invalidation as unconstitutional because they constitute a denial of due process or operate as a regulatory taking. 2) PROTECTION OF PRE-EXISTING PROPERTY RIGHTS a) Legal and Equitable Remedies A property owner who opposes or contests the imposition of, or restrictions upon land use that result from, a zoning category may seek a variety of legal and equitable remedies for relief from the effects of that category. Those remedies may be provided by common law or statute. Statutes may prescribe remedies through the use of administrative-law type procedures and processes. Three common forms exist for requesting relief from zoning provisions: variances, special permits, and rezoning. (1) Variances A variance occurs when the government office or agency that administers zoning provisions grants a property owner an exception from the zoning laws. The two types of variances are use variances that apply to a particular parcel of real property and bulk variances that apply to any parcels of real property that the zoning ordinance affects. (a) Variance Standard A common legal standard for obtaining a variance is that the enforcement of a zoning provision would result in an unnecessary hardship to a property owner. Hardship usually exists when, absent the variance, a landowner cannot make reasonable use of the land. (2) Special Use Permit A special use permit is a right to conduct certain activities, termed conditional uses, within a zoning district. A property owner must obtain special approval from the zoning authority for these permits. Some jurisdictions call this type of permit a conditional use permit. For example, suppose a local government allows, as a special use, the operation of a recreational facility within its residential zoning districts. A landowner wishing to operate a recreational facility must apply for and obtain a special use permit from the local governing authority. 3) REZONING AND OTHER ZONING CHANGES a) General Aspects of Rezoning A rezoning may occur when the government office or agency that administers zoning provisions decides to revise and modify them. Some jurisdictions refer to rezoning as amending a zoning ordinance. A property owner may challenge a rezoning that adversely affects his interest in or use of the affected land. (1) Owner Has Right Against Rezoning Owner's Land, Not Others' Land

Page 61

AMERIBAR BAR REVIEW REAL PROPERTY MBE OUTLINES REAL PROPERTY 61 Although an owner has a right that his land will not be quickly rezoned to the owner's disadvantage, he has no right against rezoning other land. Herbert Hovenkamp and Sheldon F. Kurtz, Principles of Property Law (Concise Hornbook Series), 6th ed., Thompson West, 2005, p. 467. b) Spot Zoning (1) Definition "Spot Zoning" means that only one property in an area is rezoned. Denise L. Evans, J.D. and O. William Evans, J.D., The Complete Real Estate Encyclopedia, The McGraw-Hill Companies, Inc., 2007. Spot Zoning is a variation in zoning when some land is zoned differently than all other surrounding land. Black's Law Dictionary, 2d ed. (2) Judicial Treatment Usually, the courts are not sympathetic to Spot Zoning resulting from rezoning that affords a discriminatory benefit to one owner to the disadvantage of others. Hovenkamp and Kurtz, Principles, p. 468. A court can hold Spot Zoning unlawful where it deviates from a comprehensive zoning plan because of its arbitrary, capricious, and unreasonable zoning of some land in relation to the surrounding land. For example, within an industrial park zoned for commercial buildings, it would probably be unlawful to zone one parcel as residential to accommodate construction of a single-family dwelling there. c) Contract Zoning (1) Definition Sometimes called Conditional Zoning, Contract Zoning occurs when a zoning authority provides for a private owner's interest by rezoning land via a land use regulation based on their agreed upon condition that the owner accept the restrictions or limitations applicable to that land but not necessarily other similarly zoned land. Darryl C. Wilson and Cynthia H. Debose, Mastering Property Law, Carolina Academic Press, 2011, p. 180. (2) Power to Act and Court Scrutiny A zoning authority has only limited power to exact concessions or impose conditions relative to an owner's request for rezoning or a variance. Hovenkamp and Kurtz, Principles, p. 453. The courts scrutinize such situations carefully. Id. The zoning authority may not act in excess of its lawful powers. Id. The courts usually distinguish between personal restrictions or limitations that are unrelated to the zoning decision at issue, which are not allowed; and restrictions on the land itself that are related to the zoning's purpose, which are allowed. Id. Many courts would authorize conditional rezoning unless it would be against the public interest. Joseph William Singer, Introduction to Property, Aspen Treatise Series, 2d ed., 2005. 4) APPELLATE PROCEDURE A property owner whose request for a variance or rezoning is denied at an initial administrative level, such as a local zoning commission, may seek agency review by a local zoning board of appeal. If no other level of agency review is available, the owner who seeks to contest the board of appeal’s decision may obtain judicial review of it. a) Due Process In addition to raising substantive issues on appeal, a property owner may assert a violation of procedural or constitutional due process if the contested agency action or decision does not comply with the procedures that either the state enabling act or the local zoning provision established. At a minimum, such procedures should afford a property owner with notice and a hearing in actions for rezoning or variances. Other issues include violations of constitutional equal protection or regulatory taking. E. Other Interests in Land

Page 62

AMERIBAR BAR REVIEW REAL PROPERTY MBE OUTLINES REAL PROPERTY 62 1) SCOPE AND EXTENT OF REAL PROPERTY a) Surface, Superjacent, Adjacent, and Subjacent Space (1) Nature and Characteristics (a) Surface Space Surface space is the ground level of the land that comprises an owner’s real property and surrounds the structures that are located on that property. Surface interests extend to the top and a minimal depth of the owner’s real property. (b) Superjacent Space Superjacent space is above the surface of an owner’s real property and/or structures that are located on that land. Superjacent interests extend into the airspace that is above an owner’s real property and the structures that are on that land. (c) Adjacent Space Adjacent interests extend along the edge of the ground that surrounds, or borders upon, an owner’s real property and the structures that are on that land. (d) Subjacent Space Subjacent space is below an owner’s real property and/or structures on that land. (2) Rights of Possession and Use (a) Surface Space A surface property owner may exercise the rights of possession and use upon and within that real property and any structures on the land. (b) Superjacent Space A superjacent property owner may exercise its rights of possession and use above that real property and any structures upon the land. Under the common law, those rights extended “to the heavens,” although the modern trend is to limit those rights to the height that a property owner could make use of them. (c) Adjacent Space An adjacent property owner’s rights of possession and use extend along the edge or border of that real property and any structures upon the land. (i) Lateral Support The corollary right of lateral support from adjacent land and under the ground’s vertical surface is absolute. Absolute liability arises from a violation of this right. Lateral support protects real property and/or structures on the property from excavation, erosion, or other harms that are caused by acts or omissions of an owner of adjacent real property. (A) Exceptions Question Key 23,134,172 See also Contracts 197 & 198

Page 63

AMERIBAR BAR REVIEW REAL PROPERTY MBE OUTLINES REAL PROPERTY 63 Structures: Lateral support is not an absolute right if a structure that is built upon the land artificially places weight upon the natural forces that hold the ground intact. Act of God: Neither a landowner nor an adjacent landowner is liable if a lateral support is withdrawn as a result of some event of nature or weather such as erosion or a flood. (d) Subjacent Space A subjacent property owner’s rights of possession and use extend indefinitely downwards below that real property and any structures upon the land. A subjacent owner would have subterranean, or below surface, rights to recover natural resources like precious stones, minerals, oil, and gas. (i) Subjacent Support The right of subjacent support arises from land below the ground’s horizontal surface. For example, a surface owner may possess a right to subjacent support of real property and/or structures on that property from certain types of harm arising from an underlying mineral rights owner’s below-ground activities, such as tunneling. (3) Legal and Equitable Remedies The general interests of surface, superjacent, adjacent, and subjacent space include a right to exclusive possession and use that is free from unlawful invasions or interferences. Violation of those rights of possession gives rise to causes of action arising from trespasses upon, or nuisances that harm, those rights. A property owner could also bring a negligence or strict liability action for damage to personal property. (a) Trespass Trespass mainly involves an actual physical invasion of the property owner’s possessory interest by a person or a thing. A legal privilege to enter upon land renders a trespass lawful. If a person with legal privilege takes actions beyond the permitted activities, though, such a person may become a trespasser ab initio. Examples of trespass include all of the following, if done without right: • the construction of a well or pipeline under real property; • a tree that intrudes a possessory interest in the airspace of property; • aircraft flights over property that cause actual damage to property; and • physical entry upon real property. (b) Nuisance Nuisance usually involves a “nonphysical” invasion of the property owner’s possessory interest by vibration, noise, or odors. Liability for nuisance might result from aircraft flights over property. Depending on their frequency and the severity of those flights, a property owner may recover damages, and/or an inverse condemnation might occur. The defense of “coming to the nuisance” is not absolute. Under this defense, a plaintiff is not entitled to recover for a nuisance if the plaintiff moved near that pre-existing nuisance. (i) Private Nuisance (A) Intentional An intentional private nuisance is:

Page 64

AMERIBAR BAR REVIEW REAL PROPERTY MBE OUTLINES REAL PROPERTY 64 • an unreasonable activity or condition; • on a neighboring owner’s real property that; • unreasonably or substantially interferes with a property owner’s use and enjoyment of the land. This interference is measured by the “reasonable person” test. (B) Unintentional Under the Restatements of Torts, an unintentional nuisance is conduct that could be actionable as either negligent or ultra-hazardous. (ii) Public Nuisance A public nuisance is similar to a private nuisance, except it affects more than just a few property owners. It extends to the general public or a much larger number of people. Usually a property owner may bring a private nuisance action, and a prosecutor may bring a public nuisance action. A public nuisance may also arise from illegal activities in violation of an ordinance. An example of such an activity is operating a brothel. (iii) Remedies (A) Legal Remedies A prevailing party in a nuisance action may recover out-of-pocket expenses incurred as a result of the nuisance and the loss of value in the property. (B) Equitable Remedies Injunctive relief from a nuisance may be available within a trial court’s discretion. An equitable defense to such relief is balancing the hardships or the equities. An injunction may completely prohibit the nuisance or be narrowly tailored to minimize the extent of the nuisance. The person seeking injunctive relief must assert some substantial injury to himself or herself. (c) Removal of Support (i) Strict Liability Strict liability exists if: • a neighboring property owner removes lateral support from another property owner’s adjacent property in its natural state; and • the other owner’s property would have been adequately supported in its natural condition. A neighboring property owner has a burden of proving that the other property owner’s adjacent land would have subsided in its natural condition. The neighboring property owner may be liable for damages to the other property owner’s improvements and harms to the land. (ii) Negligence Liability Negligence liability exists when a neighboring property owner engages in activities that could cause subsidence of another property owner’s adjacent land. The neighboring owner has a duty to provide notice of such activities to all other property owners. The neighboring owner must conduct those activities with the exercise of reasonable care to maintain lateral support of the adjacent land. The neighboring property owner is liable for a breach of those duties if the breach results in foreseeable harm.

Page 65

AMERIBAR BAR REVIEW REAL PROPERTY MBE OUTLINES REAL PROPERTY 65 (iii) Vicarious Liability An employer of an independent contractor is not exempt from tort liability when the contractor’s excavation activities were inherently dangerous, despite the employer’s lack of control of those activities. b) Rights in Common Resources (1) General Considerations Generally, it is difficult for an owner of real property to obtain an exclusive right to the light and air that surrounds that real property. Usually, an owner must receive an express grant to create an easement for light and air. Property owners cannot imply such easements, despite the existence of other elements for implying a quasi-easement, because such an easement would not be evident to a buyer of the servient estate. The courts generally will enforce express negative appurtenant easements for light and air. The courts, however, usually decline to enforce a prescriptive easement for light and air. (2) Light Some case law provides that a property owner may erect a structure in the airspace above the property that blocks the sunlight or views of the neighboring property owners. (3) Air A property owner may erect a structure in the airspace above the property that interferes with the neighboring property owners’ reception of television or radio signals. (a) Weather Modification Southwest Weather Research, Inc. v. Rounsaville is a decision in a property owner’s action seeking to prevent a neighboring property owner from modifying the weather over that land. The court concluded that a property owner has a right to all naturally existing clouds and any rainfall that those clouds produce above that owner’s land. Consequently, the court held that one property owner may enjoin a neighboring property owner from engaging in weather modification in airspace above the property owner’s land. 320 S.W. 2d 211 (Tex. 1959). (4) Riparian Rights (a) Moving Water Moving water is water contained in rivers, streams, and creeks. Owners of property along those channels of water possess certain riparian rights to use those watercourses. (b) Stationary Water Stationary water is water contained in reservoirs, lakes, and ponds. Owners of property along stationary water possess riparian rights to access those bodies of water. (c) Reasonable Use Doctrine (Majority Rule) The modern reasonable use doctrine applies in a majority of the states. It provides that a riparian property owner may reasonably use the water and cannot unreasonably interfere with the other riparian owners’ ability to reasonably use the water. Question Key 24,50,112

Page 66

AMERIBAR BAR REVIEW REAL PROPERTY MBE OUTLINES REAL PROPERTY 66 (i) Balancing Test A judicial determination on whether a particular use of the water is permissible will depend upon a balancing of the following factors: • the purpose and importance of a use; • the social and economic value of a use; • the impact of a use on other riparian owners; • the appropriateness of the use to a specific watercourse; and • the feasibility and expense of adjusting a use or of other uses. A domestic use usually prevails over other uses. Water use for irrigation may be of secondary importance. (d) Colorado Doctrine (Minority Rule) The Colorado Doctrine of prior appropriation applies in many of the western states. The doctrine provides that an initial user of water for a “beneficial purpose” acquires the right to continue that use. This right of the initial user, who need not be a riparian property owner, is subject to using a decreased amount of water if a subsequent user can show that water is needed for a preferred beneficial purpose. (5) Underground Water Underground water in percolating aquifers and subterranean streams may form or be a part of the water table that could extend under the surface land of several property owners. (a) Underground Streams and Aquifers If the water flows in a subterranean stream or aquifer, then the same riparian rules that are described above will apply. The majority rule of reasonable use applies to limit any taking of underground water that harms other surface property owners. (6) Surface Water Diffuse surface water is water that results from the melting of snow, rainfall, overflowing springs, or a diversion of those types of water by one neighbor upon another neighbor’s property. (a) Impounding Surface Water An owner may impound diffuse surface water, especially when this is done in the absence of any malice towards others. (b) Expelling Surface Water (i) Reasonable Use (Majority Rule) There are different views regarding the way an owner may expel surface water. First, the reasonable use doctrine applies in several states. It provides that a property owner is privileged to reasonably use his land and to only modify its surface water drainage so that it would not unreasonably interfere with a neighboring property owner’s use of the land. (A) Balancing Test

Page 67

AMERIBAR BAR REVIEW REAL PROPERTY MBE OUTLINES REAL PROPERTY 67 A court may determine a reasonable use issue by considering the following factors: • the degree of necessity for a property owner to modify the drainage onto another’s land; • if the property owner made the modification with reasonable care; • if other more beneficial modifications were possible; and • the extent of damages that the neighboring property owner incurred. (ii) Common Enemy Doctrine (Minority Rule) A minority of states follows the common enemy doctrine. It allows a property owner to utilize any means to remove surface water without becoming liable to neighboring property owners for any resulting flood upon their land. This rule is subject to several qualifications. One imposes liability for the collection of and expulsion of surface water in greater quantities than those naturally present. Another imposes liability for a property improvement that results in greater water runoff. Lastly, an unreasonable alternative means of drainage may result in greater liability. (7) Navigable Water Navigable water is subject to the federal and state governments’ “navigation servitudes.” They limit a riparian property owner’s rights under the federal navigation power arising from the Commerce Clause of the U.S. Constitution. (8) Vegetation (a) Definition Natural vegetation, or fructus naturales, belongs to a surface property owner. It includes perennial shrubs, grasses, and trees that grow on the ground. Adjacent property owners own trees growing along a border of real property as tenants in common. (b) Status and Rules Fructus naturales is real property until it is severed from the ground. It passes with a transfer of the land and is subject to a mortgage unless otherwise agreed. In the absence of immediate severance of such natural vegetation from real property by a vendor for a vendee, the sale of that vegetation is a sale of an interest in land. Thus, the Statute of Frauds requires that a sale without a severance be reflected in writing. (9) Crops (a) Definition Crops, or fructus industriales, belong to a surface property owner. They are annual plants that are cultivated through human effort and intended for human or animal consumption. (b) Status and Rules Growing crops constitute personal property subject to the laws of wills, trusts, and intestacy. They pass with a transfer of the land and are subject to a mortgage unless otherwise agreed. They may be subject to an agreement of sale before the time of harvest. (10) Emblements

Page 68

AMERIBAR BAR REVIEW REAL PROPERTY MBE OUTLINES REAL PROPERTY 68 Qualified estates and leasehold estates of an uncertain duration include a license or right of emblements. The right applies in the following circumstances: • A qualified estate holder or a tenant for an uncertain period (e.g., a life estate or a tenancy for an uncertain period that was not terminated for a tenant’s misconduct) plants crops; • The crops exist in one growing season when the estate ends (but not thereafter); • The crops need to be harvested and removed from the land; and • An instrument of conveyance, or lease, or will prohibits emblements. The doctrine of emblements entitles a tenant or the representatives of a deceased life estate holder to harvest the crops. The doctrine applies to estates of uncertain duration but does not apply to leasehold estates of certain duration. F. Taking 1) TYPES OF TAKING A taking occurs when a government appropriates a private owner’s real property for public use. Under the Takings Clause of the U.S. Constitution, the owner of property that is taken for public use is entitled to just compensation. A less traditional type of taking occurs when the government appropriates private property for a specified type of public purpose, such as an economic development plan under certain limited circumstances. 2) APPLICATION If a property owner declines to sell or dedicate real property to a government for public use, then the government may exercise its legal power to acquire that property. a) Eminent Domain Eminent domain is one type of power that the government may assert to acquire land that is needed for the purpose of public transportation or government buildings. b) Regulation Regulation is another type of less extreme power that the government may impose to apply certain restrictions on, or zoning provisions about, land use. A regulatory taking may occur when a state enabling act and/or a local zoning provision: • deprives a property owner of reasonable use of its parcel; • without providing the constitutionally required just compensation. 3) INTERESTS SUBJECT TO TAKINGS A government may appropriate all types of private property interests by condemnation. a) Freehold Estates If the interest taken is a freehold estate, the taking terminates an owner’s interest, and the owner is entitled to compensation. b) Leasehold Estates If the interest taken is a qualified estate, then two situations may arise. Question Key 95

Page 69

AMERIBAR BAR REVIEW REAL PROPERTY MBE OUTLINES REAL PROPERTY 69 (1) Entire Leasehold Premises If the government condemns the entire leased premises, the action will cancel the lease. Accordingly, if the interest taken is an entire leasehold estate, then the leasehold and all duties under the lease end. Thus, the tenant would no longer be obligated to pay rent for any remaining duration of the lease. The owner of the property would be entitled to just compensation. In the absence of a controlling provision in the lease agreement, a tenant may be able to recover part of the compensation that a landlord receives. The part the tenant is entitled to is the amount of the leasehold’s fair market value for its remaining duration that exceeds the tenant’s lease obligations. Consequently, to ascertain if the tenant can recover any compensation from the landlord, one must determine if the fair market value of the lease exceeds the cost of the tenant’s remaining lease obligations. (2) Partial Leasehold Premises If a government condemns only part of the leased premises, the action will not cancel the lease. If the interest taken is only part of a leasehold estate, then a tenant remains obligated to pay the entire rent. A landlord has no duty to abate the rent obligation as a result of the government’s taking. In the absence of a controlling provision in the lease agreement, a tenant may be entitled to recover part of a landlord’s award of just compensation. The first type of recoverable compensation is the portion of the rent attributable to the taken part of the leasehold estate. The second type of recoverable compensation is the amount of the leasehold’s fair market value for its remaining duration in excess of the tenant’s lease obligations. In other words, the portion the tenant can recover will be equal to the capitalized cost of remaining rent on the condemned portion and any surplus value consisting of the excess of fair market value of the condemned portion of the lease compared to the costs of the remaining lease obligations. Accordingly, if the tenant had a good bargain on the lease, he will be entitled to the fair value of the good bargain. For example, suppose Tenant rents a warehouse for $10,000 per year. With one year remaining, before the final payment, the government exercises its power of eminent domain and takes one-quarter of the warehouse. The lease will not be cancelled. Tenant will still be required to pay $10,000 under the lease. However, he will be entitled to $2,500 from the condemnation award. This amount is the pro-rata amount of remaining rent under the lease for the condemned property. Additionally, if he had a good bargain, for example if the market value of the rental was $20,000, instead of $10,000, but Tenant negotiated a great deal, then Tenant will be entitled to the surplus value of the condemned portion of the lease. In this case, that amount would be an additional $2,500. 4) LEGAL AND EQUITABLE REMEDIES A property owner whose land is taken by the government without any condemnation proceedings may bring a civil action to recover damages in a reverse condemnation action or seek the equitable relief of an injunction against the taking. III. REAL ESTATE CONTRACTS A. Real Estate Brokerage 1) GENERAL CONSIDERATIONS a) Basic Definition of Real Estate Broker Essentially, a real estate broker is a professional licensed to engage in real estate transactions for a fee (e.g., particular amount) or commission (e.g., specified percentage).

Page 70

AMERIBAR BAR REVIEW REAL PROPERTY MBE OUTLINES REAL PROPERTY 70 b) Actions that a Real Estate Broker May Take A broker can be an individual or entity (e.g., partnership) that makes offers to sell or buy, sells or buys, or negotiates selling, buying, or exchanging real estate. A broker can rent or lease to others any real property or improvements upon it. Typically, brokers dispose of others' interest in real estate with which they deal, instead of their own interest in real estate. c) Broker Must Have License to Act Lawfully, as well as Education An individual broker must have a license to take those actions lawfully after fulfilling all applicable education prerequisites to conduct such real estate business. d) Actions Taken for Valuable Consideration or Compensation The broker can take those actions for valuable consideration or compensation (e.g., a fee or commission). The broker may hire agents from which the broker may earn a fee or commission on real estate that they sell. e) Real Estate Broker Distinguished from Real Estate Agent In some states, the word "broker" refers to all real estate agents. In many states, a broker is one level above a real estate agent by having the authority to supervise such agents and more training than they do. 2) REPRESENTATION IN REAL ESTATE TRANSACTION a) Parties May Contract for Professional Representation Parties may contract with professionals to represent them in relation to a real estate transaction. Wilson and Debose, Property Law, p. 121. (1) Representation of Seller by Agent or Broker Commonly, sellers may have a sales agent or a broker represent them. Id. (2) Representation of Buyer by Broker as Exclusive Agent Moreover, a buyer may contract with a broker to serve as the buyer's exclusive agent. Id. 3) COMMISSION FOR REAL ESTATE TRANSACTION a) Commission under Exclusive Right to Sell Contract Under an exclusive right to sell contract, a real estate broker has a right to a commission regardless of who sells the real estate subject to it, even the real estate owner, during the contract's term. b) Exclusive Agreement Provides Right to Commission If the owner enters into an exclusive agency or listing agreement, then the owners' sales agent or broker would have a right to a commission when the agent or broker sells the owner's real estate. Id., p. 123. However, the agent or broker would not receive a commission when the owner sells the real estate. Id. c) No Compensation for Efforts under Non-Exclusive Open Listing An agent or broker would not receive any compensation for their efforts made pursuant to a non-exclusive open listing agreement, when someone other than the agent or broker sells the real estate. Id.

Page 71

AMERIBAR BAR REVIEW REAL PROPERTY MBE OUTLINES REAL PROPERTY 71 d) Transaction Broker Also known as a non-agent broker, a transaction broker has a limited role of arranging the parties' real estate transaction without representing either of them as to their particular transactional objectives. Id. Each of the parties executes an agreement with a transaction broker acknowledging the transaction broker's particular, restricted duties. Id. e) Whether Agent or Broker Is Entitled to Commission An issue may arise as to whether an agent or a broker is entitled to a commission. (1) Common Law Provides Commission for Producing Buyer Under the common law, a broker is entitled to a commission after the broker has provided a buyer that is willing, able, and ready to purchase the real estate per the owner's terms of sale, regardless of whether the real estate is sold. Id., p. 124. (2) States Following Approach Departing from Common Law Many states follow a modern approach that replaces or rejects the common law and instead provides that a broker is entitled to a commission after a sale transaction occurs. However, an exception allows the broker to receive the commission without a sales transaction having occurred where the owner causes that nonoccurrence. Id. 4) ETHICAL CONSIDERATIONS a) Possible Liability for Unauthorized Practice of Law An agent or a broker may be liable for unauthorized practice of law by completing legal documents beside a contract relating to professional representation. Id., p. 124. An agent or a broker may complete particular legal documents drafted by lawyers, such as proposals, earnest money agreements, listing contracts, options, affidavits, offers to purchase, exchange agreements, purchase agreements, short form leases, bills of sale, and buy and sell contracts. Id.; State ex rel. Indiana State Bar Ass'n v. Indiana Real Estate Ass'n, 191 N.E.2d 711 (Ind. 1964). Only persons authorized to practice may execute legal documents (e.g., deed to real property). Id. Although an agent or a broker may complete the data entry fields of a standard sales agreement, the agent or the broker may neither draft nor complete a legal document (e.g., deed, mortgage, sales agreement or addendum to it) or other form of paperwork required to complete a transaction. Id., p. 124. b) Fiduciary Duties A broker has a fiduciary duty to the person that the broker represented. Id., p. 125. Usually, a broker may only represent one party to a transaction unless both of the parties to the transaction have received sufficient disclosure to provide informed consent. Id. c) Legal Obligations Regarding Physical Defects in Real Estate Usually, a broker has a duty to disclose material, physical defects that the broker knows that the particular real estate has. Id. Likewise, typically a broker will not be liable for remaining silent about physical defects of which they are not aware. Id. Only a few states impose a duty upon a broker to diligently inspect the owner's real estate for any undisclosed defects and to reveal those defects to the buyer. Id. Commonly, brokers' agreements include exculpatory terms placing the burden upon the parties as to knowledge of defects. Id. B. Types of Contracts 1) CONTRACTS TO SELL AND BUY Question Key 86,125,163,167

Page 72

AMERIBAR BAR REVIEW REAL PROPERTY MBE OUTLINES REAL PROPERTY 72 A buyer and a seller of interests in real property usually enter into a marketing contract reflecting the terms and conditions of their agreement. The contract may exist separate and distinct from any document regarding either: 1) the financing of the purchase of the real property interest, such as a mortgage; or 2) a conveyance thereof, such as a deed to a parcel of land. The parties may also refer to a marketing contract as a sell and buy contract. 2) INSTALLMENT CONTRACTS An installment contract, otherwise known as a contract of deed or a land contract, is an agreement pertaining to a purchase of real property and the financing of that purchase. Often, an installment contract requires that a buyer provide a specified down payment and make periodic payments for a particular duration. After the buyer fulfills those payment obligations, then the buyer will receive a deed for that interest in real property. C. Creation and Construction 1) STATUTE OF FRAUDS AND EXCEPTIONS a) General Rule A Statute of Frauds exists in all of the United States. A Statute of Frauds generally requires that, in order to be enforceable, an agreement conveying an interest in land must be in writing, be signed by the party to be charged, and contain all essential terms of the parties’ agreement. The Statute of Frauds also applies to an option to purchase real property and a promise to provide a right of first refusal. Courts will generally not enforce oral modifications of written agreements unless they constitute a waiver, as in terms of the time specified for performance, estoppel, or part performance. (1) Part Performance Exception Part performance of a contract is an exception to the Statute’s requirement of a writing. This exception applies when the parties only have a verbal agreement, and the buyer has fulfilled at least two or more of the following conditions: • A buyer makes substantial improvements to the property; • A buyer is in open and notorious possession of a seller’s property; and • A buyer pays the purchase price in part or in full. In other words, many courts may grant specific performance of a verbal contract to sell land if a buyer’s conduct was sufficient to constitute part performance. This exception is based on the theory that a person who has reasonably relied to his detriment on the assurances of the landowner should not suffer because the agreement was not recorded in writing. 2) ESSENTIAL TERMS a) Required Information (1) Sell and Buy Contract To be enforceable, a written sell and buy contract must also be sufficiently definite by containing, at a minimum, the first four items of the following information: • the parties’ names and addresses; • an accurate legal description of the land; • a signature by the party to be bound; Question Key 2,29,35,40,63,73,75,93,135,151 See also Contracts 31

Page 73

AMERIBAR BAR REVIEW REAL PROPERTY MBE OUTLINES REAL PROPERTY 73 • language indicating an intention to buy and to sell the land; • statement of the land’s selling price; • a recital of earnest money and consideration for the purchase; and • a covenant of marketable title. (a) Description of Land The land’s description must be adequate. It does not need to be as precise as the description that is contained in a deed. For example, a proper address for the property being bought and sold would generally suffice. The description must be sufficient to facilitate specific performance. (2) Deed of Conveyance A deed is an instrument that represents the title to real property. The Statute of Frauds requires that, other than for short-term leases, a conveyance of a real property interest must be in writing. Thus, a grantor must transfer a valid instrument of conveyance, or deed, to a grantee to make an effective sale of real property. To be valid and enforceable, a deed must include the following information: • the parties’ names and addresses; • an accurate legal description of the land; • language indicating an intention to transfer possession of the land; and • a signature by a grantor. If a deed does not satisfy the Statute of Frauds, then recording it will not render it a sufficient writing for purposes of complying with the Statute of Frauds. Destruction of a written deed does not affect the grantee’s title, except if the law requires recording of the deed to transfer title. A person may appoint an agent to sell his real estate. That agent, however, only possesses the authority to do acts specifically set forth in the authorization. Additionally, courts may not construe the authority to sell or to convey real estate as the authorization to execute a deed. Courts are divided as to whether agents who are authorized to sell or to sell and convey have the power to include covenants in the conveyance. (a) Attestation of Deed Attestation occurs when a lay witness to a grantor’s signing of a deed informally executes the deed to verify that the deed is authentic. Although attestation is not a common law requirement, some state statutes mandate it. (b) Acknowledgement of Deed Acknowledgement occurs when a public witness to a grantor’s signing of a deed formally executes the deed to verify that the deed is authentic. The witness may be a notary public or a judicial official. Some state statutes require acknowledgement in order for a deed to be self-authenticating as evidence or to be recorded. 3) TIME FOR PERFORMANCE The date and time for the parties to complete a real estate transaction is important for reasons of cost and convenience. a) Parties May Stipulate Time for Performance; Consequence of Nonperformance

Page 74

AMERIBAR BAR REVIEW REAL PROPERTY MBE OUTLINES REAL PROPERTY 74 The parties may stipulate certain dates and times for performance of certain steps that their contract requires, such as a final closing meeting. Failure to perform as stipulated may give rise to litigation. (1) Legal Standard The law generally provides that, unless the parties make a contrary agreement, time is of the essence. The law may require a person who breaches a contract’s requirement of time for performance to pay damages. (2) Equity Standard At equity, time is not of the essence unless the contract or surrounding circumstances make it so. When time is not of the essence, a first person’s tardy performance does not excuse a second person’s performance that occurs within a reasonable time. In that event, the first person may obtain specific performance of the contract. A party may not rescind or abandon the contract for such a nonmaterial breach. b) Determining if Time Is of Essence The fact that a sell and buy contract includes a closing date does not prove that the parties considered time to be of the essence. The following may establish the existence of that term: • a clause in the contract stating that time was of the essence; or • the parties’ conduct that indicated an implicit agreement that time was of the essence. c) Curing a Breach If time is not of the essence, a party who has failed to perform contractual duties may cure that breach by substantially performing those duties within a reasonable time. Conversely, a party who breaches a time is of the essence clause will not be entitled to specific performance. 4) REMEDIES FOR BREACH a) Seller’s Remedies (1) Damages A seller may seek the difference between the contract price and the market price at the time of closing while retaining the real property that is subject to the parties’ sell and buy contract. (2) Specific Performance A seller may obtain specific performance of a buy and sell contract that a buyer breaches because money damages are inadequate. Specific performance is available even if the seller’s title includes an immaterial defect. In that event, the buyer is entitled to an abatement in price for the value of any defect. b) Buyer’s Remedies (1) Damages Nearly half of the jurisdictions follow the American rule that is also used for personal property sales. If a seller intentionally breaches a sell and buy contract, a buyer may recover the difference between the real property’s market value and the contract price on the date that the closing is to occur.

Page 75

AMERIBAR BAR REVIEW REAL PROPERTY MBE OUTLINES REAL PROPERTY 75 The other half of jurisdictions follow the English Rule, which applies when a seller does not intentionally breach a contract. In that event, a buyer can only recover damages that will restore the buyer to its original position. (2) Specific Performance A buyer may obtain specific performance of a buy and sell contract that a seller breaches if money damages are inadequate. Specific performance is not available when a buyer seeks to have a seller cure a defect. In that event, the buyer may only obtain an abatement of price in the amount of the defect. (3) Rescission If a buyer elects not to seek either damages or specific performance, then the buyer may request rescission of a sell and buy contract and recover any down payment. D. Marketability of Title 1) MARKETABLE TITLE a) General Rules Marketable title means that the title is free from reasonable doubt about its validity, any threats of litigation regarding the title’s validity, or actual lawsuits relating to the title. A sell and buy contract usually includes an implied or express warranty of marketable title. If the contract does not address the issue of quality of title, a court will imply the requirement of marketable title. Unless the sell and buy contract allows for anything other than marketable title, a seller cannot require a buyer to accept less than marketable title. The buyer may waive the requirement that the seller provide marketable title because the right to marketable title benefits the buyer. Title does not need to be marketable until the time that the seller must deliver it to the buyer. This point in time could be, for example, the date set for closing the real estate transaction. As a practical matter, the parties' sell and buy agreement may provide that certain circumstances may render title to the real property unmarketable. b) When Title Is Not Marketable A title is unmarketable when a reasonable person would not purchase it. Title is not marketable if, among other things, any of the following conditions exist: • The real property violates a zoning statute, ordinance, or regulation; • The real property is subject to an adverse possession claim; • An encumbrance exists on the real property, such as a servitude or mortgage; or • The real property is subject to, or at issue in, ongoing litigation. c) When Title Is Not Rendered Unmarketable Conversely, unpaid liens or mortgages on the real property do not render title unmarketable if: • A seller gives a buyer reasonable assurances that any encumbrances on the title will be discharged when the seller tenders the title; and • The proceeds of a sale under a sell and buy contract are adequate to, and actually applied to, extinguish all encumbrances. Generally, easements do not render title unmarketable. (1) Failure to Deliver Marketable Title Question Key 26,28,42,117,122,127 See also Contracts 174

Page 76

AMERIBAR BAR REVIEW REAL PROPERTY MBE OUTLINES REAL PROPERTY 76 A seller who does not deliver marketable title at the mutually agreed upon time and/or place breaches a covenant in the sell and buy contract that marketable title will be tendered. This breach involves the failure of a constructive condition, which discharges a buyer from performing further and provides the following remedies: potential loss-of-bargain damages, rescission and restitution, specific performance and abatement, and reimbursement for out-of-pocket expenses. A maxim provides that equity will not compel a buyer to purchase a lawsuit. (a) Zoning and Marketable Title If a seller’s property is in violation of zoning regulations when the seller enters into a sell and buy contract with a buyer, then the title is unmarketable and the buyer may decline to perform the contract. Marketable title is not lost if the buyer plans to use the property in a manner that will violate existing zoning regulations. (2) Breach of Contract Although a seller may lack marketable title at the execution of a sell and buy contract, the seller will only breach the contract by failing to provide marketable title at the closing. When the seller fails to provide marketable title on the closing date, a buyer may: • elect to not perform under a sell and buy contract; • rescind the contract and recover its down payment; • commence a breach of contract action for damages; or • file an action seeking specific performance of the contract and request a purchase price abatement as relief for the title’s defect. (a) Curing Title Defects Examples of title defects and their cures include: • Applying the sale proceeds towards satisfaction of the mortgage debt may cure an outstanding mortgage. • Obtaining a judgment that quiets title in the adverse possessor may cure a claim to title by adverse possession. • Obtaining a termination of the easement may cure an easement that burdens the land. d) Insurable Title The parties’ contract to sell and buy real property may include a clause requiring insurable title, which involves a different standard than the marketable title requirement. To be enforceable, the clause language must be clear and unambiguous. The parties may also stipulate to their awareness of and acceptance of any encumbrances and other interests to which the real estate to be conveyed is subject. E. Equitable Conversion 1) EQUITABLE CONVERSION DOCTRINE (MAJORITY RULE) Equitable conversion applies when parties have entered into a specifically enforceable sell and buy contract. The equitable conversion doctrine considers a buyer as the owner of real property during a contract’s executory period. Thus, the risk of loss falls upon a buyer. This doctrine requires the buyer to fulfill a contract and pay the purchase price despite a major loss in the property’s value due to an unanticipated event that the buyer did not cause. A minority of jurisdictions rejects the doctrine and place the risk of loss upon a seller. Question Key 42,98,99 See also Contracts 133

Page 77

AMERIBAR BAR REVIEW REAL PROPERTY MBE OUTLINES REAL PROPERTY 77 a) Effect of the Equitable Conversion Doctrine Under the doctrine, a sell and buy contract with a seller is specifically enforceable in equity. The doctrine furthers the maxim that equity considers that as having been done that which should have been done. The doctrine provides that the execution of the contract yields the following results: • The seller retains a legal interest in the real property. • The seller receives a right in personal property to obtain the purchase price. • The buyer obtains an equitable interest in the real property. • The buyer receives a right in realty to a conveyance of that property. (1) Death of a Party Equitable conversion applies to the distribution of property under both testate and intestate succession. Courts consider a seller’s interest personal property secured by a lien. Courts consider a buyer’s interest real property. (a) Exception - Option to Buy Equitable conversion does not apply to an option that a buyer fails to exercise before a seller’s death. Only an optionee (i.e., buyer) may specifically enforce an option. (2) Right of Possession A buyer who obtains equitable title does not gain the right of possession that is derived from legal title. If, however, the buyer is purchasing real property under a land contract (i.e., installment contract), the buyer may occupy the property. In that event, the buyer may not cause waste harming the seller’s security interest. 2) RISK OF LOSS a) Intervening Events Events may occur between the formation and performance of a sell and buy contract that affect one party or reduce the real property’s value. The following events may cause a loss to the property: • natural conditions that may cause physical damage to the property (e.g., weather-related damage to structures or crops, fires, floods, and storms); • changes in the land’s legal status (e.g., a taking through eminent domain and modified zoning provisions or housing codes); • an adverse judgment that involves judgment liens upon the land. b) In Relation to Equitable Conversion Doctrine Jurisdictions are split regarding risk of loss rules. Many jurisdictions follow the doctrine of equitable conversion, whereby the risk of loss transfers to the buyer at the time the parties enter into the contract. Thus, if a house on the property burns down after the parties sign the contract, but before the grantor conveys the deed, the buyer will still have to pay the contract price for the land. c) The Uniform Vendor and Purchaser Risk of Loss Act The Uniform Vendor and Purchaser Risk of Loss Act (“Act”) allocates the risk of loss to a seller until the earlier of a transfer of title to the real estate to the buyer or when the buyer takes possession of the real estate. For example, if a fire damaged a seller’s house before the buyer received title or possession of it, the buyer may obtain a reduction in

Page 78

AMERIBAR BAR REVIEW REAL PROPERTY MBE OUTLINES REAL PROPERTY 78 the purchase price if the sell and buy contract is not avoidable. The Act is the minority rule and only applies in those states that have adopted it to some extent. F. Options and Rights of First Refusal 1) OPTION CONTRACTS a) Right to Purchase An option contract must be in writing to be enforceable under the Statute of Frauds. A seller (i.e., optioner) provides a buyer (i.e., optionee) with a right to purchase an interest in real property at a future time under certain conditions. An optionee may seek specific enforcement of the option contract as an equitable interest in the real property. An option is subject to the Rule Against Perpetuities. b) Preemptive Option A prospective seller (i.e., optioner) may provide a prospective buyer (i.e., optionee) with a preemptive option (i.e., a conditional option) that constitutes a right of first refusal to purchase the seller’s interest in real property if and when the seller decides to sell that interest. A preemptive option is subject to the Rule Against Perpetuities. A court may strike down as void ab initio a preemptive option that violates the Rule Against Perpetuities. 2) RIGHT OF FIRST REFUSAL a) Seller Must Give Other Party Chance to Match Offer to Purchase A contract may provide a right of first refusal. A right of first refusal requires a seller to give the contract's other party a chance to match a third person's offer to purchase the seller's specified property. Black's Law Dictionary, 2d ed. Suppose, for example, that a landowner rents a house to an occupant pursuant to their rental agreement. The agreement provides that, in the event that the landowner decides to sell the house, the occupant has a right to buy the house for the same price that a prospective buyer offers to pay the owner for the house. In response to the landowner's notice that the house is "for sale," an investor offers to purchase the house for $300,000. Here, the landowner must notify the occupant of his notice to sell, the investor's offer, and allow the occupant to buy the house for $300,000 within a reasonable time period. If the occupant exercises the right of first refusal within the time period, then instead of the investor he would be entitled to buy the house. G. Fitness and Suitability 1) GENERAL CONSIDERATIONS a) Disclosure of Defects Usually a deed does not include a covenant that warrants the fitness and suitability of a premises that is subject to a conveyance of an interest in real property. The common law doctrine of caveat emptor does not require a seller of such an interest to disclose any defects of the interest to a buyer. Modern case law may invoke, but does not necessarily follow, this doctrine in instances when premises contain defects. Additionally, some state statutes require disclosures of certain defects of a premises. (1) Seller’s Disclosure Obligations The following rules apply to a seller’s nondisclosure of defects of a premises: • A seller will be liable for fraud for making affirmative misrepresentations about a property that are relied upon by a buyer.

Page 79

AMERIBAR BAR REVIEW REAL PROPERTY MBE OUTLINES REAL PROPERTY 79 • A seller who has special relationship of trust with a buyer, such as a fiduciary, will be subject to a duty of full disclosure and candor about any defects on the premises. • A seller will be civilly liable for failing to disclose material defects that cannot be easily discovered by an inspection of the property. H. Merger 1) MERGER DOCTRINE (a) Common Law The doctrine of merger provides that if a buyer accepts a deed from a seller, the buyer may only bring claims that arise from title defect issues that may be asserted pursuant to the deed’s title covenants. The merger results in the deed’s covenants of title subsuming any covenants of title in the sell and buy contract. (b) Modern Law The modern application of that rule is to only merge those covenants of title in a sell and buy contract that are the same as the covenants in a deed of conveyance. Thus, any covenants in the contract that are different from those in the deed will not be merged. Although this trend is the modern trend, the common law approach is the majority approach. Later, this outline details covenants of title. I. Interests Before Conveyance 1) EARNEST MONEY DEPOSIT a) Definition The parties’ sell and buy contract may require a buyer to pay funds as a deposit to a seller or a third party who normally will apply them towards the purchase price of the interest in real property. The parties may refer to these funds as earnest money because they indicate the buyer’s serious interest in completing the parties’ proposed transaction. b) Payment Earnest money is similar to, and may be part of, a buyer’s down payment of the total purchase price. But the buyer generally pays, and the seller receives, the earnest money well in advance of closing on a real estate transaction because it serves to secure the buyer’s right of priority to the property over other potential buyers. c) Liquidated Damages A sell and buy contract may allow a seller to retain a down payment of earnest money if a buyer fails to complete performance of the contract. Such a provision is unenforceable if the amount retained is large enough to constitute a penalty. Such a provision is enforceable if the amount of down payment is reasonably related to the seller’s actual damages. J. Relationships After Conveyance 1) CONDITION OF PREMISES a) Seller’s Failure to Disclose Defects (1) Seller’s Decreased Liability Question Key 34,35,158 Question Key 14,49,51 See also Contracts 191

Page 80

AMERIBAR BAR REVIEW REAL PROPERTY MBE OUTLINES REAL PROPERTY 80 The following circumstances may decrease the prospects that courts will impose liability for a seller’s nondisclosure of a premises’ defects: • A buyer learned of defects of premises before the purchase from an inspection or other sources; or • Information about the defects of premises was available to a buyer. (2) Seller’s Increased Liability The following considerations may increase the likelihood of imposing liability for a seller’s nondisclosure of a premises’ defects: • A novice buyer is acquiring a residential property; • A property defect presents a safety or health risk; • A seller reassured a buyer about the property’s quality; • A buyer neither received nor depended upon a professional inspection of the property; or • A seller possesses reason to expect that the seller’s nondisclosure will impact subsequent buyers despite a lack of privity between them and the seller. b) State Disclosure Statutes Several state statutes deny a remedy to a buyer of a house in which an AIDS patient died or a crime occurred. Many state statutes require a house seller to make a written disclosure of the seller’s knowledge of whether items on the property are defective or not. c) Contract Disclaimers Although a seller could contractually disclaim a duty to disclose defects, courts do not usually completely enforce general disclaimers or “as is” clauses. Such clauses will prevent lawsuits for passive nondisclosure of defects but will not prevent suits for concealment of, or misrepresentations about, defects. d) Contractor Liability The common law doctrine of caveat emptor usually does not protect contractors who build new housing in their sales of that housing. Most courts consider the contractor legally responsible for the property’s quality on three theories that are similar to the bases of manufacturer tort liability: negligence, implied warranty, and strict liability. IV. MORTGAGES / SECURITY DEVICES A. Types of Security Devices 1) MORTGAGES a) In General (1) Definition A mortgage is an interest in land created by a written instrument providing security for payment of a financial obligation or performance of a duty. A mortgage typically is an agreement between a mortgagor (i.e., borrower) and a mortgagee (i.e., lender). (2) Parties’ Roles and Obligations

Page 81

AMERIBAR BAR REVIEW REAL PROPERTY MBE OUTLINES REAL PROPERTY 81 A mortgagor borrows funds from a mortgagee to obtain an ownership interest in real property. As security for the loan, the mortgagee holds a mortgage (a lien) on the property. The mortgagee is the lender of the funds, issues the mortgage, and holds a security interest in the real property for payment of the funds. A promissory note usually documents the mortgagor’s financial obligation of payment. (3) Statute of Frauds In order to satisfy the Statute of Frauds, a mortgage must be in writing. At a minimum, a written mortgage instrument needs to contain certain essential information including: • the parties’ names and addresses; • an accurate legal description of the land; • the amount of funds that are borrowed; and • a mortgagor’s promise to pay. b) Purchase-Money Mortgage A purchase-money mortgage is a mortgage under which the seller of the real property loans money directly to a buyer of real property. Typically, the seller conveys title to the real property in exchange for a down payment, a mortgage on the property, and a promissory note or loan for paying the remaining balance due on the purchase price. c) Future-Advance Mortgage A typical construction loan provides that the lender will advance funds to the borrower over a fixed time period. The lender secures a mortgage on the property for the entire amount of the money it has agreed to lend, including future advances. Such “future-advance” mortgages may provide for obligatory advances, or they may provide for advances that are optional. Whether the future-advance mortgage payments are obligatory or optional is critical to the rights of a junior lender. (1) Junior Lender's Rights If payments under a future-advance mortgage are obligatory, then the junior lender’s lien is junior both to amounts loaned to the debtor before the junior lien was recorded and to amounts loaned after the junior lien is recorded. If the payments under a future-advance mortgage are optional, the junior lender has a priority over amounts transferred to the debtor by the senior lender after the junior lender transfers funds to the debtor and records its mortgage. (2) Rationale Based on Whether Obligatory or Optional Loan The rationale for this rule is that in the case of an obligatory loan, the junior lender can ascertain from public land records the maximum amount of the senior lender’s claim before loaning money to the creditor, while with an optional loan, the junior lender cannot know whether subsequent advances will be made and the senior lender can protect itself against junior lenders by searching the land records before making additional advances. d) Equitable Mortgage An equitable mortgage is a “mortgage” that lacks one or more required formalities (e.g., Statute of Frauds). Under the equitable mortgage doctrine, courts may enforce such a defective mortgage as an equitable mortgage. e) Deeds of Trust A deed of trust requires three parties. The parties include the trustor (e.g., borrower), the lender, and a neutral third party. The third party is the trustee, who essentially holds temporary title until the lien is paid. Payment of the debt

Page 82

AMERIBAR BAR REVIEW REAL PROPERTY MBE OUTLINES REAL PROPERTY 82 in full cancels the deed of trust. Until then, the trustee has the power to foreclose if the debt is not paid without having to resort to the court system. This power makes it easier and quicker than foreclosing on a mortgage. 2) INSTALLMENT LAND CONTRACTS a) Definition A land contract provides for a future purchase of real property through the financing of that purchase. A land contract allows a buyer to take possession of the property while making regular payments towards the property’s purchase price. The seller retains a deed to the property that she neither conveys nor delivers until after the buyer completely satisfies the payment obligation. b) Seller’s Rights Under a land contract, a seller possesses two rights of enforcement in the event of nonpayment: 1) to recover the unpaid balance; and 2) oust the buyer from possession of the land. In either event, the seller neither loses possession of the deed that serves as the seller’s security device nor is obligated to refund any of the buyer’s installment payments. c) Default Procedures The common law and most states require a seller to provide a buyer with fair notice of a default on a land contract prior to bringing a forfeiture action. That notice should describe the default, what steps would cure it, and how much time is allowed for the cure. The notice needs to declare that a forfeiture will occur if the default is not cured. State law determines whether a buyer possesses a right to redeem the property or to reinstate the land contract. d) Statute of Frauds A land contract is subject to the Statute of Frauds. Thus, to be enforceable it must, at a minimum, be in writing, be signed by the party to be charged, and contain all of the contract’s essential terms. 3) ABSOLUTE DEEDS AS SECURITY a) Definition Two primary transactions for the conveyance of an interest in real property are land contracts or sell and buy contracts that involve a mortgage agreement. The sell and buy contract may, instead of including a mortgage agreement, provide an absolute deed as security. The parties may even possess just an oral understanding that once the debt is paid, the creditor will convey the deed to the debtor. In other words, the creditor holds the deed in absolute form. b) Admissible Evidence Neither the parol evidence rule nor the Statute of Frauds precludes the presentation of evidence that parties intended such an absolute deed to be a mortgage. In that event, the court may consider the deed as their mortgage if the evidence indicates that the deed was intended as security for the debt. B. Security Relationships 1) NECESSITY AND NATURE OF OBLIGATION a) General Considerations Question Key 55

Page 83

AMERIBAR BAR REVIEW REAL PROPERTY MBE OUTLINES REAL PROPERTY 83 A mortgage represents the security interest, and a promissory note represents the debt that the interest secures. A mortgage does not need to be recorded to create a binding legal agreement between the parties. (1) Recorded Mortgages Recording acts usually classify a mortgage as an encumbrance on title to real property, which creates a security interest that a creditor may enforce by selling that property to recover any unpaid balance under the secured promissory note. (2) Unrecorded Mortgages If a mortgagee has not recorded a mortgage, a mortgagor could convey the secured real property to a bona fide (good-faith) purchaser who would not be subject to the mortgage. 2) MORTGAGE THEORIES: TITLE, LIEN, AND INTERMEDIATE a) Title Theory (Minority Rule) A minority of states follow the common law title theory. Under that theory, a mortgagor owns the real property, although the mortgagee possesses legal title to the property. The mortgage serves as a conveyance of legal title to the property to a mortgagee. Upon payment of the mortgage debt, legal title is subject to defeasance. The title provides a mortgagee with a security interest until legal title is returned to the mortgagor when the mortgage is paid in full. (1) Severance of Joint Tenancy A mortgaging of a joint tenancy interest in real property will automatically sever the title of a joint tenancy in a title theory jurisdiction. For example, an equity loan creates a type of mortgage lien that will result in severance of a joint tenancy before a loan default and a foreclosure sale occurs. b) Lien Theory (Majority Rule) The majority of the states follow a lien theory, which provides that a mortgagor owns the real property. This property is subject to a lien that a mortgage imposes, irrespective of a mortgage’s operative language. The mortgagor retains the legal title until a foreclosure occurs. (1) Joint Tenancy Not Severed Mortgaging a joint tenancy interest in real property does not automatically sever the title of a joint tenancy in a lien theory jurisdiction. For example, an equity loan creates a type of mortgage lien that will not result in a severance of a joint tenancy unless a loan default and foreclosure sale occurs. c) Intermediate Theory Under the intermediate theory, the lien theory applies prior to a default on a mortgage. The title theory applies after a default on the mortgage. 3) RIGHTS AND DUTIES PRIOR TO FORECLOSURE a) Rights Prior to Foreclosure After a mortgagor’s default and prior to the foreclosure of a mortgage by a mortgagee, a mortgagor possesses a right to pay the outstanding deficiency balance and get current on the mortgage obligation. If the mortgagor does that within a notice period or prior to a deadline, then the mortgagee would not need to foreclose on the mortgage. Question Key 161

Page 84

AMERIBAR BAR REVIEW REAL PROPERTY MBE OUTLINES REAL PROPERTY 84 b) Duties Prior to Foreclosure Before a mortgagee commences foreclosure of a mortgage, the mortgagee must provide the mortgagor with adequate written notice of the deficiency balance that is due upon default and a reasonable amount of time to pay that amount. c) Mortgagee in Possession A mortgagee may obtain possession of a parcel of real estate that secures the mortgage during its existence. The mortgagee may do so if a mortgagor is in default of a mortgage or abandons the mortgaged parcel. The mortgagee need not occupy a parcel in order to be in possession of it. The mortgagee needs to exercise some “dominion and control” over the parcel to be in possession of it. (1) Title Theory (Minority Rule) In title theory jurisdictions, a mortgagee has a right to possession of the mortgaged parcel prior to or following a mortgagor’s default unless a statute or their agreement provides otherwise. (2) Lien Theory (Majority Rule) In lien theory jurisdictions, a mortgagee does not have a right to possession of the mortgaged parcel prior to or following a mortgagor’s default until after either a foreclosure has occurred or the duration for statutory redemption has expired, unless a statute or their agreement provides otherwise. (3) Intermediate Theory In intermediate theory jurisdictions, a mortgagee is entitled to possession of the mortgaged parcel prior to or following a mortgagor’s default unless a statute or their agreement provides otherwise. (4) Mortgagee in Possession’s Liability A mortgagee in possession is personally liable in tort for injuries attributable to its use of the property or lack of performing legally required duties as a landowner. 4) RIGHT TO REDEEM AND CLOGGING THE EQUITY OF REDEMPTION a) Right to Redeem Redemption is the right of a mortgagor to purchase the real property by fully paying the mortgage debt (and any applicable late fees or collection costs) within a certain time before the date of foreclosure. The effect of foreclosure is to prevent a mortgagor from redeeming the default and recovering the real property. (1) Common Law The common law generally provides an equitable right of redemption within a reasonable time before a foreclosure occurs. (2) Statutes States may provide a statutory right of redemption within a specified interval after a foreclosure occurs. A mortgagee cannot include language in the mortgage that waives a mortgagor’s right of redemption. (3) Title Theory States Question Key 152

Page 85

AMERIBAR BAR REVIEW REAL PROPERTY MBE OUTLINES REAL PROPERTY 85 In a title theory jurisdiction, a mortgagor possesses an equity of redemption (after defaulting on an obligation) in order to make mortgage payments to a mortgagee. Equitable principles will provide the mortgagor relief from a conveyance of the mortgaged real property that the law renders absolute in the mortgagee. The mortgagor may possess an equity interest in that property in the amount that the mortgagor has paid to the mortgagee in excess of the original mortgage debt. (4) Junior Creditors’ Interests A mortgagor's (borrower’s) exercise of the right to redeem a default will not affect the mortgagor's obligation to junior interests or the rights of those junior creditors. However, all junior creditors’ interests will be extinguished at a foreclosure sale. Before a foreclosure sale, a redeeming party may pay off the mortgage that is being foreclosed. b) Clogging Equity of Redemption (1) Definition A clog is any impediment a mortgagee imposes upon a mortgagor’s ability to redeem a mortgage. A doctrine that prevents a mortgagee from making a mortgage irredeemable prohibits clogging the equity of redemption. It is the opposite of the equity of redemption doctrine, which provides for the redemption of a mortgage. Clogging the equity of redemption refers to a method that a mortgagee would employ to obtain fee simple title to a mortgaged interest in real estate upon a mortgagor’s default. Mortgagees may, however, use the following permissible means to limit a mortgagor’s late exercise of a power of redemption: 1) a deed absolute; or 2) a land contract. (2) Prohibited Provisions (a) General Equity prohibits the following types of provisions that would “clog” the equity of redemption: • a provision that restricts redemption to a certain time period; • a provision restricting the power of redemption to only a mortgagor and/or the mortgagor’s male heirs; and • a provision obligating a mortgagor to provide a mortgagee with a quit claim deed upon foreclosure. (b) Specific Performance of Mortgagee Option A mortgagee is not necessarily entitled to specific performance of the mortgagee’s option to purchase the mortgaged real property that is contained in an agreement with a mortgagor. Courts may not enforce this option because it allows a mortgagee to obtain the real property without following the foreclosure procedures. (3) Permitted Provisions (a) Increased Interest A stipulated provision for an increased interest rate following a default would not be an impermissible clog, but may be invalid if it provides for a usurious interest rate. (b) Due-on-Sale Clause A due-on-sale clause is not a clog, even if it is contained in a purchase money mortgage. A due-on-sale clause is a provision in a mortgage or deed of trust that permits the lender to demand immediate payment of the full balance of the mortgage if the mortgagor (i.e., borrower) sells the home. c) Deeds of Trust

Page 86

AMERIBAR BAR REVIEW REAL PROPERTY MBE OUTLINES REAL PROPERTY 86 The deed of trust provides a trustor with an equity of redemption equal to that which a mortgagor possesses under a mortgage. It enables a lender to both exercise the power of sale to foreclose and to bid at sales that the lender initiates. C. Transfers 1) BY MORTGAGOR a) Assumption and Transfer Subject To A mortgagor may make these types of transfer of property: • a transfer “subject to” a mortgage; • a transfer “assuming” a mortgage; and • a transfer in exchange for the land’s full value. (1) Distinguishing "Subject To" and "Assuming" (a) Transfers “Subject to” a Mortgage (i) Majority Rule If a transferor conveys real property to a transferee that is “subject to” a mortgage, the transferee does not become personally liable for the mortgage debt. The transferee may, however, lose its right to that property by failing to pay the debt when the mortgage is foreclosed. (ii) Minority Rule A few states, by statute or in case law, provide that a conveyance that is subject to a mortgage imposes a direct obligation upon a transferee. (b) Transfers “Assuming” a Mortgage (i) General Rule If a buyer “assumes” a mortgage when purchasing real property from a seller, the buyer becomes personally liable for a mortgage debt. Consequently, the buyer becomes primarily liable for a deficiency judgment if a foreclosure sale does not result in fully repaying the debt. In that event, the seller becomes secondarily liable unless the seller has obtained a release from the mortgagee. (ii) Statute of Frauds The predominant common law rule is that the Statute of Frauds does not preclude the creation of an oral assumption of mortgage agreement. Statutes in a minority of states, however, require that such an agreement be in writing in order for it to be enforced. (iii) Novation When a promisor agrees to assume a mortgage, a mortgagor will still be liable for its mortgage obligations to a mortgagee unless a novation expressly substitutes the promisor for the mortgagor and releases the mortgagor from any further liability. The mortgagee may recover on the mortgage from the promisor as a third-party beneficiary of the assumption. The promisor and the mortgagor will be jointly and severally liable to the mortgagee, although they will not be equally subject to the mortgage obligation among each other. Their contractual relationship will determine Question Key 16,54,138,147

Page 87

AMERIBAR BAR REVIEW REAL PROPERTY MBE OUTLINES REAL PROPERTY 87 their respective liability. (c) Assignment of a Mortgage An assignor may transfer a mortgage or a promissory note to an assignee. If the assignor transfers the mortgage or the note to different assignees, then the mortgage follows the note. The note for the debt is the primary obligation, and the mortgage is a secondary provision of security for the debt. b) Rights and Obligations (1) Transferor’s (Borrower) Rights A transferor (i.e., mortgagor or initial borrower) may usually transfer a mortgage interest to a transferee without a mortgagee’s consent, unless the mortgage document requires consent. The mortgagor’s right to transfer is derived from its ownership interest in the real property. (2) Transferor’s Obligations (a) Non-assuming Transferee If a transferee does not assume a transferred mortgage, then a transferor cannot make a personal claim to recover the mortgage debt from the transferee. Thus, the transferor remains entirely liable (without recourse to the transferee) to the mortgagee for the mortgage debt. (b) Assuming Transferee If a transferee assumes a transferred mortgage, then a transferor can make a personal claim to recover the mortgage debt from the transferee. Therefore, although the transferor remains liable to the mortgagee for the mortgage debt, the transferor may seek a recovery for that debt from the transferee. (c) Personal Liability The personal liability of a mortgagor to a mortgagee continues after a transfer of the mortgage to a third party unless: • Those parties enter into a legally sufficient agreement to the contrary; • The mortgagor fully pays off the mortgage balance; or • Certain other principles of suretyship or contract apply. c) Application of Subrogation and Suretyship Principles (1) Subrogation Principles (a) General Considerations Subrogation refers to the substitution of one person in the place of another person with reference to a specific claim, demand, or right. The substituting party generally pays the debt or claim of another and, as a result, succeeds to the rights of the other in relation to the debt or claim and obtains the corresponding rights, remedies, or securities. Generally, three types of circumstances give rise to subrogation issues in the mortgage context: • A subrogee voluntarily extinguishes the mortgage of another;

Page 88

AMERIBAR BAR REVIEW REAL PROPERTY MBE OUTLINES REAL PROPERTY 88 • A subrogee pays a debt for which the subrogee is not responsible in order to preserve some interest in the property; and • A surety on a mortgage debt pays this debt. (b) Subrogation Rights A party’s payment of the obligation that a mortgage secures will result in subrogation of that party to the mortgagee’s rights in the mortgage. For example, suppose Sal, a surety (guarantor) of Mac’s mortgage, pays Mac’s liability upon Mac’s default of the mortgage. Sal would step into the shoes of the mortgagee (i.e., lender) and could exercise the right of foreclosure against Mac. The subrogee must completely pay the debt to obtain the right of subrogation. (2) Suretyship Principles (a) Transfers “Subject to” a Mortgage A transferor remains liable on the mortgage when a grantee (i.e., transferee) takes property “subject” to a mortgage. The grantee is not liable on the mortgage, but the mortgagee can still foreclose on the land to secure payment of the debt. The transferor has no recourse against the grantee if the mortgage is foreclosed. (b) Transfers “Assuming” a Mortgage A transferor becomes a surety (i.e., guarantor) to a mortgagee when a transferee “assumes” a mortgage. A transferee and the land are the primary obligors on the mortgage. Additionally, the transferee may indemnify the transferor if the transferor is required to satisfy the mortgage. d) Restrictions on Transfer (1) Due-on-Sale Clause Some mortgages include clauses providing that a mortgage is “due on sale” and “due on encumbrance” for the purpose of preventing a transfer of mortgages. Those clauses provide for acceleration of the mortgage by making the principal and interest immediately due and payable. Accordingly, a mortgage may include a clause requiring that, upon a sale of the mortgaged real estate, a mortgagor must fully pay the amount that remains due on the mortgage. (a) Example A due-on-sale clause in a mortgage may provide that upon the happening of certain events, the mortgagee may accelerate the mortgage debt so that it must be paid earlier than otherwise provided by the mortgage. For example, the clause can state that the mortgagee may accelerate the debt and foreclose on the mortgage in the event of a transfer of the real property subject to the mortgage by the mortgagor to a third party without the mortgagee's prior consent. In that example, the mortgagor's sale of the real property to a third party by means of an installment land contract before discharge of the mortgage could constitute a transfer that may trigger the clause and consequent foreclosure. (b) Legal Restraint on Alienation If a mortgagor grants a mortgage to nationally chartered bank (i.e., a mortgagee), a due-on-sale clause contained in the mortgage is valid and a legal restraint on alienation. The Garn-St. Germain Depository Institutions Act preempts any state law that provides otherwise. e) Methods of Transfer

Page 89

AMERIBAR BAR REVIEW REAL PROPERTY MBE OUTLINES REAL PROPERTY 89 “Voluntary inter vivos transfer of the mortgagor’s interest is ordinarily effected by a deed of conveyance in the usual form. At the mortgagor’s death, transfer of his interest may be effected either by a duly executed will or by the statute of descent if [the mortgagor] dies intestate. Successive transfers of the mortgaged land may take place in any of these ways or by various modes of involuntary transfer, such as foreclosure, execution, or bankruptcy sale.” Cunningham and Tischler, Transfer of the Real Estate Mortgagor’s Interest, 27 Rutgers L.Rev. 24, 25 (1973). 2) BY MORTGAGEE a) Common Law (1) Mortgagee A mortgagee may transfer its mortgage interest to a third party. (2) Means of Transfer The type of mortgage and obligation involved will determine the ways in which the mortgagee may transfer an obligation. The UCC describes the means of transfer of an obligation to pay funds. b) UCC Article 3 (1) Negotiable Instruments Subject to certain changes, most states have enacted statutes based on Article 3 of the UCC as the law governing negotiable instruments. The UCC defines a negotiable instrument as a writing that promises or mandates the payment of a specified sum of money. As its title indicates, parties may transfer such a writing between themselves by means of negotiation. (a) Promissory Notes A promissory note is a negotiable instrument that a party may use to obtain payment in property law transactions. It secures mortgages and similar loans. A note must satisfy the requirements of Article 3 to be considered negotiable. (b) Transfer by Negotiation A transfer may be accomplished by negotiation if the payee-mortgagee endorses a negotiable promissory note and the payee-mortgagee physically transfers the note into the new holder’s possession. When a transferee is either ineligible for holder in due course status or would forego that status, then the parties may use methods to assign a non-negotiable instrument instead. (c) Holder in Due Course A holder in due course takes possession of a negotiable instrument free from certain defenses such as payment, failure of consideration, and fraud. A holder in due course is someone that receives a note: • in good faith; • for value; and • without notice that it has been dishonored, is overdue, or is subject to any claim or defense. (i) Defenses

Page 90

AMERIBAR BAR REVIEW REAL PROPERTY MBE OUTLINES REAL PROPERTY 90 A holder in due course possesses a note free from many of the defenses that a note’s maker might raise against foreclosure or collection. The following “real” defenses are valid even against a holder in due course: • misrepresentation that induces a party to execute an agreement; • duress regarding, or illegality of, a transaction; • the infancy of a party to an agreement; or • discharge in an insolvency proceeding. c) Derivative Title Rule The derivative title rule prevents a property owner from transferring rights in a piece of property that exceed the owner’s rights. (1) Exception for Negotiable Instruments The derivative title rule does not apply to negotiable instruments. A bona fide purchaser who lacks awareness of a defect in title to an instrument, or any claims against it, takes that title free of any defects or claims. With respect to the inapplicability of the derivative title rule, Article 3 furnishes warranties for transactions that involve negotiable instruments. d) Non-negotiable Instruments A party may only transfer a non-negotiable instrument by assignment, not by negotiation. The ways to assign a non-negotiable instrument are: • An assignor’s verbal communication that an assignment is being made; • A written instrument of assignment that a mortgagee-payee executes to an assignee; • The mortgagee-payee indorses an original note; or • A transfer of possession of a note, which is not a necessary part of the above methods. e) Payment to Assignor A mortgagee (i.e., lender) may assign an interest in a mortgage to an assignee. The mortgage will then be payable to the assignee. A question that may arise is what happens if the mortgagor pays the mortgagee as assignor instead of the assignee. An assignee of a mortgage may seek to recover payment of the mortgage from a mortgagor who previously paid for the balance due on the mortgage to a mortgagee. The mortgagor may potentially raise the defense of prior payment. (1) Notice Received If a mortgagor receives notice of the assignment, then any subsequent payment that the mortgagor makes to the mortgagee as assignor will not serve as a defense to the assignee’s action on the obligation or for a foreclosure. (2) No Notice If a mortgagor (i.e., borrower) does not receive notice of the assignment, then any subsequent payment that the mortgagor makes to the original mortgagee as assignor will serve as a defense to the assignee’s action on the obligation or for a foreclosure. D. Discharge of the Mortgage

Page 91

AMERIBAR BAR REVIEW REAL PROPERTY MBE OUTLINES REAL PROPERTY 91 1) PAYMENT a) Discharge Occurs upon Full Payment of Entire Mortgage In both title theory and lien theory states, a discharge occurs when the entire mortgage debt is paid in full, such that a mortgagor and his transferee have no further payment obligation to a mortgagee. b) Prepayment The majority rule provides that a mortgagor may not prepay a mortgage debt prior to its due date unless the parties have expressly stipulated to the contrary. A provision allowing prepayment may be enforceable if the parties were knowledgeable and entered into their agreement after dealing at arm’s length. c) Late Payment (1) Title Theory States A mortgagor’s late payment does not discharge a mortgage lien if a mortgagee rejected it. (2) Lien Theory States The majority rule is that a mortgagor’s late payment discharges a mortgage lien if a mortgagee rejected it. d) Merger If a fee title and a mortgagee’s interest both coexist with respect to the same individual or entity, a lesser estate (i.e., a mortgage), merges into a greater estate (i.e., a fee interest). Consequently, the merger extinguishes the mortgage. This doctrine allows a mortgagor to argue that a mortgage does not exist and provides a defense in a foreclosure action. 2) DEED IN LIEU OF FORECLOSURE Certain jurisdictions allow a mortgagor who otherwise would be subject to foreclosure to provide a mortgagee (i.e., lender), with the deed as additional security for the unpaid mortgage debt, in lieu of foreclosure a) Definition A deed in lieu of foreclosure is an instrument in which a mortgagor (i.e., borrower) conveys all interest in real property to the mortgagee (i.e., lender) to satisfy a loan that is in default and avoid foreclosure proceedings. b) General Considerations A deed in lieu of foreclosure allows a mortgagor to sell the equity of redemption to a mortgagee. This type of deed refers to the use of a written agreement created subsequent to the parties’ original transaction. The deed must be based on new consideration. The deed may not be based on any oppression or fraud by a mortgagee. E. Foreclosure 1) GENERAL CONSIDERATIONS A mortgagor’s failure to make payments will result in a default of the mortgagor’s obligations to a mortgagee. A mortgagor’s failure to cure a default may cause a mortgagee to seek foreclosure of the mortgage. Foreclosure is a way Question Key 162,170

Page 92

AMERIBAR BAR REVIEW REAL PROPERTY MBE OUTLINES REAL PROPERTY 92 for a mortgagee to legally enforce a mortgage by applying the proceeds from a sale of the security, the mortgaged premises, towards satisfying the mortgagor’s debt obligation. a) Mortgagor’s Responses A mortgagor may respond to a foreclosure either by voluntarily redeeming (i.e., paying-off) the mortgage obligation or by disputing the foreclosure. 2) TYPES a) Judicial Foreclosure A judicial foreclosure is a judgment rendered by a court in favor of foreclosure of a mortgage or deed of trust. The judicial foreclosure judgment will order that the real property that secured the debt be sold under foreclosure proceedings to pay the debt. Thus, if a mortgagee prevails in a foreclosure action to recover the balance due on a mortgage, then a trial court will direct a sale of the mortgaged interest in real property to satisfy the amount of a judgment of foreclosure. (1) Liability for Deficiency If the sale proceeds are insufficient to satisfy the judgment, then the court will calculate the amount of deficiency for which it may award a decree. A mortgagor may be personally liable for the amount of that deficiency. b) Power of Sale (i.e., Non-Judicial Foreclosure) (1) Definition A foreclosure under power of sale only occurs if the mortgage’s terms provide for this private remedy that does not involve the courts. (2) Statutory Protections State statutes may provide certain procedural safeguards for a mortgagor that is subject to a power of sale. Those legal provisions apply in addition to any process or procedure that the parties stipulated in the mortgage. State statutes may also allow a mortgagor to bring a legal challenge to the results of a sale under a power of sale if: • the sale price “shocks the conscience” because of gross inadequacy; or • the sale involves unconscionable or fraudulent conduct. (a) Trust Deed Foreclosure Some states allow a trustee to foreclose on a trust deed without judicial proceedings if the trust deed contains a power of sale clause. c) Strict Foreclosure The most commonly used types of foreclosures are the judicial foreclosure and the power of sale. Another less prevalent type of foreclosure is the strict foreclosure. Under a strict foreclosure, the mortgagor is given a certain amount of time to pay the debt. If he fails to do so, under a strict foreclosure, the mortgagee (i.e., lender) takes title to the property directly instead of forcing a sale of the property.

Page 93

AMERIBAR BAR REVIEW REAL PROPERTY MBE OUTLINES REAL PROPERTY 93 Most states do not permit strict foreclosures. 3) ACCELERATION a) Acceleration Clause As a general matter, an acceleration clause (the "Clause") enables its holder to declare the outstanding balance due and payable immediately upon a default in fulfilling the obligation. An acceleration clause may appear in a mortgage, as well as a deed of trust or installment (i.e., promissory) note. An acceleration clause grants the creditor (e.g., mortgagee) the right to demand payment in full upon the occurrence of a specified event. (1) Acceleration of Mortgage Payments upon Specified Events The parties to a mortgage may elect to have all remaining mortgage payments accelerated as a result of a Clause in the mortgage. Essentially, the Clause is a mortgage term requiring the mortgagor to immediately pay the unpaid mortgage amount due, upon existence of the events specified in the Clause. Most mortgages allow the mortgagee to accelerate and the mortgagor to pay it off in one lump sum. (a) Applies upon Violation of a Mortgage Term Typically, a Clause would apply upon violation of a term in the mortgage, such as the term obligating the mortgagor to timely make periodic installment payments of a specified amount, to the mortgagee. For example, the Clause may provide that the entire unpaid balance will be payable to the mortgagee upon the mortgagor's uncured failure to make regular mortgage payments (i.e., default). (b) Other Events to which Clause Can Apply Alternatively, the Clause could apply in the event of these other events that would make the mortgagee wish to foreclose on the mortgage, such as the mortgagor's: • destruction of the property, • attempts to transfer or sell the land (e.g., due-on-sale clause) without the mortgagee's consent, • failure to maintain insurance on the property, • failure to pay property taxes, • failure to make payments on another mortgage on the same property, or • any other event that renders repayment insecure. (2) Invoking Acceleration Clause Unless the Clause states that it will automatically take effect, otherwise, once the Clause applies the mortgagee may elect whether to invoke the Clause. In that event, the mortgagee may lose that right of election when the mortgagor cures its default before the mortgagee invokes the Clause. (a) How to Cure Default; Goal of Cure A mortgagor can undo a mortgagee’s invocation of the Clause and avoid foreclosure by curing a default. The cure may result, for example, from making all past due payments and compensating the mortgagee for the costs of default. Such cure would place the mortgagee in the position it would have been in but for the mortgagor's default.

Page 94

AMERIBAR BAR REVIEW REAL PROPERTY MBE OUTLINES REAL PROPERTY 94 (b) Waiver of Invocation of Clause The parties may waive the right to invoke the Clause by either written agreement or by inducing another to detrimentally rely on the parties' conduct. (3) Clause Requires Notice of Default and Acceleration Commonly, a Clause requires that the mortgagee provide notice of default and acceleration, to the mortgagor. The notice would state the time period that the mortgagor has to cure the default. Sometimes, the notice is referred to as a breach and/or demand letter. Among other things, the notice would inform the mortgagor of the payment amount and deadline. (4) Effects of Mortgage Lacking Acceleration Clause Absent an effective Clause in a mortgage, • the mortgage payments may not be accelerated; • the mortgagee can only recover the current amount due from the mortgagor; and the mortgagee could either wait until every payment is due to foreclose on the mortgage or persuade a trial court to force a sale of some portion of the mortgaged real property for the mortgagee to receive the sale proceeds. 4) PARTIES TO THE PROCEEDINGS As a practical matter, parties to a foreclosure proceeding would include the mortgagee, the mortgagor, and any other party having an interest in the property subject to the mortgage in foreclosure (e.g., other person(s) having a lien on the property, i.e., another creditor having security in the property). a) Rights of Omitted Parties (1) Parties Possessing Right of Redemption If a foreclosure action does not include a party possessing a right to redeem a mortgage, that action does not eliminate the party’s interest. Rather, the foreclosure is void with respect to the party’s right. The omission of the party from a foreclosure does not prevent a purchaser at the foreclosure sale from succeeding to all of the real property owner’s rights. The omitted party may redeem the real property and title to it by paying the purchaser the amount of the mortgage debt. This rule also applies to junior lienholders who may exercise a right of foreclosure, redemption, or to recover on a surplus. (2) Junior Lienholders If a foreclosure action resulting in a sale does not include a junior lienholder, then the action does not automatically destroy the interest of the junior lienholder. This rule applies because the junior lienholder did not have the opportunity to be present at the foreclosure sale in order to maximize the price obtained in order to preserve his interest. Courts have primarily afforded such an omitted junior lienholder with two remedies under these circumstances. First, the junior lienholder could foreclose on the property again. The junior lienholder would be entitled to any funds in excess of funds originally owed to the senior lienholders. In that case, the original buyer would stand in the shoes of the senior lienholder. Second, the junior lienholder could pay the full amount due under the original senior lien on the property. As a result, the junior lienholder would be assigned the senior lienholder’s interest. Consequently, the junior lienholder would possess rights under both the senior and junior liens. b) Priority Among Mortgagees Typically, the type of recording statute used in the jurisdiction will determine priority. Usually, a prior properly recorded interest is senior to a subsequent properly recorded interest.

Page 95

AMERIBAR BAR REVIEW REAL PROPERTY MBE OUTLINES REAL PROPERTY 95 (1) Destruction of Junior Interests A foreclosure will destroy any interest junior to the interest being foreclosed upon if that junior interest was made a party to the foreclosure action (e.g., duly received notice of the action) and did not participate in it or take any other action. A senior interest to the one foreclosed upon, however, will remain intact. Therefore, any person who purchases property upon the foreclosure of a junior mortgagee will take the property subject to the interests of any and all senior mortgagees. As a practical matter, if the title to a mortgaged property purchased at foreclosure sale by a buyer other than the former mortgagor is free from any mortgage liens, the buyer could subsequently sell the property to the former mortgagor. If, for example, the buyer and the former mortgagor did not act in collusion with respect to this transaction, a former mortgagee of the property with a junior interest in the property--who duly received notice of the foreclosure and did not participate in it or take any other action--cannot successfully challenge this transaction on the basis of fraud. (2) Subordination Agreement (a) Definition A subordination agreement modifies the respective priorities of mortgage interests that the law and those documents provide. Such an agreement is subject to the recording statutes, which are detailed later. (b) General Considerations A subordination may be effective for any time duration. It may reverse the normal order of priority for liens. A subordination agreement may become effective upon the future execution of a mortgage that is adequately described. 5) DEFICIENCY AND SURPLUS a) Results of a Foreclosure Sale If a lawful forfeiture results in a sale of a mortgaged interest in real property, then at least three results could occur. First, if the amount that a buyer paid is equal to the balance that is due on the mortgage, then the mortgagee receives a full recovery. Second, if the amount that the buyer paid is less than the mortgage balance that is due, then a deficiency remains due to the mortgagee (i.e., lender). In that event, the mortgagor (i.e., borrower) remains liable to pay that balance due. Third, if the buyer pays an amount that is greater than the remaining mortgage balance, then a surplus exists. b) Allocation of Surplus The junior lienholders receive first payment of the surplus, and the mortgagor receives any residue. c) Priority among Multiple Mortgages Two or more simultaneously executed mortgages have equal priority to the surplus unless either a collateral agreement provides otherwise, or a recording act affects their priority. Two or more consecutively executed mortgages have priority in order of their sequence unless either a collateral agreement provides otherwise, or a recording act affects their priority. 6) REDEMPTION AFTER FORECLOSURE a) Statutory Right Some states provide a statutory right of redemption to a mortgagor whose mortgage went through foreclosure. Thus, the statute affords the mortgagor a specified time period in which to pay the mortgagee, at a minimum, the amount of

Page 96

AMERIBAR BAR REVIEW REAL PROPERTY MBE OUTLINES REAL PROPERTY 96 the deficiency on the mortgage. A statute might also require the mortgagor to pay the costs and fees of the foreclosure. Fulfillment of the statutory requirements entitles the mortgagor to the real property. b) When Mortgagee May Purchase a Mortgagor’s Equity of Redemption A mortgagee may purchase a mortgagor’s equity of redemption (the right to redeem the property) under the following circumstances: • The purchase occurs after the mortgage exists, not before or when it is created; • The transaction involved no fraud; • Sufficient consideration supports the transaction; and • The mortgagee carries the burden of proving the transaction’s fairness. V. TITLES A real property owner ordinarily possesses a deed. A deed is a documented form of title. However, title to real property is not necessarily represented by one document. Ownership may even be obtained without obtaining any documented form of title. For example, under the doctrine of adverse possession, ownership of real property may be obtained without a written instrument. A. Adverse Possession 1) DEFINITION A person in possession of real property may obtain ownership of the property under the doctrine of adverse possession. a) Elements In order to claim title by adverse possession, a person must prove possession that is: 1) open and notorious; 2) actual; 3) hostile; 4) continuous; and 5) exclusive; 6) for the required period of time. (1) Open and Notorious A person claiming ownership through adverse possession must be engaged in open use of the estate that is visible in an inspection of the estate. The claimant’s use of the estate must also be notorious or obviously evident. In other words, the claimant must engage in acts of possession consistent with owning the property in a manner which is capable of being seen. (2) Actual The claimant’s use of the estate must be actual use in the manner that an owner would use the estate. This use, however, does not need to be the estate’s best and highest use (3) Hostile The claimant’s use of the estate must be hostile. Courts determine hostile or adverse use by employing one of three tests. (a) Trespass Under the trespass test, a court would determine whether the claimant acted with a deliberate intention of obtaining title to real property while aware that the property belonged to another. Question Key 10,44,130,155

Page 97

AMERIBAR BAR REVIEW REAL PROPERTY MBE OUTLINES REAL PROPERTY 97 (b) Good Faith Under the good-faith test, a court would determine whether the claimant used the property based on a good faith belief that the non-owner held title to the property. (c) Objective Under the objective test, a court would determine whether the claimant used the property without permission as an owner would. The objective test disregards the non-owner’s subjective belief. • Permission Exception Generally, an owner’s permission to occupy the premises negates the element of hostile or adverse use unless the claimant’s occupancy exceeds the duration of the owner’s permission. For example, if the owner of a piece of property permits a friend to live on the property, the friend’s possession is not hostile. However, if the owner terminates the permission, and the friend remains on the land, the friend’s possession after the termination of permission is considered hostile. (4) Continuous The claimant’s use of the estate must be continuous and uninterrupted. (5) Exclusive The claimant’s use of the estate must be “exclusive” without sharing it with another. (6) Period The claimant’s possession must be for the legally-required period in the jurisdiction. (a) Typical Adverse Possession Period State statute usually dictates the period that is required for adverse possession to occur. The most common statutory periods are 7, 10, 15, or 20 years in length. The statutory period for adverse possession commences when a non-owner makes a hostile entry onto the premises. (as) Tolling (A) Disabilities Tolling of the statutory period may occur for the duration of an estate owner’s suffering from a legal disability such as imprisonment, infancy (i.e., age of minority), or legal incapacity. (B) Statutes A state statute may provide for a specific amount of time within which an estate owner may bring an action to challenge a claim of adverse possession after the legal disability ends. Certain states may require that the legal disability must have existed when the adverse possession began. For example, a statute may provide that the tolling period is 20 years, or until the owner reaches the age of majority, whichever occurs first. (b) Tacking of Statutory Period

Page 98

AMERIBAR BAR REVIEW REAL PROPERTY MBE OUTLINES REAL PROPERTY 98 Under the principle of tacking, a person claiming adverse possession of property may combine his period of possession with a prior possessor’s period of “continuous” use. In order to tack possession periods, all of the possessing parties must be in privity. (i) Bases of Privity A successor will be in privity with a prior possessor (predecessor) if the subsequent possessor (successor) obtains the real property by deed, devise, or descent. A successor who takes possession through a voluntary transfer from a predecessor may tack his period of possession from when the predecessor commenced possession. (c) Interruption of Statutory Period Interruption of adverse possession occurs when an estate owner retakes possession before the legally-required period of adverse possession expires. b) Constructive Adverse Possession (Color of Title) Constructive adverse possession applies when a non-owner takes possession of another’s tract of real property under color of title. The term color of title refers to a claim based on a written instrument such as a deed, a will, or a judgment that may have been defective and invalid. The person claiming adverse possession under color of title must have reasonably believed that the defective deed was valid. (1) Reasonable Connection Rule The adverse possession of part of an owner’s tract will be adequate to provide the non-owner title to the whole parcel of land, provided that a reasonable connection exists between that part of the parcel that is actually possessed and the entire tract. (2) Shorter Time Period for Color of Title In many jurisdictions, a claim for adverse possession under color of title will not need to meet the time period of possession required for a standard claim. Instead, a shorter period of time (such as 7 years) may be required. c) Marketability of Title As a general matter, adverse possession secures equitable title. Equitable title to property obtained by adverse possession is not marketable. Title that is obtained through adverse possession is usually deemed marketable only if an adverse possessor receives a judgment quieting title. A judgment quieting title will make the adverse possessor the owner of record, which can be the basis for issuing a deed that provides marketable legal title. d) Payment of Property Taxes The rule in a majority of states is that a property owner’s property tax payment does not defeat a claim for adverse possession. Additionally, under the majority rule, a claimant is not required to pay property taxes to establish adverse possession. However, in a minority of states, a claimant cannot establish adverse possession unless he pays property taxes. e) Government Land Government-owned land is exempt from adverse possession. B. Transfer by Deed Question Key 1,20,46,63,67,70,80,84,86,102,107,108,129,132,144,157

Page 99

AMERIBAR BAR REVIEW REAL PROPERTY MBE OUTLINES REAL PROPERTY 99 Most property ownership is evidenced by a deed. A deed is a document that facilitates the conveyance of a property interest between a grantor and a grantee. 1) REQUIREMENTS FOR DEED Generally, to be effective, a deed must be executed, delivered, and accepted. It should also be recorded. a) Execution For a deed to be valid, it must, at a minimum, set forth in writing: • the parties’ names; • an accurate legal description of the land; • language indicating an intention to convey the land; and • a signature by a grantor. A grantor’s signature on a deed fulfills the execution requirement. A deed is void if it lacks one of the four essential items listed above. The grantee is not required to sign the deed. (1) Necessity for a Grantee In order for a deed to be valid, a grantor must specifically identify a grantee in the deed. Although use of the grantee’s name may be sufficient, the grantor may also include, or be required to include, the grantee’s address for identification purposes. A deed to a grantee that does not exist is void. When the grantee’s name does not appear in a deed, a court may find the deed valid if it includes language that enables the court to determine the grantee’s identity, such as “to the grantor’s sister.” (2) Land Description and Boundaries As with sell and buy contracts, the Statute of Frauds requires that a deed contain a written identification and description of the land that it conveys. A more precise and accurate land description decreases likelihood of dispute over what land is conveyed and increases the likelihood of enforceability. For example, a statement that a grantor is conveying half of the land is an insufficient description of the land to allow for specific performance of a conveyance pursuant to a deed. (a) Official Descriptions The federal Government Survey System and the local subdivision plat are two types of official land descriptions. Official maps from local governments or boundaries described by metes and bounds are also valid types of descriptions. (b) Monuments and Watercourses The use of monuments or streams and rivers as boundaries may present difficulties with land descriptions because the monuments may be removed, and the streams and rivers may change course. If the shift in a watercourse boundary is gradual, whether caused by artificial or natural processes, the boundary will likely still follow the course of the water, so long as the benefiting owner did not act to deprive the other owner. (c) Correcting Descriptions

Page 100

AMERIBAR BAR REVIEW REAL PROPERTY MBE OUTLINES REAL PROPERTY 100 If a description is ambiguous, courts will allow the introduction of extrinsic evidence to aid in identifying the land; however, if there is no ambiguity, the grantor cannot introduce such evidence to show that he did not intend to convey the land described. (d) Reformation Doctrine (i) Majority Approach The majority of states recognize the doctrine of reformation, which allows for the modification of a written description that does not satisfy the parties’ intentions. The modification’s purpose is to fulfill the parties’ intentions regarding the boundaries of the conveyed premises. (ii) Minority Approach The minority of states only permit reformation to limit or decrease the size of a parcel of land. In that event, a reformed deed may not convey more or different property. b) Delivery For a deed to operate as a valid conveyance, a grantor needs to: • show intent to immediately transfer an interest in real property; and • engage in conduct or cause someone else to engage in conduct conveying the deed. (1) Intent Requirement Evidence of intent to convey the deed may include words or conduct showing an interest in making a transfer. Physical transfer of a deed from a grantor to a grantee demonstrates intent, although it is not a necessary step in making a delivery. For example, a grantor’s agent may deliver the deed to a grantee or the grantee’s agent. (2) Delivery Requirement The grantor must unconditionally deliver title to the grantee. Therefore, a grantor cannot condition delivery of title to a grantee upon the occurrence of a future event. For example, suppose a grantor attempts to convey property to a grantee when the grantor dies at some time in the future. A court would not enforce the transfer because the grantor lacked an effective present intent to deliver title. (a) Presumption against Delivery A presumption exists that there is no valid delivery of a deed if a grantor does not transfer the deed to a grantee. However, the parties may introduce evidence to overcome this presumption. (i) Parol Evidence Admissible Parol evidence is admissible to prove whether a grantor intended to make a present transfer of a deed. For example, testimony is admissible to show that a grantor instructed an agent to transfer a deed, but subsequently retrieved the deed before the agent delivered it to the grantee. (A) Detrimental Reliance If a written deed does not exist, a purported grantee must establish that the grantee acted in detrimental reliance upon an alleged grantor’s asserted promise to transfer the deed.

Page 101

AMERIBAR BAR REVIEW REAL PROPERTY MBE OUTLINES REAL PROPERTY 101 c) Acceptance A grantee has no obligation to accept a deed. The grantee must accept the deed in order to complete delivery. (1) Presumption of Acceptance A majority of states presume acceptance by a grantee if the deed would be beneficial to the grantee. A grantee must accept or reject a deed within a reasonable period after delivery. (a) Recording Not Required The fact that a grantee neither acknowledges a deed’s delivery and acceptance, nor records it, will not affect the deed's validity. (2) Rebutting the Presumption of Acceptance The courts may consider a grantee’s conduct and statements to ascertain whether the grantee accepted or rejected the delivery of a deed. Evidence of the grantee's contrary conduct may rebut the presumption of acceptance. d) Escrow Escrow is a means by which a grantor may transfer a deed to a grantee. The grantor places an executed deed in the possession of a third party that lacks any conflict of interest with either the grantor or the grantee. The third party holds the deed pursuant to instructions on the conditions for release of the deed. When a grantee fulfills those conditions, such as making a payment to the grantor, then the third party will deliver the deed to the grantee. (1) Delivery to Agent Insufficient The majority of courts hold that a grantor lacks a present intent to transfer title if, in an escrow situation, the grantor only provides a deed to the grantor’s agent. In that event, the grantor fails to make a delivery because the grantor could require the agent to return the deed instead of providing it to the grantee. (2) Types of Escrow (a) Sales Escrow A custodian that is a professional individual or a commercial entity processes a sales escrow. For certain purposes, and if specified conditions are fulfilled, a delivery of the deed by the custodian to a grantee will relate back to the date that the grantor put the deed in escrow. (i) Relation-Back Doctrine This relation-back capability allows for a closing to occur even if the grantor dies or becomes incapacitated prior to the closing date. The relation-back feature applies if a true escrow occurred under these two conditions: • A grantee and a grantor entered into an enforceable sale agreement; and • The grantor failed to reserve a right to recall the deed from its custodian. (b) Death Escrow A death escrow is an arrangement between a grantor and a layman custodian. A death escrow exists when a deed is put in escrow until the grantor dies and then the custodian provides it to the grantee upon the grantor's death. The

Page 102

AMERIBAR BAR REVIEW REAL PROPERTY MBE OUTLINES REAL PROPERTY 102 requirement for a death escrow to be valid is that the grantor not be able to control the deed. Thus, a death escrow would not be valid if the grantor possessed an express right to recall the deed. (i) Relation-Back Doctrine Generally, in a death escrow situation, the courts consider the grantor’s delivery of the deed to the custodian as immediately conveying title to the grantee, so the relation-back doctrine does not apply. e) Other Types of Conveyances (1) Reconveyance The courts in most jurisdictions will presume that a grantee accepted a grantor’s conveyance if it was legally sufficient in terms of execution, delivery, and acceptance. Thus, in order to reconvey title to the grantor, the grantee must use a new deed that includes all of the necessary formalities. In other words, the grantee cannot effectively reconvey title simply by returning the delivered deed to the grantor. (2) Conveyance by Gift A party may make a gift of real property. In order for the gift to succeed, the law requires a deed as well as the elements for a gift. The elements of a gift are 1) intent; 2) delivery; and 3) acceptance. The donor must possess a donative intent. Delivery (actual or constructive) must occur. Courts presume acceptance if the gift is beneficial, but the parties can rebut the presumption. Once complete, the donor or donee cannot revoke the gift. Thus, as with a conveyance for consideration, if the elements of a gift are satisfied, in order to reconvey title to the donor, the donee must use a new deed that includes all of the necessary formalities. (3) Conveyance by Dedication A landowner may choose to transfer an interest in real property to the public in the form of a dedication. Landowners may make dedications for public use for several different purposes including property dedicated for use as public schools, museums, streets, and parks. A dedication requires both an offer and acceptance to be valid. An oral offer of dedication does not fall under the Statute of Frauds. Therefore, such a dedication does not have to be in writing to be valid. The public entity must take affirmative action to accept. In other words, courts do not presume acceptance because public entities may want to avoid assuming the burdens, such as maintenance, that come with the property. 2) TITLE PROBLEMS a) Title Defects Some title defects occur at the time of the conveyance; however, parties may discover other title defects only after a conveyance occurs. In that event, a buyer may seek a remedy against a seller. The following types of defects may render a title defective: 1) complete claims to the land made by a third party that deprive the grantee of title, such as through adverse possession or a defect in the chain of title; and 2) undisclosed partial claims to the land, such as those of encumbrancers (easements, covenants, etc.). b) Defective Deed Issues (1) Deed Construction Rule In dealing with disputes concerning ambiguous provisions in a deed, many courts admit testimony in order to clarify those provisions. The courts generally resolve such ambiguities against a deed’s drafter.

Page 103

AMERIBAR BAR REVIEW REAL PROPERTY MBE OUTLINES REAL PROPERTY 103 (2) Deed Reformation If a formally correct deed contains inadvertent errors, a court will reform the deed in equity based on “clear and convincing” evidence. (a) Good Faith Purchaser Exception A court will not reform an erroneous deed in favor of a third party if a bona fide purchaser has relied upon that deed’s provisions. c) Voidable or Void Deeds If a deed involves defects regarding its execution or delivery, or that raise a question of a grantor’s capacity, then the grantor and his successors might seek a declaration that the deed is voidable or void. (1) Voidable Deeds (a) Fraudulent Inducement A deed is voidable due to a grantee’s fraudulent inducement by means of: • misrepresentation; • false financial statement; or • bad check. (b) Other Grounds for Deed Voidable Specific bases upon which a deed becomes voidable are: • execution by a minor; • lack of capacity; • breach of fiduciary duty; • insanity; • duress; • mistake; and • undue influence. A bona fide purchaser usually acquires valid title if a deed is voidable unless a minor executed the deed and disaffirms it in a reasonable time after attaining the age of majority. (2) Void Deeds A bona fide purchaser will not obtain title to a deed that includes a defect that renders it void. A deed is void under these circumstances: • lack of delivery; • forgery; • fraudulent execution; • fraudulent inducement; and • grantor’s illiteracy, senility, illness, and confusion.

Page 104

AMERIBAR BAR REVIEW REAL PROPERTY MBE OUTLINES REAL PROPERTY 104 3) TYPES OF DEEDS AND COVENANTS OF TITLE The main types of deeds include the quitclaim deed, the special warranty deed, and the general warranty deed. a) Types of Deeds (1) Quitclaim Deed A quitclaim deed includes no promises or covenants from a grantor to a grantee. Consequently, a grantee takes title “as is” under a quitclaim deed. (2) Special Warranty Deed A special warranty deed is a deed that includes six essential covenants (discussed below) for a grantee. The special warranty deed covenants are limited to remedying defects of title that are attributable to the transferor. It is a deed in which the grantor warrants, or guarantees the title, only against defects arising during the period of his ownership of the property. It does not warrant against any defects existing before that time. (3) General Warranty Deed A general warranty deed provides six primary covenants to a grantee that warrant against all defects of title. A grantor is liable for a breach of any covenant. The six primary covenants include three present covenants and three future covenants. Those covenants are detailed next. b) Covenants for Title Covenants are promises that relate to the real property. As indicated earlier, a warranty deed usually includes six covenants unless the terms of that deed explicitly exclude them. These covenants fall into two categories: present covenants and future covenants. (1) Present Covenants The present covenants of title are 1) of seisin, 2) right to convey, and 3) against encumbrances. A breach of the present covenants may only occur at the time when the conveyance occurs. The statute of limitations for lawsuits seeking to enforce present covenants starts to run when title to real property passes in a conveyance by deed. (a) Covenant of Seisin The majority of states consider a covenant of seisin as a promise that a grantor owns the estate that a deed purports to convey. The minority of states consider a covenant of seisin as a promise that a grantor has possession of land, whether or not that possession is wrongful. (b) Covenant of the Right to Convey The covenant of the right to convey provides that the seller warrants that he possesses the right to convey the property. Some minor differences exist between the covenants of seisin and of the right to convey. The right to convey is a promise that no valid restraint on alienation that applies to a real property interest prevents a grantor from making a conveyance (even assuming that he is the owner of the property). (c) Covenant against Encumbrances

Page 105

AMERIBAR BAR REVIEW REAL PROPERTY MBE OUTLINES REAL PROPERTY 105 Encumbrances are the rights or interests of third parties to which a deed is subject. Examples of encumbrances are restrictive covenants, leases, easements, mortgages, and liens. The covenant against encumbrances is a promise that the deed is free from such third party interests. (2) Future Covenants In a future covenant, the buyer can sue a prior grantor. In other words, the buyer can sue someone who is further up the chain of title if the prior grantor deeded the property with a future covenant. (a) Types of Future Covenants The three future covenants of title are of warranty, quiet enjoyment, and further assurances. They are promises that nothing will disturb a grantee’s possession. (i) Covenant of Warranty The covenant of warranty is that the grantor will warrant and defend the title of the grantee against rightful claims regarding the title conveyed. See e.g., Utah Code Ann. § 57-1-12. (ii) Covenant of Quiet Enjoyment According to the covenant of quiet enjoyment, a grantor warrants that the grantee may possess and quietly enjoy the land. See e.g., Utah Code Ann. § 57-1-12 (2000) (providing that grantor "guarantees the grantee, his heirs, and assigns . . . the quiet possession" of premises conveyed). (iii) Covenant of Further Assurances Under the covenant of further assurances, the grantor represents that he will do whatever is necessary in the future to establish title in the grantee. (b) Eviction There is no breach of future covenants until a grantee is disturbed in possession, which constitutes an eviction. In the context of future covenants, eviction possesses a different meaning than this word does in the context of landlord and tenant relations. Accordingly, the consequence of an eviction is not an actual loss of possession of the premises. (c) Enforcing Future Covenants No cause of action accrues until one in possession of a title superior to that of the grantee causes a grantee’s eviction. An “eviction” sufficient to trigger the protection may result from several occurrences including, for example: a judicial decree that the original grantee lacks title or must leave the property; another party’s interference with the original grantee’s possession; or the grantee’s relinquishment of possession of the real property to the third party. (i) Damages Generally, the grantor cannot be liable for damages amounting to more than the grantee paid for the property. When the breach is for encumbrances, the grantee may be able to receive damages in the amount of the diminution of value. (ii) Statute of Limitations The statute of limitations runs from the eviction date. The covenant of quiet enjoyment, however, runs with the land. Thus, the statute of limitations does not apply to actions contesting an eviction based on a breach of the covenant of

Page 106

AMERIBAR BAR REVIEW REAL PROPERTY MBE OUTLINES REAL PROPERTY 106 quiet enjoyment. Therefore, a remote grantee may bring a lawsuit against an initial grantor seeking damages if a third party claimant interferes with the grantee's possession and enjoyment. (iii) Liquidated Damages A party may enforce a deed’s liquidated damages clause if it is reasonably related to the amount of damages that were foreseeable when the parties entered into the contract. 4) DRAFTING, REVIEW, AND NEGOTIATION OF CLOSING DOCUMENTS a) Contract and Closing Are Two Phases of Sale of Land A typical sale of land has two main phases. First, a seller and a buyer execute a sell and buy contract. Second, a closing occurs. b) Closing Is Last Phase; Parties Fulfill Contract Duties The closing is the last phase of a real estate sell and buy transaction. At the closing, the parties fulfill their duties under the buy and sell contract. Fulfillment of the contract is conditional upon the title being marketable and the financing being obtained before the closing. The parties enter into the contract initially, to allow the buyer to verify the title and obtain financing before the closing. c) Between Phases, Buyer Can Obtain Financing, Examine Title, and Develop Closing Documents Between the two phases, the buyer can obtain financing, examine the title to the land, and develop the closing documents with or without professional assistance (e.g., financiers, lawyers). (1) Lawyers Can Negotiate, Draft, and Review Closing Documents Between the two phases, the parties to the contract can each have their respective lawyers negotiate, draft, and review the closing documents (i.e., documents that the parties expect to execute at closing). As a practical matter in residential real estate transactions, in most states lawyers are not often involved in preparation for the closing or the closing itself. Joseph William Singer, Introduction to Property, 2d ed., Aspen Publishers, 2005, p. 511. (2) Lawyers and Insurers Can Examine Title Between the two phases, the buyer can have its lawyer and/or a title insurance company examine the title to the land for any defect(s) of title. d) Issues Can Surface between the Two Phases Between the two phases, issues might surface such as inability to obtain financing, discovery of a title defect, disagreement over the closing documents' terms and conditions, or destruction of a structure on the land subject to the contract. Some sections of this outline address such issues. The closing can occur in the absence of such issues or after their resolution. e) What Parties Exchange at the Closing At the closing, the seller provides a deed of title and receives payment from the buyer. Id. Also, the parties exchange documents regarding financing (e.g., mortgage or promissory note), title insurance, and any other related affidavits and regulatory forms. Id.; West's Encyclopedia of American Law, 2d ed., The Gale Group, Inc., 2008. f) Other Issues Can Surface after the Two Phases

Page 107

AMERIBAR BAR REVIEW REAL PROPERTY MBE OUTLINES REAL PROPERTY 107 After the closing, issues might surface such as the seller's liability for a defective structure on the land or matters regarding title to the land. Other sections of this outline address those issues. 5) PERSONS AUTHORIZED TO EXECUTE REAL ESTATE DOCUMENTS a) Power of Attorney in General; Agency Concepts Relevant Power of attorney is a legal document authorizing one person to act on behalf of the other person who grants that power of attorney. Black's Law Dictionary, 2d ed. The grantor is the principal and the grantee is the agent. Thus, general Agency Law concepts are relevant, such as the agent's duties to the principal. The agent may be referred to as an "attorney-in-fact," even though the agent may or may not be an actual "attorney-at-law" having a license to practice law in a jurisdiction as a lawyer in real fact. Common types of power of attorney include general, special (i.e., limited), or durable. (1) General Power of Attorney: All Legal and Business An agent having general power of attorney agrees to act as proxy in all legal and business matters for the principal. Black's. For example, the agent can execute contracts and legal documents for the principal. Hill and Hill, The People's. Commonly the agent acknowledges, before a notary public, that the agent signed that document, to make the document recordable in a property transaction per law. Id. Many states' law requires recordation of a power of attorney regarding property transactions. (a) Ineffective on Grantor's Death or Incapacity A general power of attorney becomes ineffective by its terms or by operation of law when the principal dies or becomes incapacitated, unless its document states that the power of attorney is durable, as detailed later. (2) Special Power of Attorney: Only Specified Matters An agent having special power of attorney agrees to act as proxy only in specified matters for the principal, such as in a particular property transaction. Id.; Black's. (3) Power of Attorney of Property A "power of attorney of property" grants the agent the right to manage the principal's properties, as the agent deems fit. Black's. (4) Durable Power of Attorney for Real Estate A durable power of attorney may be used specifically for real estate dealings. Black's. It differs from a general power of attorney essentially in that the durable power of attorney may remain in effect even after the principal dies or becomes incapacitated. Id. (a) It May Spring into Effect A springing durable power of attorney comes into effect under the circumstances specified in its document, such as: • after expiration of specified duration (e.g., one year from an anniversary date); • when a particular event does not happen (e.g., the principal does not pay taxes); or • when a particular event happens (e.g., the principal becomes incapacitated). Id. (b) It May Take Effect Immediately An immediate durable power of attorney takes effect upon execution of the document creating it. Id.

Page 108

AMERIBAR BAR REVIEW REAL PROPERTY MBE OUTLINES REAL PROPERTY 108 b) When Power of Attorney May End A power of attorney may end upon a date stated in its source document or by written cancellation (i.e., revocation). Hill and Hill, The People's. The principal may revoke the power of attorney at any time that the principal is competent. In addition, if the document does not designate a successor agent, then the power of attorney would end upon the initial agent's death. C. Transfer by Operation of Law and by Will 1) IN GENERAL Unlike a will, a deed cannot provide that it must take effect upon a grantor’s decease. A deed and a will lack the same operative effect because they involve very different formalities. A grantor who wishes to make a conveyance effective upon his death must use a testamentary instrument, rather than a deed, to fulfill that purpose. Such an instrument may not be recorded or appear in a chain of title, which may create issues of conflicting claims of ownership under a recording act. 2) ADEMPTION a) Doctrine of Ademption by Extinction Under the doctrine of ademption, if the subject matter of a specific devise is not in the probate estate at the time of the testator's death, the bequest to the devisee adeems (i.e., fails). See generally William H. McGovern, Jr. and Sheldon F. Kurtz, Wills, Trusts, and Estates, p. 295, (2d ed. 2001). The doctrine of ademption by extinction applies only to specific devises. It does not apply to general or demonstrative devises. For example, suppose a testator’s will leaves a fee simple estate to a beneficiary. Before the testator’s death, the testator sells that estate to a third party. Under the general rule, the devisee receives nothing because the gift adeems by extinction. (1) Modification by Intent Theory Some courts will temper this result and implement an intent theory to determine if the testator intended for the gift to adeem. If a beneficiary can prove that the testator did not intend for the gift to adeem, then the beneficiary will be entitled to a gift of equal value to the one that adeemed. 3) EXONERATION a) Common Law Under the common-law doctrine of exoneration, a specific beneficiary of encumbered real property is entitled to have a mortgage on the property paid from the estate as a debt of the decedent unless the evidence indicates that the testator had a contrary intent. Martin v. Johnson, 512 A.2d 1017 (D.C. 1986). b) Statutory Law Today, many states have adopted statutes contrary to this common-law rule of exoneration. For example, the Uniform Probate Code provides that "a specific devise passes subject to any mortgage interest existing at the date of death, without right of exoneration, regardless of a general directive in the will to pay debts." Unif. Probate Code § 2-607. In states with statutes of this type, the specific devisee of encumbered real property takes it subject to a mortgage notwithstanding the fact that a will contained a clause directing an executor to pay the decedent's debts. 4) LAPSE Question Key 45

Page 109

AMERIBAR BAR REVIEW REAL PROPERTY MBE OUTLINES REAL PROPERTY 109 If a devisee named in a testator’s will predeceases the testator, the devise lapses into the residuary estate unless the jurisdiction’s anti-lapse statute preserves the devise for the devisee’s descendants. In many jurisdictions, a devise to a person who is a descendant of the testator's grandparents does not lapse if the deceased person's descendants survive the testator. Rather, a substituted gift is created in the deceased person's descendants. For example, suppose Terrence passes away, leaving a will. The will provides that his stock in IBM is to be left to his cousin Sally. Unfortunately, Sally died just a few months before Terrence, and Terrence had not changed his will. Normally, the IBM stock would adeem to the estate and pass by intestacy. Many jurisdictions, though, have anti-lapse statutes that save a gift made to a descendant of a grandparent (some statutes deviate from this rule and will save gifts made to a descendant of parents or even descendants of the grandparents of a testator’s spouse). If the jurisdiction possesses such a statute, then the gift of the IBM stock would be made to Sally’s descendant(s). Therefore, absent a contrary intent in the will, Melanie, Sally’s only daughter, would receive the stock. D. Title Assurance Systems 1) GENERAL CONSIDERATIONS The existence of and access to a public recording system does not usually provide a title researcher with a definitive summary statement as to all interests that exist in certain real property. Rather, those systems provide a repository for all documents that reflect ownership interests in and encumbrances upon the real property. a) Indexes Individuals may record deeds of conveyance in either of two types of systems: • indexes of grantor-grantee and grantee-grantor; or • a tract index that lists properties by geographic location. Most states possess an index of grantors and an index of grantees. In theory, one would perform a title search by beginning with the grantor index. One would look up the name of the first recorded owner of title to property. This owner is usually the government. The search would attempt to locate the first grant from the government to the first grantee. The next step is to look up the grantee's name in the grantor index to locate the deed by which it subsequently conveyed the title to the property. The search is continued with each subsequent grantor until the searcher finds no more grants. An alternative method is to reverse the process. Thus, one may search backward in the grantee index. An individual would conduct this search by beginning with the name of the person or entity who is thought to own the land to find the grantor to it. Then the individual would search the grantee index again to find the source of that grantor's title, and so on until reaching the grant from the government. These links from grantor to grantee are called the “chain of title.” The last grantee found is the “record title holder.” b) Chain of Title A prospective purchaser must research the chain of title. The holder of record-title is not automatically the owner of the land. For example, if a person conveys title to property to someone else, even though it is not recorded, he may have nothing left to transfer to any subsequent person. As a result of the various state recording laws, though, the courts will protect a purchaser who pays valuable consideration and does not have knowledge of the prior unrecorded deed from the claims of a prior grantee under that deed. 2) RECORDING ACTS a) Types Question Key 8,18,19,37,43,46,56,71,94,105,119,139,141,142,148,156,159,165

Page 110

AMERIBAR BAR REVIEW REAL PROPERTY MBE OUTLINES REAL PROPERTY 110 In the absence of a state recording act that establishes the priority of conveyances of the same interest in real property from a common grantor to different grantees, the actual sequence of the delivery of each deed may establish that priority. The recording acts may operate to alter that common law approach of “first in time, first in right” and its results. The three primary types of acts are race, notice, or race-notice. (1) Pure Race Recording Act Some states have a pure race recording act (“act”). A pure race recording act provides that a buyer for value who is first in time to record an acquired interest in real property possesses a priority interest over any other buyers. The act is a derivative of the first-in-time rule. The act applies to situations when a grantor of a real property interest conveys a deed to a first buyer and then subsequently conveys another deed for the same interest to a second buyer. Of those two buyers, the one who records first will prevail. (2) Notice Recording Act Some states have a notice recording act. A notice recording act provides that regardless of which of multiple buyers for value first records, a subsequent bona fide purchaser for value of an interest in real property will prevail if that purchaser lacked notice of the other buyers’ prior interests. That purchaser will also prevail over a grantee or devisee who fails to record his interest. (3) Race-Notice Recording Act Some states have a race-notice recording act (“act”). This type of act allows a subsequent buyer for value without notice of a prior interest in the same real property to prevail over other prior buyers for value. Under a race-notice act, a subsequent buyer of a real property interest generally will only prevail if he is a bona fide (i.e., good faith) purchaser, for value, who also records before the initial buyer. Typically, in a race-notice jurisdiction, a new buyer has a grace period after the sale in which to record. A grace period would prevent the prior buyer who failed to record from recording immediately upon finding out about the sale to the new owner but before the new owner has the time to record the conveyance. Under this type of act, usually in a dispute between a subsequent good faith buyer relying upon the record title and a prior grantee (e.g., a purchaser or donee) of a previously unrecorded conveyance of the same real property, the subsequent good faith buyer prevails if he acquired the property without knowledge or notice (i.e., actual, constructive, or record) of the prior grantee’s interest. However, when a party has obtained equitable title to the real property by means of adverse possession before the good faith buyer obtained legal title from the record title owner, this equitable title cannot be defeated by the later conveyance from the record title owner to the good faith buyer, even though the real property is vacant when the buyer acquires it. QUESTION A creditor received a valid judgment against a debtor and promptly and properly filed the judgment in the county. Two years later, the debtor purchased land in the county and promptly and properly recorded the warranty deed to it. Subsequently, the debtor borrowed $30,000 from his aunt, signing a promissory note for that amount, which note was secured by a mortgage on the land. The mortgage was promptly and properly recorded. The aunt failed to make a title search before making the loan. The debtor made no payment to the creditor and defaulted on the mortgage loan from his aunt. A valid judicial foreclosure proceeding was held, in which the creditor, the aunt, and the debtor were named parties. A dispute arose as to which lien has priority. A statute of the jurisdiction provides: “Any judgment properly filed shall, for 10 years from filing, be a lien on the real property then owned or subsequently acquired by any person against whom the judgment is rendered.” A second statute of the jurisdiction provides: "No unrecorded conveyance or mortgage of real property shall be good against subsequent purchasers for value without notice, who shall first record."

Page 111

AMERIBAR BAR REVIEW REAL PROPERTY MBE OUTLINES REAL PROPERTY 111 Who has the prior lien? A) The aunt, because a judgment lien is subordinate to a mortgage lien. B) The aunt, because she is a mortgagee under a purchase money mortgage. C) The creditor, because its judgment was filed first. D) The creditor, because the aunt had a duty to make a title search of the property. The correct answer is choice (c). This is a race-notice jurisdiction which protects a bona fide purchaser for value without notice who records first. The creditor filed first, giving the aunt constructive notice of the judgment lien. Accordingly, the judgment lien has priority. b) Types of Notice Notice is an essential element in determining rights of priority that arise from a conveyance of an instrument that reflects an interest in real property, such as a deed. The types of notice are as follows. (1) Constructive Notice Recording acts charge grantees with constructive notice of all correctly recorded prior conveyances of interest in real property that affect the same real property. In other words, a buyer is considered to possess constructive notice of recorded prior conveyances that appear in the chain of title. A person does not have constructive notice of a conveyance that is outside the chain of title. (a) Collateral Documents Rule The collateral documents rule provides that a grantee of real property possesses constructive notice of the contents of instruments regarding adjacent parcels of land if those parcels were conveyed by a common owner. (2) Record Notice Record notice exists if a reasonable review of public records would reveal a previously recorded instrument. (3) Inquiry Notice Inquiry notice occurs when a reasonable investigation would reveal facts about claims regarding real property if a buyer is aware of circumstances or facts that would lead a reasonable person to inquire further. For example, a potential buyer could visit the real property before making a purchase to determine who is in possession of it, if it contains defects, and if it matches the description of the land and any structures that are on the land. Thus, recording acts charge a buyer with actual knowledge of those facts that would have been discovered through a reasonable investigation. (4) Actual Notice Actual notice arises from knowledge that one literally obtains from another person. For example, a seller communicates to a buyer that, before a closing occurs on their contract to buy and sell real property, the seller will remove an encumbrance from title to that property. (5) Imputed Notice

Page 112

AMERIBAR BAR REVIEW REAL PROPERTY MBE OUTLINES REAL PROPERTY 112 Imputed notice applies only to certain relationships and means that if one person in that relationship possesses certain knowledge, then another person in that relationship is imputed to possess the same knowledge. Imputed notice may run from agent to principal, employee to employer, or spouse to spouse. (6) Fraudulent Conveyances and Bona Fide Purchasers If a grantee obtains an interest in real property from a grantor by fraudulently inducing that conveyance, then the actual owner of a real property interest may void the conveyance in a civil action against the grantee. The actual owner, though, will not prevail in such an action against a bona fide purchaser from the grantee. c) Bona Fide Purchasers Courts consider mortgagees to be buyers under the recording acts because, like ordinary buyers of real property, they provided value for their interest in real property. Mortgagees and ordinary buyers may be bona fide purchasers. (1) Statutory Coverage The recording acts will only protect bona fide purchasers who: • have obtained an instrument of conveyance by giving valuable consideration; • without notice of another party’s prior deed that affects title to the same real estate. In addition to specific state statutory protections of bona fide purchasers, such as under recording acts and the Uniform Commercial Code, the common law provides certain protection to such a purchaser. Each jurisdiction provides different rules regarding the rights of a bona fide purchaser relative to previous buyers of the same interest. (2) Recorded Instruments (a) No Automatic Bona Fide Purchaser Status Most recording acts charge a subsequent buyer of a real property interest with constructive notice that another buyer previously recorded an instrument of conveyance for the same interest, despite the subsequent buyer’s failure to search the records. Thus, the subsequent buyer is ineligible for bona fide status and protection if the previous grantee properly registered the previously recorded instrument and entered it in the appropriate governmental office. (b) Bona Fide Purchaser Status Conversely, the subsequent buyer may be eligible for bona fide status if the previous grantee did not record the undiscovered instrument or properly register or enter it in the appropriate governmental office. (c) Instruments Outside the Chain of Title A previously recorded instrument must exist and appear within the chain of title. If a reasonable person is unable to locate the instrument after a standard title search, then that instrument does not afford constructive notice because it does not exist within the chain of title. (i) Wild Deed In an ordinary wild deed situation, someone other than the buyer’s grantor creates a prior interest in the same real property outside of the chain of title. A wild deed is a deed that is either not recorded within the buyer’s chain of title Question Key 8,43,71,141,142

Page 113

AMERIBAR BAR REVIEW REAL PROPERTY MBE OUTLINES REAL PROPERTY 113 or recorded out of sequence (e.g., too late or too early). Thus, recording acts do not charge a buyer with notice of the wild deed because the wild deed is deemed unrecorded. (ii) Improperly Indexed Deed The instrument must be correctly indexed. If a public servant improperly filed the instrument that exists outside of the proper chain of title, then it does not exist for the purpose of affording constructive notice to a subsequent buyer. Not every properly recorded instrument necessarily affords constructive notice to the world. In other words, the instrument may be ineffective to provide notice. (d) Effective Documents An instrument reflecting an interest in real property must be effective in order to provide notice. A deed, for example, will not be effective if it lacks any of the minimum necessary elements such as the parties’ identities, a statement of a grantor’s intent to convey, a sufficient property description, and the grantor’s signature. For example, if the grantor executes a deed that does not identify the grantee or describe the land, the deed will only become effective to convey title if the grantee's name and a description of the land are added. Until then, the deed is ineffective. (3) Parties Protected Obtaining bona fide purchaser status will protect new buyers (those who pay value) of interests in real property if a diligent title search fails to disclose a prior recorded interest in real property. Recording such an interest to give constructive notice to the world protects the present owners of those interests from losing them to subsequent buyers. d) Hidden Risks (e.g., undelivered or forged deed) (1) Undelivered Deed If a deed is not delivered to a grantee as detailed earlier in this outline, then the grantee cannot record it. (2) Forged Instruments The courts consider forged instruments to be completely void. Thus, a forged instrument has no legal effect and cannot convey any interest in real property to a grantee or a bona fide purchaser in the chain of title. (a) Examples of Forged Deeds Forged instruments include, for example, any type of deed of title on which: 1) the grantor's signature is false; or 2) an effort is made to increase a deed's scope by an alteration, deletion, or addition after it is executed but prior to delivery. William B. Stoebuck and Dale A. Whitman, The Law of Property, 3d ed., West Group, 2000, p. 817. An example of a forged instrument is a deed that was executed pursuant to the authority of a forged power of attorney from the grantor. (b) Forged Deed Could Implicate Some Legal Theories Under the right circumstances, a forged deed's grantee could obtain title by adverse possession. Id. A forged deed's purported grantor's conduct could bring equitable estoppel into play. Singer, Introduction, p. 541. 3) TITLE INSURANCE Title insurance insures a real property buyer or a secured lender (e.g., mortgagee) (the "Insured") against loss resulting from title defects. Black's Law Dictionary, 2d ed. A title defect exists where, for example, a former owner of a parcel

Page 114

AMERIBAR BAR REVIEW REAL PROPERTY MBE OUTLINES REAL PROPERTY 114 of property has a claim concerning the parcel that was sold at a tax sale that happened without constitutionally required notice of the sale to the former owner who was delinquent in paying property taxes. a) Amount of Coverage The Insured buys the title insurance coverage in amount of the property's purchase price. West's Encyclopedia. The coverage dollar value may be referred to as the face amount of the title insurance policy (the "Policy"). b) One-Time Premium Payment at Closing To obtain coverage, on the closing date, the property buyer or seller makes a one-time premium payment in full to the title insurance company (the "Insurer"). Id. (1) Coverage Is Effective While Property Is Held The coverage remains effective as long as the property is held by the buyer, the buyer's devisees, heirs, or other successors. Stoebuck and Whitman, Property, p. 918. c) Title Insurance Policy (of Indemnity) Before the Insurer issues a Policy, it examines the record title regarding the property for any errors or title defects. Id. The Insurer may issue a preliminary report or binder identifying the title defects that the Insurer will cover. Stoebuck and Whitman, Property, p. 919. When the Insurer issues a Policy, the Insurer may provide a certificate of title indicating status of the property's title. West's Encyclopedia. The Policy of indemnity identifies the persons in whom title is vested and lists those title defects that it will not cover. Stoebuck and Whitman, Property, p. 917. (1) Record Title Record title means documents affecting property that are properly recorded in the Insured's "chain of title." (a) Defect within or outside of Chain of Title A title defect can exist within or outside of the insured's chain of title. The chain of title includes properly recorded documents. The chain of title can contain a defect such as a recorded, unpaid mortgage. The chain of title would not include a defect such as an unrecorded, private arrangement between the property owner and a third party regarding property that later the property owner agrees to sell to another buyer. (2) What Policy Insures The Policy insures the title's status in relation to its owner, the Insured. The Policy insures that a "good record title" of the property exists as of the Policy's effective date. (a) Policy Insuring Good Record Title A Policy may insure only a good record title where the Insurer's examination neither extends beyond the record title nor involves inspecting the property. Such a Policy does not insure against any documents outside of the Insured's chain of title. (b) Policy Also Insuring Hidden Title Defects In addition to insuring good record title, a Policy may specify that it covers "hidden" title defects that would not be discovered in the Insurer's examination (e.g., deed of title having a grantor's forged signature) within the chain of title. (3) What Policy Warrants: No Defects, Name, Transfer The Policy warrants that

Page 115

AMERIBAR BAR REVIEW REAL PROPERTY MBE OUTLINES REAL PROPERTY 115 • the title is free from defects (at least within the chain of title and maybe others), • the title is in the owner's name, and • the owner may transfer (e.g., sell or convey) the property. Hill, Law Dictionary. (4) What Policy Does Not or Will Not Cover The Policy does not cover defects that surface after the closing. The Policy will not cover its express exclusions and exceptions. (a) Exclusions from Coverage A Policy may include exclusions for: • laws (e.g., zoning ordinances); • exercise of eminent domain rights not appearing in the public record; • title matters agreed to or created by the Insured; and • issues known to the Insured upon acquiring the property or upon the Policy's effective date, of which the Insurer lacks notice. Stoebuck and Whitman, Property, p. 921. (b) Exceptions to Coverage A Policy includes exceptions for known claims and encumbrances. A Policy may include exceptions for these matters not appearing in the public record (e.g., recorded chain of title): • claims or rights of parties in possession, • easements, • public rights of way, • air, water, and mineral rights; • mechanic's liens, • property assessments or taxes, and • issues that a land survey (i.e., property description) would discover. Id. d) Insurer's Obligations to Insured (1) When Insurer Must Indemnify Insured The Policy requires the Insurer to indemnify the Insured, by paying damages, in the event that the title is subject to title defects, or vested differently than as, when the Policy was issued. Stoebuck and Whitman, Property, pp. 917, 921. (2) Insurer Must Cure Title Defects and Defend Insured

Page 116

AMERIBAR BAR REVIEW REAL PROPERTY MBE OUTLINES REAL PROPERTY 116 The Policy obligates the Insurer to cure title defects as feasible and defend the Insured should litigation arise from them. Id., p. 921; Singer, Introduction, p. 562. If a title defect issue surfaces after the property is sold or conveyed to the Insured, then the Insurer will take steps to resolve the issue or pay for related damages incurred by the Insured. Id. e) When Insurer Is Liable for Claim by Insured The Insurer is liable to the Insured for its lack of skill, care, or diligence when it issues a title certificate. West's Encyclopedia. (1) Amount of Damages for Insured's Loss Pursuant to the Insured's claim against the Insurer, the Insured may recover damages from the Insurer only for a loss. Stoebuck and Whitman, Property, p. 922. (a) Complete Failure of Title--Policy's Amount On a claim for the property's value arising from a complete failure of title, the Insured may recover up to the Policy's face amount. Id.; West's Encyclopedia. (b) Encumbrance Payable by Sum Certain On a claim for an encumbrance that can be paid off by a specified sum certain, the Insured may recover the sum. Stoebuck and Whitman, Property, p. 922. (c) Other Encumbrances On a claim for any other encumbrances (e.g., restrictive covenants or easements), the Insured may recover the difference in the property's value without or with the encumbrance, up to the Policy's face amount. Id. (d) Policy Issued to Lender as Insured Where the Policy is issued to a lender as the Insured, then the Insurer may fulfill its duty by (A) paying the Insured the loan balance plus interest and costs and (B) taking an assignment of the mortgage and note. Id. E. Special Problems 1) AFTER-ACQUIRED TITLE a) Estoppel by Deed The doctrine of estoppel by deed creates an implied covenant to convey title when a grantor receives interest in conveyed property after delivery (i.e., after-acquired title). The doctrine applies under two conditions: • A grantor conveys a greater interest in real property than which the grantor owns; and • The grantor subsequently acquires the interest . The doctrine prevents the grantor from asserting ownership to that interest against the grantee. The grantee may choose to either pursue the action for damages or accept transfer of the title. 2) JUDGMENT LIEN a) Definition Question Key 12

Page 117

AMERIBAR BAR REVIEW REAL PROPERTY MBE OUTLINES REAL PROPERTY 117 A judgment creditor is a prevailing party in a civil action for damages. For example, suppose Carl sues Daryl. Before the court enters judgment, Daryl conveys his real property interest to Tom. Tom does not record the conveyance. Carl then obtains a judgment against Daryl. What would happen to the interest in the real property? (1) Minority Rule A minority of jurisdictions would not protect a third party’s interest (Tom’s) over a judgment creditor. (2) Majority Rule A majority of jurisdictions would not protect a creditor’s lien interest (Carl’s) over that of the third party (Tom’s). A judgment creditor lacks priority compared to a grantee of real property before the court enters judgment. (a) Judgment Liens Most jurisdictions do not consider a judgment creditor a bona fide purchaser for value. Thus, a judgment creditor may only apply a judgment lien against property that a debtor owns when the court enters judgment, excluding other conveyances, unless a recording act affords protection to judgment liens. 3) TAX LIEN A tax lien is similar to a judgment lien because it is also imposed involuntarily. a) Federal Tax Lien (1) Represents Debt Imposed for Unpaid Federal Taxes A federal tax lien represents a debt imposed for unpaid federal taxes. Black's Law Dictionary, 2d ed. (a) Lien Is Used to Obtain Payment of Taxes A tax collector may use a federal tax lien to obtain payment from a delinquent taxpayer. (2) Nonpayment after Demand Would Result in Lien (a) For Taxes, Interest, Penalty, Cost Accrued The federal government may impose a tax lien on property for unpaid taxes in the amount of taxes plus interest, penalty, and costs. 26 U.S.C. § 6321. (3) Record Notice of Tax Debt by Filing in Office A federal tax collector provides record notice of tax debt by recording a lien in the appropriate local government office. 26 C.F.R. 301.6323(f)-1(a)(1)(i), (b)(1)(i). (4) Tax Lien Is Encumbrance Preventing Purchase A tax lien is an encumbrance on title to property. As a title defect, existence of a tax lien prevents purchase of the property. (5) Release of Lien A lienholder may release a lien.

Page 118

AMERIBAR BAR REVIEW REAL PROPERTY MBE OUTLINES REAL PROPERTY 118 (6) Enforcement of Tax Lien (a) Auction of Taxpayer Property A tax collector may sell taxpayer property by auction to collect unpaid tax debt. Id. 4) PURCHASE-MONEY MORTGAGE a) Right of Priority A purchase-money mortgage receives priority over competing claims against the property. This right of precedence in the purchase-money mortgage exists even if a third party, such as a lender, funded the purchase. Thus, a purchase-money mortgage receives precedence over the following claims: • homestead; • community property; • dower; • judgments entered prior to execution of the purchase-money mortgage; • a vendor’s lien; and • other mortgages created prior to or concurrently with the purchase-money mortgage. 1) Exceptions A recording act may provide exceptions to the rule of priority for purchase-money mortgages. A majority of jurisdictions protect a mortgagee from an earlier purchase-money mortgage that was not recorded if: • The mortgagee took the mortgage without notice; and/or • The mortgagee recorded its mortgage first (before the other purchase-money mortgagee).